Manual of Clinical Cases in ENT and Head-Neck Surgery [First Edition] 9789389688221

465 115 45MB

English Pages [271] Year 2021

Report DMCA / Copyright

DOWNLOAD FILE

Polecaj historie

Manual of Clinical Cases in ENT and Head-Neck Surgery [First Edition]
 9789389688221

Table of contents :
Cover
Half Title Page
Title Page
Copyright
Preface
Acknowledgments
Contents
Section I: Otology
1. Clinical Anatomy of Ear
2. Diseases of External Ear
3. Otitis Media
4. Facial Nerve
5. Otosclerosis
6. Trauma to Middle Ear
7. Middle Ear Tumor
8. Meniere’s Disease
9. Acoustic Neuroma
10. The Deaf Child
11. Hearing Loss
12. Vertigo
Section II: Nose and Sinus
13. Diseases of the External Nose
14. Diseases of the Nasal Septum
15. Epistaxis
16. Allergic Rhinitis
17. Sinusitis
18. Nasal Polypi
19. Granulomatous Diseases of the Nose
20. Neoplasms of the Nose and Sinus
21. Headache
22. CSF Rhinorrhea
23. Foreign Body in Nose
Section III: Head and Neck Diseases
24. Oral Cavity
25. Tonsils and Adenoids
26. Tumours of Oral Cavity and Oropharynx
27. Neck Mass
28. Vocal Cord Paralysis
29. Nasopharynx
30. Tumors of Hypopharynx
31. Tracheostomy
32. Congenital Lesions of Larynx
33. Infections of Larynx
34. Foreign Body in Airway
35. Hoarseness of Voice
36. Laryngeal Growth
37. Esophagus
38. Neck Abscess
Index
Back Cover

Citation preview

Manual of

Clinical Cases in

ENT and Head–Neck Surgery A Stepwise Approach According to the latest CBME Guidelines | Competency Based Undergraduate Curriculum for the Indian Medical Graduate

ii

Manual of Clinical Cases in ENT and Head–Neck Surgery

iii

Clinical Anatomy of Ear

Manual of

Clinical Cases in

ENT and Head–Neck Surgery A Stepwise Approach According to the latest CBME Guidelines | Competency Based Undergraduate Curriculum for the Indian Medical Graduate

Santosh Kumar Swain MS, DNB, MNAMS, Fellowship in Neuro-otology (SGPGI, Lucknow), Fellowship in Endoscopic Ear Surgery (Germany)

Professor and Head Department of Ear, Nose and Throat and Head and Neck Surgery Institute of Medical Sciences and SUM Hospital Siksha “O” Anusandhan University, Bhubaneswar Odisha, India

CBS Publishers & Distributors Pvt Ltd New Delhi • Bengaluru • Chennai • Kochi • Kolkata • Lucknow • Mumbai Hyderabad

• Jharkhand

• Nagpur • Patna • Pune

• Uttarakhand

Disclaimer Science and technology are constantly changing fields. New research and experience broaden the scope of information and knowledge. The authors have tried their best in giving information available to them while preparing the material for this book. Although, all efforts have been made to ensure optimum accuracy of the material, yet it is quite possible some errors might have been left uncorrected. The publisher, the printer and the authors will not be held responsible for any inadvertent errors, omissions or inaccuracies. eISBN: xxxx Copyright © Authors and Publisher First eBook Edition: 2022 All rights reserved. No part of this eBook may be reproduced or transmitted in any form or by any means, electronic or mechanical, including photocopying, recording, or any information storage and retrieval system without permission, in writing, from the authors and the publisher. Published by Satish Kumar Jain and produced by Varun Jain for CBS Publishers & Distributors Pvt. Ltd. Corporate Office: 204 FIE, Industrial Area, Patparganj, New Delhi-110092 Ph: +91-11-49344934; Fax: +91-11-49344935; Website: www.cbspd.com; www.eduport-global.com; E-mail: [email protected]; [email protected] Head Office: CBS PLAZA, 4819/XI Prahlad Street, 24 Ansari Road, Daryaganj, New Delhi-110002, India. Ph: +91-11-23289259, 23266861, 23266867; Fax: 011-23243014; Website: www.cbspd.com; E-mail: [email protected]; [email protected].

Branches Bengaluru: Seema House 2975, 17th Cross, K.R. Road, Banasankari 2nd Stage, Bengaluru - 560070, Kamataka Ph: +91-80-26771678/79; Fax: +91-80-26771680; E-mail: [email protected] Chennai: No.7, Subbaraya Street Shenoy Nagar Chennai - 600030, Tamil Nadu Ph: +91-44-26680620, 26681266; E-mail: [email protected] Kochi: 36/14 Kalluvilakam, Lissie Hospital Road, Kochi - 682018, Kerala Ph: +91-484-4059061-65; Fax: +91-484-4059065; E-mail: [email protected] Mumbai: 83-C, 1st floor, Dr. E. Moses Road, Worli, Mumbai - 400018, Maharashtra Ph: +91-22-24902340 - 41; Fax: +91-22-24902342; E-mail: [email protected] Kolkata: No. 6/B, Ground Floor, Rameswar Shaw Road, Kolkata - 700014 Ph: +91-33-22891126 - 28; E-mail: [email protected]

Representatives Hyderabad Pune Nagpur Manipal Vijayawada Patna

Preface E

very patient is unique and presents his/her distinct story. The same clinical problem may affect two different patients in different ways. So every patient create a puzzle in front of physician. This book in the field of otolaryngology and head and neck surgery is an unique creation for solving the jigsaw puzzle. This book presents numerous clinical cases and each case is followed by concise discussion regarding different issues related to clinical presentations and management in stepwise manner. While writing this book I took suggestions and constructive criticism from students, colleagues and friends. This book is a useful tool for undergraduates, postgraduates and general practitioners (GP) for dealing different clinical cases of ear, nose and throat and head–neck surgery. MS/MD/DNB aspirants and students preparing for USMLE and PLAB will also get benfit to get maximum score. I hope this book surely helps all levels of medical professionals such as undergraduate medical students, clinicians, residents and postgraduate aspirants with maximum benefit in the subject of ENT and head and neck surgery. Friends, I have tried my level best to provide useful clinical cases and stepwise discussion for helping them to get easy diagnosis. Teaching and learning go hand in hand. But ‘to err is human’ if there are questions where you disagree or have suggestions for the betterment of the book, kindly let me know.

Santosh Kumar Swain [email protected]

Acknowledgments I

am deeply indebted to my teacher and mentor Prof (Dr) Abhoy Kar, Prof (Dr) GC Sahoo, Prof (Dr) SN Panda, Prof (Dr) RN Samal, Prof (Dr) S Behera, Prof (Dr) RK Pattnaik, Prof RN Biswal, Dr KC Mallick, Dr Souvagini Acharya and Dr Satyajit Mishra, for their great moral support in writing this book. I sincerely thank to Prof (Dr) Manoj Ranjan Nayak, Founder President of Siksha ‘O’ Anusandhan University, Prof Gangadhar Sahoo, Dean of the IMS and SUM Hospital (Siksha ‘O’ Anusandhan University), for their great support behind this work. I thank whole team of CBS Publishers & Distributors Pvt Ltd, New Delhi, for their painstaking effort to make this book attractive and creating strong belief in this book for benefiting the medical fraternity. I can never forget people, who came in my professional training to teach, appreciate and encourage my hard work, Prof Ananad Job, Prof Rita Ruby Albert, Prof Achma Balraj, Prof Rupa Vedanta of Christian Medical College, Vellore. I am very much thankful my colleagues Prof SR Das, Dr SK Pani, Dr Alok Das, Dr Bulu Nahak, Dr Lokanth Sahoo, Dr Prasenjit Baliarsingh and Dr RV Natraj. I am also thankful to Dr Manoj Mishra and Mr Susanta Kumar Khuntia of Central Library for their encouragement to write such book for our students. I am grateful to my dear undergraduate (MBBS) and postgraduate (MS) students who always talked of and requested me to write a book on manual of clinical cases in ENT and head–neck surgery for making the exam easy and comfortable. I cannot forget motivation of my friends those motivate always are Dr Russel, Dr Pragnya, Dr Jasashree, Dr Nivedita, Dr Malvika, Dr Aswini and many more. My patients are another source of inspiration for attempting to write this book in simple and lucid manner. Last but not the least I am grateful to my parents, wife (Swagatika) and son (Ishan), without their encouragement and cooperation this book could never have been written.

Santosh Kumar Swain

Contents Preface

v

Section I: Otology 1. Clinical Anatomy of Ear

3

2. Diseases of External Ear

13

3. Otitis Media

27

4. Facial Nerve

47

5. Otosclerosis

50

6. Trauma to Middle Ear

55

7. Middle Ear Tumor

57

8. Meniere’s Disease

59

9. Acoustic Neuroma

63

10. The Deaf Child

66

11. Hearing Loss

70

12. Vertigo

79

Section II: Nose and Sinus 13. Diseases of the External Nose

87

14. Diseases of the Nasal Septum

94

15. Epistaxis

103

16. Allergic Rhinitis

108

17. Sinusitis

113

Manual of Clinical Cases in ENT and Head–Neck Surgery

x

18. Nasal Polypi

117

19. Granulomatous Diseases of the Nose

124

20. Neoplasms of the Nose and Sinus

133

21. Headache

138

22. CSF Rhinorrhea

143

23. Foreign Body in Nose

146

Section III: Head and Neck Diseases 24. Oral Cavity

153

25. Tonsils and Adenoids

161

26. Tumours of Oral Cavity and Oropharynx

180

27. Neck Mass

187

28. Vocal Cord Paralysis

202

29. Nasopharynx

208

30. Tumors of Hypopharynx

214

31. Tracheostomy

217

32. Congenital Lesions of Larynx

221

33. Infections of Larynx

224

34. Foreign Body in Airway

230

35. Hoarseness of Voice

232

36. Laryngeal Growth

241

37. Esophagus

246

38. Neck Abscess

249

Index

257

I

Otology 1. Clinical Anatomy of Ear 2. Diseases of External Ear 3. Otitis Media 4. Facial Nerve 5. Otosclerosis 6. Trauma to Middle Ear 7. Middle Ear Tumor 8. Meniere’s Disease 9. Acoustic Neuroma 10. The Deaf Child 11. Hearing Loss 12. Vertigo

1

Clinical Anatomy of Ear CASE 1: COUGH DUE TO EAR CANAL MANIPULATION

Presenting complaints: A 62-year-old male patient came to outpatient department of otorhinolaryngology for complaints of ear blockade. On examination there was wax in the left ear. With the help of a probe, wax removal was done. Patient developed coughing during wax removal, so doctor stopped the procedure for some time. Setting: Outpatient department. Examination of other ear: Normal. General examination: Blood pressure, pulse, respiratory rate are in normal range. Q.1: What is the cause for cough during wax removal? Ans: The posterior half of external auditory canal is get nerve supply from auricular branch of vagus nerve. Instrumentation in the posterior half of external auditory canal causes stimulation of auricular branch of vagus which is the most likely causes of cough in this patient. Q.2: What is the other name for auricular branch of vagus? Ans: Arnold’s nerve or Alderman’s nerve. Q.3: What is the nerve supply of external auditory canal? Ans: Anterior wall and roof by auriculotemporal nerve (V3), posterior wall and floor by auricular branch of vagus. Posterior wall of the external auditory canal also receives sensory fibres of 7th cranial nerve through auricular branch of vagus. Q.4: What is Hitzelberger’s sign? Ans: Hypoaesthesia of posterior wall of external auditory canal. It is seen in acoustic neuroma due to involvement of sensory fibres of 7th cranial nerve. Q.5: What is the anatomical profile of external auditory canal? Ans: The external auditory canal is S shaped and tortuous. The canal in adults measures about 2.4 cm from the opening in the concha to the tympanic membrane. The anterior and inferior walls of the canal are longer than the posterior and superior walls. The external auditory canal can be straightened by pulling the pinna upwards, backwards and laterally in adults and downwards and backwards in children, since the bony canal in children is not developed. 3

4

Manual of Clinical Cases in ENT and Head–Neck Surgery

Q.6: What is fissure of Santorini? Ans: Sometimes, there is dehiscence in the cartilaginous part of the external auditory canal in the floor and anterior wall, called fissure of Santorini. It provides potential paths for infections to spread between the external auditory canal, the parotid gland and superficial mastoid region and vice versa. Q.7: What is foramen of Huschke? Ans: In children up to the age of 4 years and sometimes in adults, anteroinferior part of the bony meatus may present as a dehiscence, called foramen of Huschke. It permits infections to and from the parotid. Q.8: What is nerve supply of the external auditory canal? Ans: The anterior wall and roof were supplied by auriculotemporal nerve (V3), posterior wall and floor are supplied by auricular branch of vagus. Posterior wall of the auditory canal also receives sensory fibers of CN VII through auricular branch of vagus. Q.9: What is Winkler’s nodule? Ans: Winkler’s nodule (chondrodermatitis nodularis chronic helices) occurs on rim of the helix of pinna. It is seen in old age. Q.10: Which is the narrowest part of the external auditory canal? Ans: About 6 mm lateral to tympanic membrane, the bony part of the external auditory canal presents a narrowing called isthmus. Foreign body often lodged medial to the isthmus get impacted and difficult to remove. Anteroinferior portion of the bony meatus beyond the isthmus presents a recess called anterior recess which acts as a cesspool for discharge and debris in case of external and middle ear infection. CASE 2: REFERRED OTALGIA

Presenting complaints: A 45-year-old male complains right ear pain since last 6 months. Setting: Outpatient department. On examination: Both ears are found to be normal. Examiantion of the oral cavity: Right side second molar shows carious tooth. General examination: Blood pressure, pulse and respiratory rate are in normal range. Q.1: What is this condition? Ans: Patient complaining of ear pain with normal examination of ear suggests referred otalgia. Q.2: What is the next step to find out the cause? Ans: Careful examination of oral cavity, oropharynx and temporomandibular joint is required in all cases with pain in the ear. Pain in ear may be presenting symptoms of carcinoma of oropharynx or carcinoma of larynx. This is because of carcinoma of the common neural innervations.

Clinical Anatomy of Ear

5

Q.3: What are the anatomical areas must be examined in referred otalgia? Ans: 5T: Tonsil, Tooth, Tongue, Temporomandibular joint and Throat (pharynx and larynx). Q.4: What are common nerve supply of ear and other organs? Ans: Ear

Other organs

External ear (auriculotemporal nerve)

TM joint (auriculotemporal nerve), oral cavity (lingual nerve), tooth (lingual nerve)

External ear (auricular branch of vagus)

Larynx (superior and recurrent laryngeal nerve)

Middle ear (glossopharyngeal nerve— tympanic plexus)

Oropharynx sensory innervation

Q.5: What are the causes of otalgia? Ans: Local causes of otalgia

Referred causes of otalgia

1. Otitis externa, furunculosis in external auditory canal, perichondritis, etc. 2. Impacted hard wax. 3. Impacted foreign body in ear canal. 4. Herpetic zoster oticus. 5. Myringitis and traumatic perforation of ear drum. 6. ASOM, acute mastoiditis. 7. CSOM with impending complications. 8. Malignancy of external and middle ear. 9. Barotraumatic otitis media.

1. Dental lesions: Impacted wisdom tooth, caries tooth, apical tooth abscess, gingivitis, etc. 2. Tongue lesions: Ulcer, glossitis, malignancy, etc. 3. Tonsillar lesions: Acute tonsillitis, quinsy, malignancy, etc. 4. Lesions in floor of mouth: Ulcer, malignancy. 5. Lesions of pharynx: Ulcers, malignancy, foreign body. 6. Laryngeal lesions 7. Temporomandibular arthralgia. 8. Sinonasal diseases. 9. Cervical spine lesions, neck lesions, etc.

Q.6: What are the differential diagnosis of referred otalgia? Ans: Via 5th Cranial Nerve • • • • • •

Sinusitis Nasopharyngitis Nasopharyngeal malignancy Carious tooth Costen syndrome Deviated nasal septum

Via 2nd and 3rd Cervical Nerves • Arthritis • Cervical disc lesions • Temporomandibular joint diseases

Manual of Clinical Cases in ENT and Head–Neck Surgery

6

• Fibrositis of sternomastoid muscle • Herpetic lesions Via 9th and 10th Cranial Nerves • • • • • •

Acute tonsillitis Retropharyngeal abscess Post-tonsillectomy Laryngopharyngeal lesions Glossopharyngeal neuralgia Styloid process neuralgia

Via 7th Nerve • Herpes zoster oticus CASE 3: ENDAURAL INCISION

Presenting features: An otolaryngologist planned for tympanoplasty with an endaural incision. At which part of the pinna, the surgeon will prefer to make an incision? Ans: Incisura terminalis. Q.1: Where the incisura terminalis present? Ans: The area between the tragus and crus of the helix called incisura terminalis. There is no cartilage in incisura terminalis. Incision at incisura terminalis will not cut through the cartilage and is used in endaural approach in surgery of the middle ear, mastoid and external auditory canal. Q.2: How cartilage framework of pinna is formed? Ans: The entire pinna except its lobule and incisura terminalis is made up of yellow elastic cartilage. Cartilage from the tragus or concha used for reconstructive surgery of middle ear. Sometimes conchal cartilage is used to correct the depressed nose. Q.3: What is the Rosen’s incision? Ans: It is the incision used to raise the tympanomeatal flap in order to expose the middle ear in case of endomeatal or transcanal approach. Rosen’s incision is the most commonly used for stapedotomy. It need the meatus and canal to be wide enough to work. It consists of two parts: A small vertical incision at 12 o’clock position near the annulus and a curvilinear incision starting at 6 o’clock position to meet the first incision in the posterosuperior region of the canal, 5–7 mm away from the annulus. This incision is also used for exploratory tympanotomy to find the cause for conductive hearing loss, inlay myringoplasty or ossicular reconstruction. Q.4: What is MacEwen’s triangle? Ans: It is bounded by temporal line, posterosuperior segment of bony external auditory canal and the line drawn as a tangent to the external auditory canal (Fig. 1.1). It is an important landmark to locate the mastoid antrum in mastoid surgery. The mastoid antrum is situated about 1.5 cm deeper to this triangle.

Clinical Anatomy of Ear

7

Fig. 1.1: MacEwen’s triangle

Q.5: What is postaural incision? Ans: It is also called Wilde’s incision. Postaural incision starts at the highest attachment of the pinna, follows the curve of postauricular groove, lying 1 cm behind it and ends at the mastoid tip. In case of infants or children up to 2 years of age , the mastoid process is not developed and the facial nerve lies exposed near its exit and the incision therefore is slanting posteriorly, avoiding lower part of the mastoid. Postauricular incision is use for cortical mastoidectomy, modified radical mastoidectomy, radical mastoidectomy, tympanoplasty, exposure of facial nerve in vertical segment and surgery of endolymphatic sac for treatment of Meniere’s disease. CASE 4: PREAURICULAR SINUS

Presenting complaint: The parents brought their 5-year-old child to ENT OPD and complained a small dimple or punctum anterior to the right tragus. Setting: Outpatient department History of presenting illness: A 5-year-old child apparently well but presenting a pit infront of right tragus since birth. There were two three episodes of abscess formation in front of right pinna. History of past illness: There were history of three times incision and drainage of the abscess infront right tragus. General physical examination: The patient is well oriented in time, place and person. Pulse—78/min, blood pressure—122/80 mmHg. Local examination: A small pit infront of right tragus (Fig. 1.2). Left pinna and right pinna are normal anatomical configuration.

Fig. 1.2: A pit infront of tragus

8

Manual of Clinical Cases in ENT and Head–Neck Surgery

Q.1: What is the diagnosis? Ans: Right side preauricular sinus. Q.2: What is preauricular sinus? Ans: It is a congenital sinus infront of tragus of the pinna since birth and is due to incomplete fusion of the hillocks of His between first and second branchial arches. It is usually located between the tragus and the crus of helix. Q.3: What is the embryological importance of preauricular sinus? Ans: The pinna develops from the fusion of the six tubercles. These six tubercles fuse together to form the pinna (six hillocks of His). Tragus develops from the tubercle of first arch while rest of the pinna develops from the remaining five tubercles of the second arch. Faulty fusion between the first and the second arch tubercles causes preauricular sinus. Q.4: What is the treatment of preauricular sinus? Ans: Complete excision of the entire sinus and its ramification. Sinus can be delineated by injecting methylene blue. In case patient present with abscess, incision and drainage by vertical incision was made first. After healing, complete excision of the preauricular sinus is made. Q.5: What is bat ear? Ans: It is an abnormally protruding ear where the concha is large with poorly developed antihelix and scapha. CASE 5: TYMPANIC MEMBRANE

Presenting complaints: An adult patient came to outpatent department of ENT for ear examination before medical fitness check up in the defence recruitment board. Examination: Examination of ear, nose and throat are within normal limit. Both sides tympanic membranes are normal. Q.1: During otoscopy, the most reliable sign over tympanic membrane is? Ans: Lateral process of malleus. The short/lateral process of the malleus is least obliterated by diseases. Q.2: For viewing the tympanic membrane properly, the pinna should be pulled in which direction? Ans: Pinna is pulled backwards and upwards in adult. The pinna is pulled downwards and outwards in infants. Q.3: What is the normal color of the tympanic membrane? Ans: The tympanic membrane appears as a greyish white, translucent membrane set obliquely inside the external auditory canal. Q.4: What is the clinical importance of anterior and posterior malleolar folds? Ans: Anterior and posterior malleolar folds separate the pars tensa from pars flaccida.

Clinical Anatomy of Ear

9

Q.5: What is cone of light? Ans: Cone of light is seen in anteroinferior quadrant of the tympanic membrane and it is actually the reflection of the light projected into the ear canal to examine it. The anteroinferior quadrant of the tympanic membrane is the only part of the tympanic membrane which is right angle to the meatus, so it reflects the exposed light. The handle of malleus causes tenting and because of tenting the anteroinferior quadrant is at right angles to the meatus and thus reflects the light which results in cone of light. Q.6: What is the surface area of the tympanic membrane? Ans: Area of tympanic membrane is 90 mm2. The effective vibrating area of the tympanic membrane is 55 mm2. Thickness of the tympanic membrane is 0.1 mm. Tympanic membrane is 9–10 mm tall and 8–9 mm wide. CASE 6: MACEWEN’S TRIANGLE

Presenting complaints: A person was presenting with right ear discharge since 3 years and decreased hearing since 6 months. The discharge was non-foul smelling, nonblood stained and profuse. Examination: There is large cental perforation in the right tympanic membrane. No active discharge is found during otoscopic examination of the ear. Q.1: If patient is planned for right cortical mastoidectomy and type-1 tympanoplasty, what is the surgical landmark for cortical mastoidectomy. Ans: MacEwen’s triangle or suprameatal triangle. Q.2: What is the boundary of the MacEwen’s triangle? Ans: It is bounded above by suprameatal line, anteroinferiorly by posterosuperior margin of external auditory canal and posteriorly by a tangent drawn from zygomatic arch. Q.3: What is Korner’s septum? Ans: Mastoid develops from squamous and petrous bone. Korner’s septum is persistence of petrosquamous suture in the form of a bony plate. Korner’s septum is surgically important as it may cause difficulty in locating the antrum and the d e e p e r cells and thus results in incomplete removal of disease during mastoidectomy. Mastoid antrum cannot be reached unless the Korner’s septum has been removed. Q.4: What are the types of mastoid bone? Ans: There are three types of the mastoid bone such as: Pneumatised/cellular (80%), sclerotic (20%) and diploic. Pneumatisation begins in first year and is complete by 4 to 6 years of age. Q.5: What is Trautmann’s triangle? Ans:It is bounded by sigmoid sinus posteriorly, bony labyrinth anteriorly and superior petrosal sinus superiorly.

10

Manual of Clinical Cases in ENT and Head–Neck Surgery

CASE.7: STAPEDIAL REFLEX

Presenting complaints: A 22-year-old boy attended the ENT OPD with presentation of sudden right side facial paralysis with intolerance to loud noise since 2 days. He had history exposure to severe cold. He had no hearing loss, tinnitus and vertigo. Examination: Right side lower motor neuron facial nerve paralysis (Bell’s pasly). Bilateral pinna, external auditory canal and tympanic membrane were within normal limits. Q.1: What is the cause of intolerance to loud noise in this patient with right side Bell’s palsy? Ans: Lesion of the facial nerve above the nerve of stapedius cause loss of stapedial reflex which result in hyperacusis or phonophobia, i.e. intolerance to loud sound. Q.2: What is the role of stapedius muscle for preventing hyperacusis? Ans: Stapedius muscle attaches to the neck of stapes and helps in dampen very loud sounds thus preventing noise trauma to the inner ear. Stapedius is a second arch muscle and is supplied by a branch of CN VII. Paralysis of the stapedius muscle results in intolerance to loud noise or hyperacusis. Q.3: What is stapedial reflex Ans: When loud sound 70–100 decibel above the threshold of hearing is exposed to a particular ear, cause bilateral contraction of the stapedial muscles. Ipsilateral: CN VIII—ventral cochlear nucleus—CN VII nucleus—ipsilateral stapedius muscle. Contralateral: VIII—ventral cochlear nucleus—contralateral medial superior olivary nucleus—contralateral CN VII nucleus—contralateral stapedius muscle. CASE 8: FACIAL RECESS

Presenting complaints: A 3-year-old boy underwent cochlear implant for bilateral profound hearing loss. The electrodes of the cochlear implant is inserted into the round window via posterior tympanotomy approach. Q.1: Through which recess, the electrode is inserted by posterior tympanotomy approach? Ans: Facial recess. Q.2: What is facial recess? Ans: Facial recess is also called suprapyramidal recess where collection of air cells lying lateral to the facial nerve. It is bounded medially by facial nerve, laterally by chorda tympani nerve and superiorly by fossa incudes. Q.3: What is the surgical importance of the facial recess? Ans: Importance of this recess is that one can approach the middle ear without disturbing posterior meatal wall. This approach is used in case of cochlear implant and facial nerve decompression. Q.4: What is fossa incudes? Ans: Fossa incudes lies in epitympanic recess and contains short process of incus.

2

Diseases of External Ear CASE 1: OTITIS EXTERNA

Presenting complaints: A 25-year-old boy presents with a painful left ear since 3 days. Setting: Out patient department. Examination of pinna and external auditory canal: On examination, reveals a swollen lesion in the external auditory canal (Fig. 2.1). The pain is severe and worsened by traction of tragus and jaw movement.

Fig. 2.1: Otitis externa

Past history: History of injury to external auditory canal one week back. Facial nerve examination: Normal. Q.1: Most probable diagnosis? Ans: Otitis externa. Otitis externa is inflammation of the external auditory canal. It may be localized (furuncle) or diffuse. Q.2: What are the common organisms responsible for otitis externa? Ans: Staphylococcus aureus, Pseudomonas pyocyaneus, Bacillus proteus and Escherichia coli. Often the infection is mixed. 13

12

Manual of Clinical Cases in ENT and Head–Neck Surgery

Q.3: What is the commonest cause of facial palsy at birth? Ans: Facial palsy at birth is more often traumatic rather than developmental. Birth trauma like forceps delivery or prolonged and difficult labor can result in injury to facial nerve. Q.4: What is the commonest congenital malformation of temporal bone? Ans: Dehiscent fallopian canal (facial nerve canal) where facial nerve has no bony covering. Q.5: What is the investigation of choice for congenital malformation of the ear? Ans: Investigation of choice for congenital malformations of the ear is HRCT. CASE 11: PATULOUS EUSTACHIAN TUBE

Presenting complaints: During third trimester of pregagncy, a 32-year-old lady complained with aware about her own sounds. Otoscopy examination: Movement of tympanic membrane which synchronous with respiration. The movement of the tympanic membrane is exaggerated when patient breathes only through the ipsilateral side of nose. Q.1. What is the possible diagnosis? Ans: Patulous eustachian tube. Q.2: What is eustachian tube? Ans: Eustachian tube provides communication between middle ear and nasopharynx and is 36 mm in length. Its lateral one-third is bony and medial two-thirds are cartilaginous. It remains closed at rest. The opening of the tube, which is an active process, occurs due to contraction of tensor veli palatini muscle while closure occurs due to recoil of the cartilaginous part. Q.3: What is Gerlach tonsil? Ans: This is lymphoid tissue of the eustachian tube. This is also called tubal tonsil. Q.4: What is Frenzel maneuver? Ans: It opens the eustachian tube and ventilates the middle ear. The muscles of the floor of mouth and pharynx are contracted while nose, mouth and glottis are closed. In comparision to Valsalva maneuver, it is difficult to learn. Q.5: What is Ostmann’s pad of fat? Ans: It keeps the eustachian tube closed and is related laterally to the membranous part of the cartilaginous tube. The closed tube protects itself and middle ear from the reflux of nasopharyngeal secretions.

2

Diseases of External Ear CASE 1: OTITIS EXTERNA

Presenting complaints: A 25-year-old boy presents with a painful left ear since 3 days. Setting: Out patient department. Examination of pinna and external auditory canal: On examination, reveals a swollen lesion in the external auditory canal (Fig. 2.1). The pain is severe and worsened by traction of tragus and jaw movement.

Fig. 2.1: Otitis externa

Past history: History of injury to external auditory canal one week back. Facial nerve examination: Normal. Q.1: Most probable diagnosis? Ans: Otitis externa. Otitis externa is inflammation of the external auditory canal. It may be localized (furuncle) or diffuse. Q.2: What are the common organisms responsible for otitis externa? Ans: Staphylococcus aureus, Pseudomonas pyocyaneus, Baciilus proteus and Escherichia coli. Often the infection is mixed. 13

14

Manual of Clinical Cases in ENT and Head–Neck Surgery

Q.3: What are the predisposing factors for otitis externa? Ans: Trauma (self cleaning habit with ear buds and match sticks), narrow external auditory canal, diabetic, swimming, keratosis obturans, environmental (Singapore ear, Hongkong ear). Q.4: What is the treatment of otitis externa? Ans: Ear toileting or mopping or suction of ear canal will clean purulent discharge. Packing ear canal with 10% ichthammol in glycerine on a ribbon gauze relieves edema, tension and pain. Broad spectrum antibiotics. Analgesics and antipyretics for relief of pain and fever. Q.5: How furuncle of the external auditory canal differs from mastoiditis? Ans: Furuncle

Mastoiditis

1. There is no history of otorrhea

History of otorrhea

2. Not associated with hearing loss

Associated with hearing loss

3. Otalgia—more acute

Less

4. Purulent ear discharge

Discharge is mucoid or mucopurulent

5. Tragal sign positive-tragal tenderness on pressure.

Negative

6. Canal stenosis (cartilaginous part only).

Posterior bony meatal wall may be bulged.

7. Normal and intact tympanic membrane.

Perforation of tympanic membrane.

8. No tenderness over mastoid.

Tenderness over mastoid present.

9. Postauricular groove-obliterated.

Deepened postauricular groove.

10. Normal hearing.

Usually associated with conductive hearing loss.

11. No radiological changes in mastoid.

Imaging shows bone erosion in mastoid.

Q.6: What is mastoidism? Ans: Presence of mastoid tenderness without mastoiditis is called mastoidism. It occurs in acute suppurative otitis media and is due to the fact that mucus lining of middle ear and mastoid are in continuity through aditus. Q.7. What is the position of pinna in furunculosis of external auditory canal? Ans: Pinna is forward and outward in furunculosis while forward and downward in mastoid abscess (erection of pinna). Q.8: What are palpatory findings of pinna in different ear infections? Ans: • Tenderness at the mastoid process and the cymba conchae signifies mastoiditis. Suprameatal triangle lies under the cymba cocnhae and can be felt through it. The mastoid antrum lies deep to the suprameatal/MacEwen’s triangle. • Tenderness of tragus indicates furunculosis at the anterior wall of external auditory canal. • Tenderness of pinna indicates perichondritis.

Diseases of External Ear

15

CASE 2: OTOMYCOSIS

Presenting complaints: A 60-year-old man presents with right side ear pain, itching, block sensation in the same ear and discharge since 15 days. Setting: OPD. Examination of right side external ear: The external auditory meatus is oedematous and the canal is stenosed (Fig. 2.2). The discharge is white and creamy in nature. The discharge appears as wet newspaper.

Fig. 2.2: Otomycosis

Examination of the other ear: Normal. Examination of nose and paranasal sinuses: Normal. Q.1: Most probable diagnosis? Ans: Right side otomycosis. Q.2: What is otomycosis? Ans: It is a fungal infection of the external auditory canal, caused by Aspergillus niger, Aspergillus fumigates or Candida albicans. Q.3: What are the clinical features in otomycosis? Ans: It is commonly seen during the hot and humid months of the ear. Common symptoms are irritation, itching and dull pain in the ear. In some patients there may be discharge, sense of blockage or having heaviness in the ear. Hearing loss is uncommon unless blocked by mycotic plug. The external auditory canal is filled up by wet debris or flakes resembling wet newspaper or blotting paper with blackish areas. In some cases, grayish-white or grayish-black cotton wool like mass lies in the deeper part of the canal. Q.4: What is the treatment of otomycosis? Ans: Thorough aural toileting for removal of ear discharge and fungal debris. It is ideally done by suction cleaning and in some cases by syringing. Topical application of antifungal ear drops are effective against the fungus.

16

Manual of Clinical Cases in ENT and Head–Neck Surgery

CASE 3: RAMSAY HUNT SYNDROME

Presenting complaints: A 50-year-old lady presented with left sided hearing loss and a painful ear. She has feeling of giddiness. Setting: Outpatient department. Examination of ear: Vesicular eruptions over skin in front of the right ear (Fig. 2.3). Normal tympanic membrane.

Fig. 2.3: Vesicular eruptions in Ramsay Hunt syndrome

Facial nerve examination: Left side lower motor nerve palsy. Examination of nose and sinus: Normal. General examination: Normal. Q.1: What is the most possible diagnosis? Ans: Ramsay Hunt syndrome. It is also called herpes zoster oticus. It was first described by Hunt James in 1907. Q.2: What are the clinical presentations of Ramsay Hunt syndrome? Ans: It is characterized by formation of vesicles on the tympanic membrane, meatal skin, concha and postauricular groove. The VIIth and VIIIth cranial nerves may be involved. In its severe form, there may be sensorineural hearing loss and disturbed vestibular function and even signs and symptoms of viral encephalitis. Q.3: What are the cranial nerves involved in Ramsay Hunt syndrome? Ans: Cranial nerves V, IX, and X in addition to VIIth and VIIIth cranial nerves may also be involved. It is caused by chickenpox causing virus varicella, which affects geniculate ganglion.

Diseases of External Ear

17

Q.4: What are the treatments of Ramsay Hunt syndrome? Ans: Antiviral drugs such as acyclovir in the form of tablets 800 mg 4–5 times a day and cream. Corticosteroids in the form of tablet and cream. Antibiotics are given to prevent secondary infection. Anti-inflammatory drugs can be added. Facial nerve palsy is also managed by facial exercises. CASE 4: OSTEOMA

Presenting complaints: A 21-year-old boy complaining decreasing hearing in right ear and block sensation in the same ear since one year. Setting: Out patient department. On examination: A localized smooth, slow growing bony swelling at the external auditory canal (Fig. 2.4). General examinations: Blood pressure, pulse and other vitals of the patient were within normal limits.

Fig. 2.4: Osteoma

Q.1: What is the most possible diagnosis? Ans: Osteoma. It is a benign tumor of the cortical bone. Q.2: How osteoma presents? Ans: Osteoma arises from the cancellous bone and presents as a single, smooth, bony, hard, pedunculated tumor, often arising from the posterior wall of bony canal, near its outer end. Q.3: What is the treatment of osteoma? Ans: Surgical removal by making fracture through its pedicle or removal with a drill. Q.4: How exostoses differ from the osteoma? Ans: Exostosis are multiple and bilateral, often seen as smooth, sessile, bony swelling in the deeper part of the external auditory canal near the tympanic membrane. Exostosis is commonly seen among cold water swimmers and divers. In exostosis, males are affected three times more than females.

18

Manual of Clinical Cases in ENT and Head–Neck Surgery

CASE 5: TRAUMATIC PERFORATION OF THE TYMPANIC MEMBRANE

Presenting complaints: A 13-year-old boy presents with hearing loss and pain in the right ear after severe slap to the right side of the face. Otoscopic examination: A moderate perforation in the lower part of the tympanic membrane (Fig. 2.5). The margin of the perforation is irregular and blood tinged.

Fig. 2.5: Traumatic perforation in TM

Q.1: What is the most possible diagnosis? Ans: Traumatic perforation of the right tympanic membrane. Q.2: What are the causes of traumatic perforation of the tympanic membrane? Ans: Direct violence (attempt to clean ear by matchstick, hair pins, etc. forceful syringing, sneezing or forceful Valsalva procedure). Indirect violence (strong slap, blast injury, skull base fracture, trauma to mandible and barotraumas). Q.3: What are the common symptoms in traumatic perforation? Ans: Pain in the ear, hearing loss, tinnitus, giddiness and bleeding from the ear. Q.4: How the perforated tympanic membrane heals? Ans: The outer epithelial and inner endothelial (mucosal) layers regenerate to seal the perforation but the middle fibrous layer does not regenerate. Newly formed two layered membrane is called monometric membrane and it is usually transparent. Q.5: What is the treatment of perforated tympanic membrane? Ans: Nothing applying in the ear. Avoid water entery into the ear canal. Keep sterilised cotton in meatus. Prophylactic antibiotics. Healing occurs by 6–8 weeks. Unhealed perforation need myringoplasty. CASE 6: FOREIGN BODY IN THE EAR CANAL

Presenting complaints: An 18-month-old male child referred to ENT department from pediatric surgery department with complaint of an asymptomatic pin like metallic

Diseases of External Ear

19

foreign body in the left external auditory canal. There was no swelling, bleeding or dischanrge in the ear canal. Examination: There was no scar mark, skin discoloration, swelling or tenderness in the preauricular and postauricular region. The foreign body was embedded between the walls and did not move when attempts was made to remove. Q.1: What is the plan in the case? Ans: Foreign body removal is planned under general anesthesia. The foreign body was removed and external auditory canal and tympanic membrane were within normal limits. Q.2: What are causes for foreign body insertion into the eaternal auditory canal? Ans: There are several reasons lead to foreign body insertion into the external auditory canal such as accidental entry of objects, curiosity of children, games, attempts to clean the ear canal or itching. Q.3: What are the prerequisites for removal of the foreign body from external auditory canal in children? Ans: Removal of the foreign bodies from the external auditory canal is an essential skill and casual removal may result in impaction of foreign body and complications. Foreign body removal from external auditory canal especially in children should be preferably removed with proper instruments under general anesthesia, good illumination and magnification if needed. As retained foreign body may go unnoticed in children for a long time, the ears and nose should be carefully examined in every child attending the ENT out patient department. CASE 7: MICROTIA

Presenting complaints: A newborn baby presents with bilateral microtia and external auditory canal atresia. Q.1: What is the ideal age for doing corrective surgery for this case? Ans: 5 to 7 years. Q.2: What is the treatment for microtia? Ans: Microtia typically presents at birth with obvious auricular malformations. Classical treatment involves auricular reconstruction in multiple stages. Patients undergo observation until the age of 5 years to allow for grown of rib cartilage which is harvested for reconstruction. This approach provides the benefit of reconstruction with autogenous material which ultimately requires little or no maintainance. Q.4: What are congenital anomalies of the pinna? Ans: • Anotia: Absence of pinna • Macrotia: Large pinna • Microtia: Small pinna • Mozart’s ear: Fusion of crura of antihelix

20

Manual of Clinical Cases in ENT and Head–Neck Surgery

• Bat ear: Abnromal protrusion of pinna characterized by absence of antihelix • Lop ear: Variant of bat ear • Down’s syndrome: Small pinna and poorly developed lobule • Wildermuth’s ear: Prominent antihelix • Malotia: Pinna placed downwards and forwards • Synotia: Pinna is below the level of maxilla • Potter’s syndrome: Pinna is low set CASE 8: MALIGNANT OTITIS EXTERNA

Presenting complaints: A 72-year-old diabetic male presented with severe right side ear pain and discharge since one month. Examiantion: There is rapidly spreading infection of the external auditory canal with involvement of the bone and presence of the granulation tissue. Q.1: What is the diagnosis of this case? Ans: Rapidly spreading infection of external auditory canal, seen in diabetic patient with involvement of bone and presence of granulation tissue point towards malignant otitis externa. Q.2: What is the treatment policy for malignant otitis externa? Ans: The treatment includes correction of immunosuppression (when possible), local treatment of external auditory canal, long term systemic antibiotic therapy and in selected patient surgery. Q.3: What is the etiology for malignant otitis externa? Ans: Malignant otitis externa is a misnomer where the term malignant does not indicate malignant pathology. It is an inflammatory condition caused by Pseudomonas infection. It is seen in elderly diabetic/immunocompromised patients/patients on immunosuppressive drugs. Q.4: Which cranial nerve most commonly affected in malignant otitis externa? Ans: As this infection spreads to temporal bone and base of skull, it may involve cranial nerves. Most commonly facial nerve is affected. Q.5: What are the diagnostic criteria for malignant otitis externa? Ans: • Refractory external otitis • Severe nocturnal otalgia • Purulent ear discharge • Granulations in deep external ear canal • Pseudomonas aeruginosa in culture • Diabetes or any other immunocompromised state

Diseases of External Ear

21

Q.6: Which is the gold standard for diagnosis of malignant otitis externa? Ans: Positive technetium 99 bone scan. Gallium scan (for prognosis) Other investigations are culture and sensitivity, CT/MRI to assess the extent and biopsy to rule out malignancy. Q.7: Which drugs are considered as drug of choice in malignant otitis externa? Ans: • Cefepime/ceftazidime (3rd generation cephalosporin) • Antipseudomonal penicillin with an aminoglycoside/fluoroquinolone CASE 9: PERICHONDRITIS OF PINNA

Presenting complaints: A 28-year-old boxer in profession suffered injury to the right pinna during the fight with another boxer since 15 days back. On examination: The right pinna deformed and appears as cauliflower look. Q.1: What is the cause of the deformed pinna which appears as cauliflower? Ans: Perichondritis of the pinna due to blunt trauma. Q.2: What is pathophysiology behind the perichondritis of the pinna of above patient (boxer)? Ans: Blunt trauma to the pinna in boxer causes hematome of pinna (collection of blood between auricular cartilage and perichondrium) which results in deformity, called cauliflower ear.If infection occurs in hematoma, it leads to severe perichondritis of the pinna. Q.3: Which bacteria is associated with perichondritis of the pinna? Ans: Perichondritis of the pinna is most commonly caused by Pseudomonas aeuroginosa. Q.4: What are clinical symptoms of the perichondritis of the pinna? Ans: Initial symptoms are red, hot and painful pinna which feels stiff. Later abscess may form between the cartilage and perichondrium with necrosis of the cartilage as the cartilage survives only on blood supply from its perichondrium. CASE 10: CERUMEN

Presenting complaints: A 10-year-old boy presenting with left ear pain and decrease hearing in the same ear since 20 days. Otoscopic examination: Dark to brownish and soft to hard material filling the right external auditory canal. Q.1: What is the diagnosis? Ans: Cerumen Q.2: How cerumen/wax is formed? Ans: Wax is composed of secretion of sebaceous glands, ceruminous glands, hair, desquamated epithelia debris, keratin and dirt.

22

Manual of Clinical Cases in ENT and Head–Neck Surgery

Q.3: What are the functions of the wax in external auditory canal? Ans: Wax has a protective function as it lubricates the ear canal and entraps the foreign body that happens to enter the canal. Its acidic pH and is bacteriostatic and fungistatic. Q.4: What are wax softners? Ans: • 5% sodium bicarbonate • Diluted hydrogen peroxide • Liquid paraffin • Olive oil • 2% paradichlorobenzene Q.5: What are the etiology factors for excess wax formation? Ans: • Obliquity of the external auditory canal/narrow ear canal • Excessive secretions of the wax • Faulty method for cleaning of the ear • Dusty environment • Dry hot climate • Exostosis • Stiff hairs in the ear canal Q.6: What are the contents of the wax? Ans: • High concentration of lipid • Lysozymes • Immunoglobulins CASE 11: INSECT IN EXTERNAL AUDITORY CANAL

Presenting complaints: A 3-year-old boy presented with severe pain in the right ear after sudden entry of an insect in ear canal. On examination: A living insect inside the right side external auditory canal with visibility of its crawling movement. Q.1: What should be done first in this case ? Ans: Live foreign body (insect) in the ear canal is a dire emergency. Before removal, it should be killed by instillation of oil/spirit, since a struggling insect can produce grievous consequences like tympanic membrane rupture, ossicular disruption and even inner ear damage. After killing of the live foreign body, it can be removed by forcep under general anesthesia in case of children or by syringing without under general ansthesia.

Diseases of External Ear

23

Q.2: What are the different types of foreign bodies in ear? Ans: • Living: Insect, maggots • Non-living: Hygroscopic and non-hygroscopic • Hygroscopic non-living foreign body: Vegetables such as peas, beans, seeds, etc. • Non-hygroscopic non-living foreign bodies: Animate—insects, flies, maggots; Inanimate—stones, stitch, rubber, pencil, plastic materials. Q.3: What are the clinical presentations of the foreign body in the ear canal? Ans: Pain, apprehension of parents, deafness (partial) and irritation in the ear canal. Q.4: How the foreign bodies from the external auditory canal are removed? Ans: • Living foreign body: Instill oil or water to the ear to suffocate to death of living foreign body, then removed by forceps or syringing. • Hygroscopic foreign body: Shrinkage of the swollen foreign body by instillation of spirits, glycerine, then remove by ring curette or syringing. • Non-hygroscopic foreign body can be removed by ring part of the Jobson Horne’s probe or by syringing. • Sometimes under general anesthesia, foreign body is removed with a view not to damage tympanic membrane/ossicles. • If the foreign body entered into the middle ear by perforating into the tympanic membrane, then should be removed on postaural approach under general anesthesia or by transcanal endoscopic method under genral anesthesia (in children)/local anaesthesia (in adult). Q.5: What are the complications of the foreign body in the ear canal? Ans: • Vasovagal attack • Perforation of tympanic membrane • Entry of foreign body into middle ear • Otitis externa • Otitis media • Ossicular injury CASE 12: EXOSTOSES (DIAPHYSEAL ACLASIS)

Presenting complaints: A 30-year-old man presenting with decreased hearing since 3 months. He had no history of ear discharge. The patient is swimmer in profession. On examination: Multiple painless nodular swelling in the lumen of external auditory canal in both sides. Q.1: What is the possible diagnosis? Ans: Exostoses (surfer’s ear)

24

Manual of Clinical Cases in ENT and Head–Neck Surgery

Q.2: What are exostoses? Ans: These are periosteal outgrowths, which grow away from the ivory bone so called exostoses. These are often occur at epiphysis of growing end of bone. Q.3: Is it a bony tumor? Ans: It is considered as cancellous osteoma but actually it is a disorder of bone growth and is not a tumor. It is cancellous bone covered with cortical bone. Bas is broad and other end is covered with cartilage. Q.4: What are salient features of the exostoses? Ans: • It is common in persons with multiple exposure to swimming in cold water. • It is painless and slow growing bony swelling. • It is mainly found in cartilaginous bone. • Swelling is adherent to underlying bone with broad base. Q.5: What are the complications of exostoses? Ans: • Infections • Deafness • Impacted wax and desquamated epithelial debris due to defective cleaning mechanism. Q.6: What is the treatment of exostoses? Ans: Excision of exostoses with micro-drill. CASE 13: MICROTIA

Presenting complaints: A 5-year-old girl presenting with small size pinna in left side (Fig. 2.6).

Fig. 2.6: Microtia

Q.1: What is provisional diagnosis? Ans: Microtia

Diseases of External Ear

25

Q.2: How does a malformed pinna develop? Ans: The pinna develops from the six hillock of His. Maldevelopment or failure of fusion can result in gross abnormalities of the ear. Microtia is a deformed small sized pinna. It may or may not be associated with middl or inner ear abnormalities. Q.3: What is the classification of congenital malformed pinna? Ans: • Grade I: Normal ear • Grade II: All pinna elements present but malformed • Grade III: Rudimentary bar only • Grade IV: Absent pinna (anotia) CASE 14: EUSTACHIAN TUBE DYSFUNCTION

Presenting complaints: A 25-year-old man presented with fullness in the right ear since 5 days. He had common cold 5 days back. Otoscopic examination: Right side: External auditory canal: Normal; tympanic membrane appears dull and lustreless. Cone of light was absent. Left ear: Normal. Valsalva maneuver: No movement of the tympanic membrane during maneuver. Q.1: What is the possible diagnosis? Ans: Right side eustachian tube dysfunction. Q.2: What are the changes in the tympanic membrane in case of eustachian tube dysfunction? Ans: • Lustreless tympanic membrane • Cone of light absent or distorted • Sickle-shaped anterior and posterior malleolar folds • Foreshortening of the handle of malleus • Tympanic membrane retracted medially • Lateral process of malleus appears prominent Q.3: What are the functions of the eustachian tube? Ans: Physiologically, eustachian tube performs three main functions such as: • Ventilation and thus regulation of the middle ear pressure • Protection against nasopharyngeal sound pressure and reflux of nasopharyngeal secretions • Clearance of middle ear secretions Q.4: What are the eustachian tube function tests? Ans: Different eustachian tube function tests are: Valsalva test, politzer test, catheterization test, Toynbee’s test, tympanometry, radiological test, saccharine or methylene blue test and sonotubometry.

26

Manual of Clinical Cases in ENT and Head–Neck Surgery

Q.5: What are the causes of eustachian tube obstruction? Ans: • Upper respiratory infection (viral or bacterial) • Sinusitis • Allergy • Nasal polyps • Deviated nasal septum • Nasopharyngeal tumors/mass • Cleft palate • Submucous cleft palate • Adenoid hypertrophy • Down syndrome • Functional

3

Otitis Media CASE 1: ASOM (ACUTE SUPPURATIVE OTITIS MEDIA)

Presenting complaints: Doctor, my son has severe right ear pain since last night. Setting: Outpatient department (OPD) History of presenting illness: A 4-year-old boy complaining severe pain in the right ear since last night. He has history of common cold since last 5 days. General physical examination: The child is well oriented with time, place and person. Pulse rate: 92 BPM. Blood pressure: 108/70 mmHg. Examinations of pinna and surrounding area: Normal Otoscopic examination: Congested tympanic membrane (Fig. 3.1). Leash of blood vessels seen along the handle of malleus and at the periphery of tympanic membrane showing a cart-wheel appearance. Facial nerve examination: Bilateral facial nerves are normal.

Fig. 3.1: Congested tympanic membrane

Q.1: What is the most possible diagnosis? Ans: Right side ASOM. Q.2: Why ASOM is more common in infant and children? Ans: Eustachian tube in infants and young children is shorter, wider and more horizontal and thus transmit more infection from nasopharynx to middle ear. Breast 27

28

Manual of Clinical Cases in ENT and Head–Neck Surgery

or bottle feeding in infant in a horizontal position may force fluids through the tube into the middle ear. The immunity of infant and young children are low which promote more infections in middle ear. Q.3: What are the common bacteria causing ASOM? Ans: Commonest bacterial agents are Streptococcus pneumonia, H. influenzae and Moraxella catarrhalis. Q.4: What is the medical treatment for ASOM? Ans: Drug of choice is amoxicillin (high dose). In resistant cases, amoxicillin plus clavulanic acid or cefuroxime is used. Q.5: What are the indications for myringotomy in ASOM? Ans: ASOM with complications, ASOM with severe pain, unresolved ASOM, ASOM not responding to medical treatment. Q.6: What is lighthouse sign? Ans: In suppurative stage of ASOM, otoscopy shows pulsatile discharge may reflect the light intermittently. This is called lighthouse sign. CASE 2: SECRETORY OTITIS MEDIA

Presenting complaints: A 5-year-old boy with ASOM undergoing treatment with pencillin therapy for 7 days now presents with subsidence of pain but persistence of deafness. Setting: Outpatient department. Otoscopic examination: Bilateral tympanic membranes are dull, bulged and air bubbles with fluid level are present behind it (Fig. 3.2).

Fig.3.2: Dull TM with air and fluid levels behind it

Examination of pinna and external auditory canal: Normal. Examination of nose, sinus, oral cavity and oropharynx: Normal. Pure tone audiometry: Bilateral moderate conductive hearing loss.

Otitis Media

29

Q.1: What is the most possible diagnosis? Ans: This is a case of secretory otitis media (SOM) developing over ASOM probably due to biofilms formation. So some symptoms of ASOM, i.e. pain and fever has subsided but due to persistence of fluid deafness is persisting. Q.2: What is the significance of otitis media in children? Ans: It is the most common cause of conductive deafness in children. Usually bilateral secretory otitis media in a child is due to adenoid hypertrophy. Q.3: What is the type of tympanogram in secretory otitis media? Ans: Type-B curve. Q.4: What is the significance of unilateral secretory otitis media in adult? Ans: An adult presenting with unilateral secretory otitis media, always rule out nasopharyngeal carcinoma. Q.5: What is the treatment of secretory otitis media? Ans: Medical treatment includes anti-allergics, steroids, decongestants, mucolytics and antibiotics. Mainstay of treatment is surgical which is myringotomy with grommet insertion. This is indicated in hearing loss of more than 25 decibel persisting for more than 3 months. CASE 3: CSOM-TTD (CHRONIC SUPPURATIVE OTITIS MEDIA-TUBOTYMPANIC DISEASE)

Presenting complaints: A 35-year-old female presented at the otolaryngology outpatient department with complaints of right ear discharge since childhood, decreased hearing in the right ear since 3 years and headache since 6 months. History of present illness: Right ear discharge was started since childhood and the discharge was insidious in onset, intermittent, profuse, mucoid to mucopurulent, nonfoulsmelling, nonblood stained discharge, more during upper respiratory tract infections, relieved on application of topical ear drops. He had last ear discharge 3 months back. Decreased hearing in the right side started 3 years back, insidious in onset and progressive in nature. There was no history of giddiness or tinnitus. He had history of headache since 6 months, insidious in onset, gradually progressive, intermittent, bifrontal, more on bending forward, more in the morning and more during upper respiratory tract infection. No history of visual aura, vomiting, decreased vision, recurrent nasal obstruction or mouth breathing, sneezing or blood stained nasal discharge. Past history: No history of diabetes mellitus, hypertension, bronchial asthma, ischemic heart disease, pulmonary tuberculosis or drug allergy and no past history of surgery in head and neck area. Patient was using topical ear drops and medication for headache from local doctors. Family history: She is married and has one child and no other significant illness in the family. Personal history: Sleep and appetite are normal. Bowel and bladder habits are normal. Menstrual history: She has regular menstrual cycles and last menstrual period 20 days back.

30

Manual of Clinical Cases in ENT and Head–Neck Surgery

General physical examination: Average body built. She has no pallor, cyanosis, icterus, koilonychias, clubbing, lymphadenopathy or pedal edema. Pulse rate: 82/min, regular, BP: 120/80 mmHg in left arm supine position. Temperature: Normal, Respiratory rate: 16/min. Examinations of pinna and pre- and postaural region: Both sides pinna and pre-/postaural region are normal. There are no tragal and mastoid tenderness. Examination of external auditory canal: Normal in both sides. Examination of tympanic membrane: Large central perforation (Fig. 3.3) in right side whereas normal in left side.

Fig. 3.3: Large central perforation

Fistula test: Negative in both sides. Facial nerve examinations: Normal in both sides. Tuning fork test: Rinne negative in right side and positive in left. Weber lateralized to the right. ABC is normal in both sides. Q.1: What is the diagnosis? Ans: Right side chronic suppurative otitis media-tubotympanic type. Q.2: Why it is not an unsafe type of CSOM? Ans: • The perforation in the tympanic membrane is central perforation. • Absence of cholesteatoma/granulations. • Mild to moderate conductive hearing loss. • No evidence of complications. Q.3: What are the investigations in safe or unsafe type of CSOM? Ans: Important investigations are: • Culture and sensitivity of ear discharge. • Pure tone audiometry. • X-ray mastoid (lateral oblique view). • CT scan of the temporal bone.

Otitis Media

31

Q.4: What are the organisms isolated from CSOM? Ans: Pseudomonas aeruginosa, Proteus, E. coli, Staphylococcus and anaerobes. Q.5: What is tubotympanic disease? Ans: It is a benign type of CSOM, characterized by copious, mucoid, odorless otorrhoea, a central perforation and mild to moderate conductive hearing loss with rare complications. Q.6: What is a central perforation? Why it is the commonest type of perforation? Ans: Central perforation is confined to the pars tensa and has an intact rim of tympanic membrane all around. It is associated with tubotympanic type of CSOM. The blood supply of tympanic membrane is anterior tympanic branch of maxillary artery and stylomastoid branch of posterior auricular artery. The pattern of blood supply is from periphery to center, making central part of tympanic membrane least vascular and thus more prone for ischemia and thus makes it a commonest site for perforation in otitis media. Q.7: Which perforation in tympanic membrane cause more hearing loss—anterior or posterior? Ans: Posterior perforation causes more hearing loss, since the reciprocal movement of the round window round membrane in response to sound stimulus (round window baffle) is impeded by sound waves directly falling on the same through the perforation. Q.8: Is there hearing improvement in CSOM with discharge? Ans: Yes. Hearing improves at the time of discharge in a large perforation due to the ‘round window shielding effect’ caused by the discharge. Q.9: What will be the management of safe type of CSOM? Ans: The aim of the treatment is to give the patient a dry and normal hearing ear. • Aural toileting. • Antibiotics as per culture and sensitivity report. • Nasal decongestants for making patent eustachian tube. • Treating the possible underlying cause like chronic sinusitis or tonsillitis. • Once ear become dry, myringoplasty or tympanoplasty can be done. Q.10: What are the stages of CSOM during its disease process? Ans: • Active stage: Here the ear is discharging and the mucosa of middle ear will appear to be hypertrophied, velvety and congested. • Inactive stage: There is dry perforation and no discharge at the time of examination. • Quiescent stage: Perforation of the tympanic membrane is present and ear is dry for quite some time. • Healed stage: Perforation of the tympanic membrane is healed with a thin scar.

32

Manual of Clinical Cases in ENT and Head–Neck Surgery

Q.11: What are the causes for sensorineural hearing loss in CSOM? Ans: • Absorption of bacterial toxin through round window. • Prolonged use of ototoxic ear drops. • Erosion of bony labyrinth by cholesteatoma. Q.12: What are the materials used for myringoplasty? Ans: • Temporalis fascia (most commonly used) • Tragal perichondrium • Conchal perichondrium • Tragal cartilage • Dorsal vein graft • AlloDerm (synthetic human dermis) • Areolar tissue which lies superficial to the temporalis fascia. Q.13: What are the prerequisites for tympanoplasty? The prerequisites for tympanoplasty are: • Good cochlear reserve • Dry ear • Good eustachian tube function • Healthy middle ear mucosa • Sound transmission to oval window and sound protection to round window CASE 4: CSOM-AAD (CHRONIC SUPPURATIVE OTITIS MEDIA-ATTICOANTRAL DISEASE)

Presenting complaints: A 15-year-old boy presents with foul smelling scanty discharge from right ear since 5 years. History of present illness: He was apparently alright 5 years back. To start with right ear discharge was started since 5 years. The ear discharge is foul smelling, scanty and throughout the year. Decreased hearing in the right side started 4 years back, insidious in onset and progressive in nature. There was no history of giddiness or tinnitus. He had history of headache since 6 months, insidious in onset, gradually progressive, intermittent, bifrontal, more on bending forward, more in the morning and more during upper respiratory tract infection. No history of visual aura, vomiting, decreased vision, recurrent nasal obstruction or mouth breathing, sneezing or blood stained nasal discharge. History of past illness: No history of diabetes mellitus, hypertension, bronchial asthma, ischemic heart disease, pulmonary tuberculosis or drug allergy and no past history of surgery in head and neck area. Patient was using topical ear drops and medication for headache from local doctors. General examination: Moderately built, nopallor, cyanosis, icterus, clubbing, lymphadenopathy, koilonychias and pedal edema. BP: 122/82 mm Hg in right arm in supine position. Respiratory rate: 14/min afebrile.

Otitis Media

33

Otoscopy examination: There is attic perforation in the pars flaccid of right tympanic membrane (Fig. 3.4).

Fig. 3.4: Attic perforation with cholesteatoma

Tuning fork test: Rinne’s test—negative in the right side and positive in left side. Weber is lateralized to the right ear. Schwabach test—lengthened on the right side while equal on the left side. Absolute bone conduction test—normal on both sides. Q.1: What is the possible diagnosis? Ans: Right side CSOM—atticoantral disease. Foul smelling otorrhea is suggestive of atticoantral disease. Patient has attic perforation, is in favor of CSOM with atticoantral disease (AAD). Q.2: What is the treatment of CSOM-AAD? Ans: As AAD is a bone eroding disease, it has to be exenterated and exteriorized, preferably by an open cavity procedure like modified radical mastoidectomy. The hearing mechanism should be reconstructed by tympanoplasty. Q.3: What are the characteristic features of atticoantral diseases/unsafe type of CSOM? Ans: Attic or posterosuperior marginal perforation in tympanic membrane. It is often associated with cholesteatoma. Patient usually present with foul smelling ear discharge and hearing loss. Q.4: What is the characteristic ear discharge in CSOM-AAD? Ans: Atticoantral type of CSOM/cholesteatoma/marginal perforation is characterized by scanty foulsmelling, painless discharge from the ear. The foul smell is due to saprophytic infection and osteitis. Q.5: What are the causes of blood stained discharge from the ear? Ans: CSOM with granulations, acute otitis media with perforation in TM, acute hemorrhagic otitis externa, Glomus jugulare and malignancy of external or middle ear.

34

Manual of Clinical Cases in ENT and Head–Neck Surgery

Q.6: What is the treatment of choice in atticoantral type of CSOM? Ans: Treatment of choice for atticoantral variety of CSOM is surgery, i.e. modified radical mastoidectomy. Q.7: What are the differences between mucosal and squamous type of CSOM? Ans: Mucosal/tubotympanic

Squamous/aticoantral

1. Ear discharge

Profuse, mucoid/mucopurulent Scanty, thick, purulent, foetid

2. Blood stained discharge

Absent

Usually present

3. Perforation

Central

Attic or posterosuperior marginal

4. Deafness

Conductive

Conductive, mixed

5. Aural polyp

Uncommon/pale polyp

Red polyp

6. Fistula test

Negative

May be positive

7. Headache, vertigo

Usually absent

Present in advanced cases

8. Pathology

In the mucosa of the middle ear cleft

Squamous epithelial ingrowth into middle ear cleft

9. Cholesteatoma

Not present

Present

Absent No bone erosion

Common Evidence of bone erosion

10. Complications 11. Radiology

Q.8: What are the causes of progressive hearing loss? Ans: • Presbyacusis • Meniere’s disease • Otosclerosis • Acoustic neuroma Q.9: What are the important clinical findings in atticoantral diseases and what is the best treatment? Ans: Important clinical findings are: • History of foul smelling ear discharge and may be associated with blood stained discharge. • Sagging of posterosuperior meatal wall. • Marginal or attic perforation of tympanic membrane. • Granulations and/or evidence of cholesteatoma. • Moderate to severe SNHL or mixed hearing loss. • Sometimes complications like facial palsy, brain abscess or meningitis or lateral sinus thrombosis are associated with otitis media. The treatment in atticoantral disease or unsafe ear is mastoid exploration (MRM). Q.10: What is ironed out mastoid? Ans: Ironed out appearance of mastoid is a feature of acute mastoiditis due to thickening of mastoid process because of periostitis.

Otitis Media

35

Q.11: What is erection of pinna? Ans: In erection of pinna, the pinna is pushed forward, outward and downward. It is seen in mastoid abscess. Q.12: What leads to foul smelling discharge from ear? Ans: Foul smelling ear discharge is due to saprophytic bacteria and osteitis caused by cholesteatoma. Causes of foul smelling ear discharge are: • Unsafe type of CSOM. • Granulomatous conditions. • Myiasis. • Foreign body. • Malignancy with secondary infection. Q.13. What are the types of tympanic membrane perforations in unsafe type of CSOM? Ans: • Attic • Posterosuperior marginal • Central (rare) Q.14. What leads to foul smelling discharge from the ear? Ans: In atticoantral/unsafe type of CSOM, the foul smelling ear discharge is due to saprophytic bacteria and osteitis caused by cholesteatoma. The causes of the foul smelling ear discharge are: • • • • •

Atticoantral type of CSOM Granulomatous condition Myiasis Foreign body in ear Malignancy of the ear with secondary infections.

CASE 5: TUBERCULAR OTITIS MEDIA

Presenting complaints: A 30-year-old man presented with painless ear discharge since 2 years. He had hearing loss since 6 months. Setting: Out patient department. Otoscopic examination: Multiple small perforations seen in the anterior part of tympanic membrane. Q.1: What is the most possible diagnosis? Ans: Tubercular otitis media. Q.2: What are the classical features of the tubercular otitis media? Ans: Multiple perforations in tympanic membrane, pale granulations in ear, painless ear discharge, early complications and hearing loss is out of the proportion to disease.

36

Manual of Clinical Cases in ENT and Head–Neck Surgery

Q.3: How the diagnosis of the tubercular otitis media is confirmed? Ans: Histological examination of the granulations from the middle ear, chest X-ray, PCR for TB. Q.4: What are the characteristic features of otoscopic picture in TB otitis media? Ans: The ear discharge is serosanguinous or sometimes blood stained, granulation even in central perforation and the edge of the perforation is pale, thin and undermined. Q.5: What are the types of the TB otitis media? Ans: Primary tuberculosis, often seen in pediatric population. Secondary tuberculosis otitis media occurs secondary to pulmonary tuberculosis which is common among young adults. Q.6: What is the treatment of tuberculosis otitis media? Ans: Antitubercular therapy. Q.7: What are the lesions in the middle ear causing conductive hearing loss? Ans: The middle ear lesions causing conductive hearing loss are: • Eustachian tube dysfunction. • Acute otitis media. • Middle ear effusion. • Adhesive otitis media. • Tympanosclerosis. • Hemotympanum. • Tympanic membrane perforation. • Cholesteatoma. • Ossicular chain discontinuity. • Ossicular fixation as in otosclerosis. • Congenital absence or malformation of the ossicular chain. • Tumors of the middle ear. • Previous middle ear surgery. CASE 6: GRADENIGO SYNDROME

Presenting complaints: A 35-year-old man presented with right ear discharge since 10 years, right side orbital pain since one month and complaining double vision since 15 days. Setting: Out patient department. History of present illness: He was apparently alright 10 years back. To start with right ear discharge since 10 years. The discharge was scanty, foul smelling and persisting throughout the year. He has associated with double vision and retro-orbital pain. History of past illness: He has no diabetes, hypertension, tuberculosis and not associated with any systemic diseases.

Otitis Media

37

Q.1: What is the most probable diagnosis? Ans: Gradenigo syndrome. The triad of otorrhea, retro-orbital pain and diplopia in apical petrositis is known as Gradenigo’s syndrome. Q.2: What is the cause of Gradenigo syndrome? Ans: It is a triad, consists of ear discharge, diplopia and retro-orbital pain. It is due to petrositis which is a complication of otitis media. Petrous apex may be pneumatised in about 30% cases with connecting cell tracts from the mastoid. This may hence predispose to spread of infection to petrous apex from the middle ear cleft suppuration. There is involvement of 5th and 6th cranial nerves which are close to the petrous apex. It can also be seen in extradural abscess and meningitis of the petrous apex. Q.3: What is the cause of diplopia and retro-orbital pain in petrositis? Ans: Diplopia is due to lateral rectus palsy due to abducent nerve involvement and retro-orbital pain is due to trigeminal neuralgia. Abducent and trigeminal cranial nerves passes near the petrous apex, involved in apical petrositis. Q.4: How cranial nerves are involved in the petrositis? Ans: The 6th cranial nerve runs in the Dorello’s canal (canal in between the petrous tip and sphenoid bone) at the petrous apex and the 5th cranial nerve lies in the Meckel’s cave which is a concavity on the anterior slant of petrous part of the temporal bone near the petrous apex in which the 5th nerve ganglion (gasserian ganglion) rests. So in petrositis, the two cranial nerves are commonly involved. Q.5: What is the treatment of petrositis? Ans: Petrositis is managed by antibiotics and mastoid exploration along with curettage of the fistulous tract to drain abscess from the petrous apex. CASE 7: CHRONIC OTITIS MEDIA WITH ATTICOANTRAL DISEASE

Presenting aymptoms: A 15-year-old boy presenting with right ear discharge since 3 years. The discharge was scanty and foul smelling. Otoscopic examination: Posterior superior retraction pocket with cholesteatoma in the right ear. Q.1: What is the diagnosis? Ans: Posterior superior retraction pocket is usually associated with primary acquired cholesteatoma which is turn causes atticoantral or unsafe type of CSOM. Q.2: What is the treatment of choice? Ans: Surgery is the treatment of choice. Mastoid exploration is the treatment. It may be canal wall down and canal wall up procedure. Surgery of the choice is modified radical mastoidectomy. Q.3: What is canal wall down procedure? Ans: Here mastoid cavity is open into the external auditory canal by removing the posterior bony meatal wall so that disease area is fully exteriorized.

38

Manual of Clinical Cases in ENT and Head–Neck Surgery

Q.4: What are the common canal wall down procedures? Ans: Modified radical mastoidectomy, radical mastoidectomy and atticotomy. Q.5: Why the hearing loss in atticoantral disease is termed ‘Variable’? Ans: As the atticoantral disease is a bone eroding disease, so associated with moderate to severe conductive hearing loss because of the ossicular disruption or mixed hearing loss due to inner ear involvement. Sometimes, there may be little hearing loss as the cholesteatoma matrix may bridge the disrupted ossicles in the middle ear providing an efficient sound conduit (bridge cholesteatoma). CASE 8: LATERAL SINUS THROMBOPHLEBITIS

Presenting complaints: A 32-year-old male having attic cholesteatoma of the left ear with hectic picket fence type of fever (fever with rigors, chills and sweating). He has also complain of headache. Examination: Tenderness over the left side internal jugular vein. There is edema over the left side mastoid area. Q.1: What is the possible diagnosis? Ans: Chronic suppurative otitis media of the left ear along with lateral sinus thrombophlebitis. Q.2: What is Griesinger’s sign? Ans: Edema appears over the posterior part of the mastoid. This is due to thrombosis of mastoid emissary vein. It is found in lateral sinus thrombophlebitis. Q.3: What is Crowe-Beck test? Ans: Pressure on jugular vein of healthy side produces engorgement of retinal veins (seen by ophthalmoscopy) and supraorbital veins. Engorgement of veins subsides on release of pressure. This is seen lateral sinus thrombophlebitis. Q.4: What is Tobey-Ayer test? Ans: This is to record CSF pressure by manometer and to see the effect of manual compression of one or both jugular veins. Compression of vein on the thrombosed side of internal jugular vein produces no effect while compression of the vein on healthy side produces rapid rise in CSF pressure which will be equal to the bilateral compression of jugular veins. Q.5: What is the radiological investigation of choice in lateral sinus thrombosis? Ans: MRI sacn; both T1 and T2 weighted images show a hyperintense signals o n MRI with absent flow. In contrast enhanced CT scan, the delta sign is demonstrated where the walls of the sinus enhance while the lumen does not due to thrombosis. Q.6: What are the complications of lateral sinus thrombosis? Ans: Meningitis and subdural abscess, cerebellar abscess, septicemia with metastatic abscess, jugular vein thrombosis with multiple cranial nerve palsy, cavernous sinus thrombosis and otitic hydrocephalus.

84

Manual of Clinical Cases in ENT and Head–Neck Surgery

CASE 5: ACUTE VERTIGO

Presenting Symptoms A 35-year-old lady complaints of vertigo spells 4–8 times in the last three days. She felt like the room was spinning and it lasted for few seconds. She experienced vertigo mostly when she changed positions. The vertigo spells seemed to resolve on its own after several seconds. Examination: • No associated neurologic symptoms. • Patient denied hearing loss, otalgia, otorrhea, aural fullness, tinnitus. • She also denied noise exposure, ear trauma, prior ear surgery, or family history of hearing loss. Q.1: What is the provisional clinical diagnosis? Ans: This is a case of BPPV with acute attack. Q.2: How the BPPV is diagnosed? Ans: Dix-Hallpike test. Q.3: What is the treatment of choice in BPPV? Ans: • In posterior semicircular canal BPPV, Epley’s maneuver, Brandt-Daroff positional exercises and Semont maneuver are useful. • Barbecue maneuver is useful treatment for horizontal canal BPPV. Q.4: What are the surgical treatment done in BPPV? Ans: Surgery is done for intractable cases not responding to particle repositioning procedures or vestibular rehabilitation. Surgical procedures for BPPV of the posterior semicircular canals are singular neurectomy and posterior semicircular canal occlusion. Q.5: What are the investigations are done in vertigo patients? Ans: • Blood investigations—full blood count, blood urea, serum creatinine, fasting blood sugar, thyroid function test, VDRL and autoimmune screening. • Audiogram—pure tone audiometry to rule out hearing loss. • MRI scan is helpful in case of brain tumor, strokes or multiple sclerosis. • ECG is done to rule out cardiac causes for dizziness such as myocardial infarction.

40

Manual of Clinical Cases in ENT and Head–Neck Surgery

Q.4: What is the treatment of the unsafe type of CSOM with facial nerve palsy? Ans: The treatment should be urgent mastoid exploration. Modified radical mastoidectomy with decompression of the facial nerve in the fallopian canal is commonly opted in this case CASE 11: MYRINGOPLASTY

Presenting complaints: A 28-year-old man with known case of right side chronic otitis media with tubotympanic disease underwent right side cortical mastoidectomy and type-I tympanoplasty. Q.1: What is myringoplasty? Ans: Closure of perforation of pars tensa of the tympanic membrane is called myringoplasty. Q.2: What is tympanoplasty? Ans: The tympanoplasty operation consists of both eradication of middle ear disease and reconstruction of hearing mechanism including tympanic membrane and ossicles. It may be done with or without mastoidectomy. Q.3: What is type III tympanoplasty? Ans: It is also called myringostapediopexy or columella tympanoplasty. Here malleus and incus are absent. Here graft lies on stapes suprastructure. Q.4: What is type IV tympanoplasty? Ans: Here, malleus, incus and stapes suprastructure are absent and graft is placed between the oval and round windows to create an air pocket around the round window. This narrow middle ear space is called cavum minor, is a mucosa lined space that extends from eustachian tube to the round window. Sound waves in thits type IV tympanoplasty directly hit on the footplate while the round window has been shielded by the cavum minor. Q.5: What is type V tympanoplasty? Ans: In type V tympanoplasty, a window (fenestration) is created on the horizontal semicircular canal that is covered with a graft. Q.6: What are the contraindications for myringoplasty/tympanoplasty? Ans: • Active infection/discharge in the middle ear • Otitis externa • Nasal allergy • When other ear is dead or not suitable for hearing aid rehabilitation • Ingrowth of squamous epithelium into middle ear • Children below 3 years Q.7: What is difference between type I tympanoplasty and myringoplasty? Ans: Though both refer to repair of tympanic membrane, type I tympanoplasty entails exposure of middle ear to inspect the middle ear and also ensures the ossicular integrity.

Otitis Media

41

Q.8: What are the materials used for closure of the perforation of tympanic membrane in myringoplasty/tympanoplasty? Ans: • Temporalis fascia (most common) • Areolar fascia overlying the temporalis fascia • Perichondrium from the tragus • Cartilage • Vein • Periosteum Q.9: If the patient has central perforation with sensory neural hearing loss, are you going to get tympanoplasty? Ans: Yes, because patient may be fitted with hearing aid as persistent discharge in non-operated ear will not allow to get hearing aid. Q.10. What are the complications of the tympanoplasty? Ans: • Displacement/extrusion of graft • Injury to chorda tympani nerve • Injury to facial nerve • Tympanic membrane residual perforation • Vertigo • Sensorineural hearing loss • Perilymph fistula Q.11: If this patient refuses for surgery, what are the problems patient may come across? Ans: • Recurrent discharge • Gradual hearing loss • Tympanosclerosis • Adhesive changes • Very rarely complications may arise due to formation of cholesteatoma CASE 12: CSOM WITH LABYRINTHINE FISTULA

Presenting complaints: A 35-year-old man presented with foul smelling, scanty and blood stained discharge in right ear since 8 years. He had giddiness during cleaning of the right ear. Otoscopic examination: Attic perforation in the right ear. Fistula test: Positive in right ear. Q.1: What is the diagnosis? Ans: Right CSOM with atticoantral disease with labyrinthine fistula.

42

Manual of Clinical Cases in ENT and Head–Neck Surgery

Q.2: What is fistula test? Ans: The fistula test is performed by applying intermittent pressure on the tragus or by using Siegel’s speculum. Normally the test is negative because the pressure changes in the external auditory canal cannot be transmitted to the labyrinth. Q.3: What is positive fistula test? Ans: The fistula test is positive when there is erosion of horizontal semicircular canal as in cholesteatoma or a surgically created window in the horizontal canal (fenestration operation), abnormal opening in the oval window (poststapedectomy fistula) or the round window (rupture of the round window membrane). A positive fistula test also implies that the labyrinth is still functioning. Q.4: What is false negative fistula test? Ans: It is seen when cholesteatoma covers the site of fistula and does not allow pressure changes to be transmitted to the labyrinth. Q.5: What is false positive fistula test? Ans: This is positive fistula test without presence of a fistula. It is seen in congenital syphilis and in about 25% cases of Meniere’s disease (Hennebert’s sign). In congenital syphilis, stapes footplate is hypermobile while in Meniere’s disease, it is due to the fibrous bands connecting utricular macula to the stapes footplate. In both these conditions, movememtns of stapes footplate of stapes result in stimulation of the utriculation of the utricular macula. CASE 13: CHRONIC EAR DISCHARGE

Presenting complaints: A 68-yea-old man presented with foul smelling discharge from left ear since one year. It was occasionally associated with blood stained discharge. Otoscopic examination: Attic retraction pocket with granulation present in left side. Q.1: What are the causes of the foul smelling ear discharge? Ans: • Unsafe type of CSOM • Myiasis • Granulomatous conditions • Foreign body in ear • Malignancy with secondary infection Q.2: What indicates blood stained discharge from ear? Ans: • Granulations • Polypoidal masses • Macerations during cleaning • Malignancy • Malignant otitis externa

Otitis Media

43

Q.3: What is the cause of foul smelling discharge from ear in atticoantral type of CSOM? Ans: Foul smelling discharge is due to saprophytic bacteria and osteitis caused by cholesteatoma. Q.4: What are the different grades of attic retraction pocket? Ans: In attic retraction pocket • Grade I: Pars flaccida not touching neck of the malleus • Grade II: In contact with neck of malleus • Grade III: Some erosion of scutum, i.e. outer attic wall • Grade IV: Marked erosion of outer attic wall Q.5: What is the most important diagnostic procedure for diagnosis of choesteatoma? Ans: Most important diagnostic procedure in confirming the presence of cholesteatoma is microscopic examination of the patient. Q.6: What is the cause of erosion by cholesteatoma? Ans: Cholesteatoma erodes due to enzymes secreted by perimatrix of the cholesteatoma, i.e. osteoclasts such as collagenase, proteolytic enzymes and acid phosphatase. CASE 14: CORTICAL MASTOIDECTOMY

Presenting complaints: A 25-year-old man underwent right cortical mastoidectomy and type I tympanoplasty for right CSOM with tubotympanic disease. Q.1: What are the other synonyms of the cortical mastoidectomy? Ans: Schwartz operation or complete mastoidectomy or simple mastoidectomy. Q.2: What is cortical mastoidectomy? Ans: Cortical mastoidectomy is complete exenteration of all the accessible mastoid air cells and converting them into a single cavity without disturbing the anatomy and physiology of the middle ear. Q.3: What are indications for cortical mastoidectomy? Ans: • Acute coalescent mastoiditis • Incompletely resolved acute otitis media with reservoir sign • Masked mastoiditis • As an approach for endolymphatic sac decompression, decompression of facial nerve, translabyrinthine or retrolabyrinthine procedures for acoustic neuroma. Q.4: What is Donaldson’s line? Ans: Donaldson’s line is an imaginary line from lateral semicircular canal, bisecting the perpendicular formed by posterior semicircular canal. Importance is that the superior part of endolymphatic sac is locted here.

44

Manual of Clinical Cases in ENT and Head–Neck Surgery

Q.5: What are the different incisions made in case of tympanomastoid surgery? Ans: • Postaural approach: Wilde’s incision • Endaural approach: Lempert’s incision • Transcanal or permeatal approach: Rosen’s incision Q.6: What is Korner’s septum (petrosquamous lamina)? Ans: It represents the site of embryological fusion of the squamous and petrous position of the temporal bone. A plate of bone persists between the two parts called Korner’s septum. Q.7: What is Trautmann’s triangle? Ans: It is atriangle which leads to posterior fossa. Boundaries are superior petrosal sinus superiorly, sigmoid sinus posteriorly and bony labyrinth anteriorly. Q.8: What is sinodural angle? Ans: Sinodural angle is also called Citelli’s angle. It is situated between the sigmoid sinus and middle fossa dura plate. Q.9: What is solid angle? Ans: It is the area where three bony semicircular canals meet. Q.10: What are the burrs used for mastoidectomy? Ans: • Cutting burrs: To open the cortex (for rapid removal of bone) and are used in most of the bony work. • Diamond burr: To smooth the cavity and they are used near facial nerve dura and sinus. Heat production will be more with these burrs and hence require more irrigation. • Polishing burrs: To smooth the cavity at the end of the operation. CASE 15: MODIFIED RADICAL MASTOIDECTOMY

Presenting complaints: A 30-year-old man diagnosed for right CSOM with atticoantral disease in right ear. He underwent right modified radical mastoidectomy under general anesthesia. During surgery extensive cholesteatoma was found. Q.1: What is modified radical mastoidectomy? Ans: This is a canal wall down procedure where disease tissue completely removed from the middle ear and mastoid along with reconstruction of hearing is done. Here middle ear, attic and mastoid cavity is made into a single space. Q.2: What are the salient features for successful modified radical mastoidectomy? Ans: • Adequate reduction of the facial ridge • Complete removal of facial bridge

Otitis Media

45

• Removal of anterior and posterior buttress • Good saucerization of the mastoid cavity • Wide meatoplasty Q.3: What is facial ridge? Ans: The part of the bone which overlies facial nerve canal (vertical segment) is called facial ridge. The ridge should be lowered up to the level of the facial nerve. It is continuous as the posterior buttress. Q.4: What is bridge? Ans: It is a part of posterior canal wall which overlies ossicles and notch of Rivinus. In the modified radical mastoidectomy, bridge is removed and facial ridge is lowered down. Q.5: What are anterior and posterior buttress? Ans: Anterior buttress: It is the part of superior bony canal wall meets with anterior canal wall which forms lateral attic wall (scutum). Posterior buttress: It is part of posterior canal wall where it meets inferior canal wall. Q.6: During modified radical mastoidectomy, bleeding from bone is controlled by? Ans: Bleeding from bone in mastoid surgery is controlled by using bone wax or by diamond burr or by bipolar cautery. Q.7: What are the boundaries of the sinus tympani? Ans: • Medial to vertical segment of the facial nerve • Lateral to postextension of promontory • Inferior to ponticulus and pyramidal eminence • Superior to subiculum Ponticulus: A bony bridge located between the posterior tympanic wall and the promontory. Subiculum: Bony ridge located between the styloid eminence to lip of the round window ridge. Q.8: What are the boundaries of the facial recess? Ans: • Medially by facial nerve • Laterally by tympanic annulus and chorda tympani nerve • Superiorly by short process of incus This is a triangle: The corresponding part in the mastoid antrum is called antrum threshold angle.

46

Manual of Clinical Cases in ENT and Head–Neck Surgery

Q.9: What is COG? Ans: It is a bony or fibrous ridge extends from cochleariform process to tegmen. Facial nerve lies anterior to the cog just before turns into first genu. Q.10: What is Prussak’s space? Ans: It lies medial to pars flaccida, lateral to the neck of malleus and above kateral process of malleus. Anteriorly, posteriorly and superiorly, it is bounded by lateral malleal ligament. Posteriorly, it also has a gap through which the space communicates with epitympanum.

4

Facial Nerve CASE 1: FACIAL NERVE PALSY

Presenting complaints: A 45-year-old lady presents with inability to close his right eye completely since 2 days. Setting: OPD. History of present illness: Patient was alright 2 days back. Infront of mirror, patient saw unable to close the right eyelids completely (Fig. 4.1). She also notices that people at home say her smile is one sided and she finds it is difficult to retain food in the mouth on the right side. No history of ear discharge, no vertigo and no tinnitus. History of past illness: Patient has no history of trauma and no history of ear discharge.

Fig. 4.1: Patient is unable to close her right eye

Q.1: What is the most possible diagnosis? Ans: Bell’s palsy. Q.2: What are the clinical presentations of Bell’s palsy? Ans: Absence of wrinkling of forehead on the affected side. Inability to close the upper eyelid. Eyeball rolls upward in attempted closure of eye (this protective mechanism is called Bell’s phenomenon and this is called Bell’s sign). Absence of nasolabial fold on affected side. Deviation of angle of mouth to the normal side. 47

48

Manual of Clinical Cases in ENT and Head–Neck Surgery

Q.3: What are the topographic tests for facial nerve? Ans: Schirmer’s test, stapedial reflex, taste test and submandibular salivary flow test. Q.4: What are causes of bilateral concurrent facial paralysis? Ans: Guillain-Barre’s syndrome, leukemia, sarcoidosis, Lyme’s disease, rabies, infectious mononucleosis, Moebius syndrome. Q.5: What are the causes of recurrent facial palsy? Ans: Idipathic (Bell’s palsy), Melkersson-Rosenthal syndrome, herpes simplex type I, tumor of VII nerve ipsilateral recurrence seen in malignant tumors contralateral recurrence is seen in benign tumor. Q.6: What are the treatments for Bell’s palsy? Ans: General: Reassurance to the patient, eye care and physiotherapy. Medical treatment: Steroids—adult dose of prednisolone 1 mg/kg/day in divided dose tapered in 15 days. Along with steroid, vasodilators, vitamins and symptomatic treatment will be given. Surgical treatment: Facial nerve decompression relieves pressure on the nerve fibers and improves microcirculation of the nerve. CASE 2: FACIAL NERVE PARALYSIS

Presenting complaints: A 35-year-old man presents with hyperacusis, loss of lacrimation and loss of taste sensation in the anterior 2/3rd of tongue since 15 days. Settting: Out patient department. History of present illness: He was apparently alright 15 days back. He developed the above clinical manifestations with sudden onset. History of past illness: No history of hypertension, DM, TB. Otoscopic examination: Normal external auditory canal and tympanic membrane. Q.1: What is the site of lesion in facial nerve? Ans: Lesion proximal to geniculate ganglion. Q.2: What are the explanations for above clinical manifestations? Ans: Hyperacusis is due to involvement of nerve to stapedius which arises from vertical part of facial nerve. Loss of lacrimation is due to involvement of greater superficial petrosal nerve which arises from geniculate ganglion. Loss of taste sensation in anterior 2/3rd of tongue is due to involvement of chorda tympanic nerve which arises from vertical segment of facial nerve. Q.3: What are the topodiagnostic tests for identifying the lesion of the facial nerve in intratemporal part? Ans: Schirmer test, stapedial reflex, taste test and submandibular salivary flow test. Schirmer test: This test compares lacrimation of the two sides. Decrease lacrimation indicates lesion proximal to the geniculate ganglion as the secretomotor fibers to lacrimal gland leave at the geniculate ganglion via greater superficial petrosal nerve.

Facial Nerve

49

Stapedial reflex: Stapedial reflex is lost in facial nerve lesion above the nerve to stapedius. It is tested by tympanometry. Taste test: Impairment of taste indicates lesion above the chorda tympani. Submandibular salivary flow test: Decreased salivation indicates injury above the chorda tympani. Q.4: What are the branches of the facial nerve? Ans: The different branches of facial nerve are: Greater superficial petrosal nerve. Nerve to stapedius. Chroda tympani nerve. Communicating branch which joins auricular branch of vagus and supplies the concha, retroauricular groove, posterior meatus and outer surface of tympanic membrane. • Posterior auricular nerve (supplies the muscles of pinna, occipital belly of occipitofrontalis and communicates with auricular branch of vagus. • Muscular branches to stylohyoid and posterior belly of digastrics. • Peripheral branches (temporal, zygomatic, buccal, mandibular and cervical). • • • •

Q.5: What is cartilaginous pointer? Ans: Cartilaginous pointer is a landmark for parotid surgery. The facial nerve lies deep and slightly anterior and inferior to the pointer. Cartilaginous pointer is a sharp triangular piece of cartilage of the pinna and points to the facial nerve. CASE 3: TRAUMATIC FACIAL NERVE PARALYSIS

Presenting complaints: A 25-year-old boy presenting with facial nerve palsy in right side and vertigo after a road traffic accident. Q.1: What is the most possible type of fracture of temporal bone? Ans: Transverse fractures of temporal bone. Q.2: What is transverse fracture of temporal bone? Ans: It results from frontal or occipital blow to head and is more likely to cause injury to the labyrinth and facial nerve. This is the less common than longitudinal fracture of the temporal bone. Q.3: What is longitudinal fracture of temporal bone? Ans: They occur due to a blow the side of the head and have less chances of facial palsy. They cause conductive hering loss. Q.4: What is concussion of labyrinth? Ans: The most common cause of high frequency sensorineural hearing loss in cases of head injury is labyrinthine concussion. Q.5: What is Battle’s sign? Ans: Ecchymosis seen over the mastoid in cases of temporal bone fracture.

50

Manual of Clinical Cases in ENT and Head–Neck Surgery

5

Otosclerosis CASE 1: OTOSCLEROSIS

Presenting complaints: A 28-year-old lady presented with hearing loss since 2 years. Setting: Out patient department. History of present illness: She is complaing hearing loss since her pregnancy. Hearing loss in both sides and slowely progressive in nature. No history of ear discharge. Family history: History of hearing loss in mother of the patient in young age. Pure tone audiogram: Conductive hearing loss and bone conduction curve showing notch at 2000 Hz (Fig. 5.1).

Fig. 5.1: Pure tone audiogram of the patient

Q.1: What is the most likely diagnosis? Ans: Otosclerosis. Q.2: What is paracusis Willisii? Ans: Patient of otosclerosis hears better in noisy environment. This is because in noisy surrounding people tend to raise their voice which is well above the threshold 50

Otosclerosis

51

of hearing for the patient. More importantly this patients donot perceive the background noise, which act as masking noise in normal individuals. Q.3: What are the examination findings in otosclerosis? Ans: Commonest tympanic membrane finding is a normal tympanic membrane. In 2–10% cases, the active otosclerotic focus that leads to vascularization of promontory is visualized as a flaming pink discoloration of the tympanic membrane, called Schwartze’s sign. Surgery is usually contraindicated in patients with positive Schwartze’s sign and is an indication for sodium fluoride therapy. Q.4: What are the audiological findings in otosclerosis? Ans: Pure tone audiogram shows conductive hearing loss with Carhart’s notch (dip at 2000 Hz in bone conduction curve). Tympanometry shows As type of curve. Stapedial reflex is absent once the foot plate is fixed. Q.5: What are the pathological changes in otosclerosis? Ans: Mature lamellar bone is replaced by immature spongy bone of more cellularity and vascularity. This changes mainly occurs in the endochondral layer of otic capsule where islands of cartilage are left unossified during development. This spongy bone later on undergoes neo-osteogenesis. Both osteoblastic and osteoclastic activities take place in the otosclerotic process that starts as finger like invasion along the blood vessels. These are sometimes called ‘blue mantles’ (blue mantles of Manasse) since they appear blusih on H&E staining. Q.6: What are the differential diagnoses of otosclerosis? Ans: Otosclerosis should be ruled out from other causes of conductive hearing loss, like • Secretory otitis media • Adhesive otitis media • Tympanosclerosis • Fixation of head of malleus • Ossicular discontinuity • Congenital stapes fixation Q.7: What is Carhart’s notch? Ans: The puretone audiogram in otosclerosis shows conductive hearing loss. The bone conduction is normal. In some cases, there is dip in bone conduction curve. It is different at different frequencies but maximum at 2000 Hz and it is called Carhart’s notch. There is 5 dB at 500Hz, 10 dB at 1000 Hz, 15 dB at 2000 Hz and 5 dB at 4000 Hz. Carhart’s notch disappears after successful stapedectomy. Q.8: What are the findings in tuning fork tests in otosclerosis? Ans: Tuning fork tests show negative Rinne (first for 256 Hz and then 512 Hz and still later, when stapes fixation is complete, for 1026 Hz). Weber test will be lateralized to the ear with greater conductive hearing loss side. Absolute bone conduction may be normal but it is decreased in cochlear ototaclerosis with sensorineural hearing loss. Gelle’s test is negative in otosclerosis.

52

Manual of Clinical Cases in ENT and Head–Neck Surgery

CASE 2: OTOSCLEROSIS

Presenting complaits: A 32-year-old lady complaints with bilateral decrease hearing since 3 years. Setting: Out patient department. Examination: Normal tympanic membrane. Audiological profile: PTA shows bilateral conductive hearing loss. Impedence audiometry shows As type of curve. Stapedial reflexes are absent in both sides. Q.1: What is the surgical treatment? Ans: Stapedotomy (Fig. 5.2).

Fig. 5.2: Stapedotomy

Q.2: What are the contraindications for surgery? Ans: Only hearing ear (absolute contraindication), active/malignant otosclerosis, professionals like pilots, airmen and deep water divers. Pregnancy and extremes are also contraindications. Q.3: Patient of stapedotomy developed vertigo, tinnitus, aural fullness and fluctuant hearing loss. What may be the cause? Ans: Perilymph fistula. It is due to a large fenestra, slippage of prosthesis or barotraumas. It is a potentially dangerous complication due to the risk of meningitis and sensorineural hearing loss. Q.4: What are the treatment options other than surgery? Ans: Hearing aid is a valuable alternative to surgery. If patient want to avoid surgery or unfit for surgery, hearing aid is a better option. Medical treatment with sodium fluoride is indicated in active otosclerotic focus and malignant otosclerosis (rapidly progressive cochlear otosclerosis).

Otosclerosis

53

Q.5: What are the complications of stapedectomy? Ans: • Floating footplate (mostly iatrogenic) • Perilymph fistula • Sensorineural hearing loss • Perforation of tympanic membrane • Vertigo • Injury to chorda tympani nerve (dysgeusia) • Labyrinthitis • Conductive hearing loss due to dislocation of the piston. Q.6: What is neostapedectomy procedure? Ans: The stapedectomy where stapedial tendon and incudostapedial joint are preserved. Q.7: What are the clinical profiles of otosclerosis? The clinical profiles of the otosclerosis are: • Progressive conductive deafness • Age: 20-40 years • Females more than males (2:1) • Commonly bilateral (70%) • Familial tendency present (50%) • Paracusis Willsii (patient hears better in noisy environment) • Normal tympanic membrane • Advanced case may have sensorineural deafness (due to cochlear involvement) • Flamingo red sign in active otosclerosis (Schwartz sign) • Pathology: Replacement of normal lamellar bone by spongy vascular bone increased cellularity and density which fixes the foot plate of stapes • Treatment: Stapedotomy/hearing aid CASE 3: VAN DER HOEVE SYNDROME

Presenting symtpoms: A young girl of 20 years of age has presented with blue sclera, fragile bone and conductive deafness. Examination: Bilateral external auditory canal and tympanic membrane are normal. Q.1: What is the diagnosis? Ans: Van der Hoeve syndrome. Q.2: Which bony disease may be associated with otosclerosis? Ans: Otosclerosis may be associated with osteogenesis imperfecta with history of multiple fractures. The triad of symptoms of osteogenesis imperfecta, otosclerosis and blue sclera is called van der Hoeve syndrome. Lesions of the otic capsule found in osteogenesis imperfecta are histologically indistinguishable from those of the otosclerosis and both are due to genes encoding type I collagen.

54

Manual of Clinical Cases in ENT and Head–Neck Surgery

Q.3: Which viral infections are thought to be the etiology for otosclerosis? Ans: Electron microscopic and immunohistochemical studies have shown RNA related to measles virus. It is likely that otosclerosis is a viral disease as has been suggested for paget’s disease. CASE 4: STAPEDOTOMY

Presenting complaints: A 30-year-old lady underwent stapedotomy in the right ear for otosclerosis. Her hearing improved just after the surgery. Q.1: What is postoperative care advised to the patient? Ans: Antibiotics and anti-inflammatory drugs should be prescribed. Patient is advised to avoid flying, swimming, straining or lifting heavy weight and sneezing. Q.2: How definite diagnosis of the otosclerosis can be made? Ans: Exploratory tympanotomy is helpful for giving definite diagnosis of the otosclerosis and not by CT scan or impedence audiometry. Q.3: What are the selection criteria of patients for stapedotomy? Ans: • Hearing threshold for air conduction should be 30 decibel or worse. It is this level when patient starts feeling socially handicap. • Average air-bone gap should be at least 15 decibel with Rinne negative for 256 and 512 Hz. • Speech discrimination score should be 60% or more. Q.4: What is the role of sodium fluoride in otosclerosis? Ans: Sodium fluoride therapy has a role in helping maturity of active focus to arrest cochlear loss. Q.5: Which crus of stapes is long and thick? Ans: Posterior crus is long, thick and curved in comparision to anterior crus. Q.6: What is Bezold’s triad? Ans: Bezold’s triad is seen in otosclerosis and consists of negative Rinne’s, prolonged bone conduction and raised lower tone limit.

55

Trauma to Middle Ear

6

Trauma to Middle Ear CASE 1: TRAUMATIC PERFORATION OF THE TYMPANIC MEMBRANE

Presenting complaints: An 8-year-old boy come to ENT OPD with complaints of hearing loss after he was slapped in class one day back. Examination: It showed small central perforation with irregular margins. The margin is blood stained. Q.1: What is the treatment of this condition? Ans: Treatment of traumatic perforation of tympanic membrane has high probability of spontaneous resolution provided some of the precautions are observed. Topical ear drops are contraindicated in this cases. Patient need to be followed up for observation and the timely advice. Q.2: What is the next management, if traumatic perforation will not heal? Ans: Myringoplasty. Myringoplasty is the surgical procedure involving simple repair of tympanic membrane perforation in which no ossicular reconstruction is involved. Q.3: What is the tympanoplasty? Ans: It is the surgical procedure which includes eradication of disease from tympanic cavity, reconstruction of hearing apparatus, with/without tympanic membrane grafting and without mastoid exploration. Q.4: What are the materials used for grafting in myringoplasty? Ans: Temporalis fascia (most common), tragal perichondrium, fascia lata, vein graft and dura. Q.5: What are other procedures used for closure of tympanic membrane perforation? Ans: Cauterization—50% trichloroacetic acid, phenol. Splinting of traumatic perforations, closure of prosthesis like paper disk moistened with 1% phenol in glycerine.

55

56

Manual of Clinical Cases in ENT and Head–Neck Surgery

Q.6: What are the different perforations (other than traumatic) in the tympanic membrane and its significance? Ans: Types perforations

Characteristics

Site of pathology

Significance

Central

It is seen in pars tensa with a rim of tympanic membrane around the perforation. It can be of different sizes and different quadrants of the pars tensa.

Tubotympanic type Signifies mucosal or of CSOM; discharge tubotympanic type of is mucoid or CSOM, Cholesteatoma is mucopurulent. uncommon, complications usually absent.

Marginal

Usually seen in the posterosuperior part of the pars tensa; loss of fibrous annulus; bony annulus us exposed; one margin of the perforation is formed by bony annulus and other sides by pars tensa.

Atticoantral type of CSOM; discharge is foetid and purulent. Granulations and blood stained discharge are commonly seen.

It signifies atticoantral/ squamous type of CSOM. Cholesteatoma may be present and various fatal complications may occur

Attic

It is seen in the pars flaccid.

Similar to marginal perforation.

Same as marginal perforation.

Q.7: Sometimes the trauma cause temporal bone fracture along with injury to middle ear. How we will differentiates the different fractures of the temporal bone? Longitudinal fracture

Transverse fracture

Frequency

Common (80%)

Less common (20%)

Injury type

Parietal blow

Occipital blow

Bleeding from ear

Common

Absent

Fracture line

Parallel to long axis of petrous pyramid

Run across the petrous.

CSF otorrhea

Present

Absent

Hearing loss

Conductive

Sensorineural hearing loss

Vertigo

Less/uncommon

Severe

Facial palsy

Less common

More common

57

Middle Ear Tumor

7

Middle Ear Tumor CASE 1: GLOMUS TUMOR

Presenting complaints: A 43-year-old lady presented with hearing loss and pulsatile tinnitus in the right ear since two year. Otoscopic examination: It showed a red mass in inferior part of the middle ear behind the intact tympanic membrane. The mass blanched on applying positive pressure with siegle pneumatic speculum. Q.1: What is the most probable diagnosis? Ans: Glomus tumor. Glomus tumor is a benign, nonencapsulated, slow growing, locally invasive and vascular tumor from the glomus bodies (resembling carotid bodies) are present at dome of jugular bulb, hypotympanum, promontory and along the tympanic plexus, arising from the tympanic branch of IX nerve (Jacobson’s nerve). It is also known as chemodectoma as it resembles carotid bodies. Glomus tumor is common in people between 40 and 50 years of age. It is 5 times more common in females. Q.2: What is the blood supply of the glomus tumor? Ans: It usually arises from the tympanic branch of ascending pharyngeal artery. Q.3: How glomus tumors are classified? Ans: According to Oldering and Fisch, classification of glomus tumor is: Type I: Tumor localized to the middle ear cleft. Type II: Tympanomastoid tumors with no destruction of infralabyrinthine compartment. Type III: Tumor involving infralabyrinthine region. Type IV: Tumor with intracranial extension. Q.4: What are the clinical manifestations of the glomus tumor? Ans: Slowly progressive conductive hearing loss, tinnitus, otoscopy shows rising sun appearance in case tumor at hypotympanum, Brown’s sign is positive (blanching of tympanic membrane after extering positive pressure over tympanic membrane by Seigle’s speculum). Bleeding from the ear. Ear pain and facial palsy in tympanomastoid group. 57

58

Manual of Clinical Cases in ENT and Head–Neck Surgery

Q.5: What is CT picture in glomus tumor? Ans: Phlep’s sign refers to the absence of the normal crest of bone (Crotch) between the carotid canal and jugular fossa. Q.6: What is the four vessel angiography and its importance? Ans: It is done to find out extent of the tumor, compression of internal carotid artery and other associated carotid body tumor or embolization of the tumor. It is helpful in assessing brain perfusion study. Q.7: What are the treatment options of glomus tumor? Ans: Surgery is definitive treatment. Radiotherapy in very elderly and inoperable tumor, residual or recurrence. Embolization to reduce vascularity is done preoperatively or in inoperable cases or following radiation. Surgical procedure depends on the extent of the tumor Type I: Transmeatal approach and excision. Type II: Extended facial recess approach. Type III: Fisch infratemporal fossa approach (lateral approach). Type IV: Skull base approach and posterior fossa craniotomy. CASE 2: GLOMUS TUMOR

Presenting complaints: A 28-year-old man presenting with bleeding from ear, tinnitus and progressive deafness. On examination: Examination, there is a red swelling behind the intact tympanic membrane that blanches on pressure with pneumatic speculum. Q.1: What is the diagnosis? Ans: The diagnosis is with typical history and positive Browne’s sign (blanching of the tympanic membrane by positive pressure in external auditory canal) is obviously glomus tumor. Q.2: What is the appearance of the tympanic membrane in glomus tumor? Ans: Rising sun appearance. Q.3: What is Aquino’s sign? Ans: Compression of carotid artery leads to blanching of the glomus jugulare tumor. Q.4: What is the treatment of glomus tumor? Ans: Surgery is the treatment of choice with preoperative DSA with embolization to reduce vascularity. Q.5: What is the role radiotherapy in glomus tumor? Ans: Radiotherapy may be employed in those patients with glomus tumor who are unfit for surgery or in extensive inoperable lesions.

59

Meniere’s Disease

8

Meniere’s Disease CASE 1: MENIERE’S DISEASE

Presenting complaints: A 40-year-old man complains dizziness since 3 months. Setting: OPD History of present illness: He was apparently alright 3 months back. To start with giddiness of 4 attacks. Each attack was persisting for 2 to 3 hours and associated with nausea and vomiting. He also complained hearing loss, tinnitus and aural fullness. Past history: He is a known case of hypertension under treatment and nondiabetic. He has no history of ear surgery and no history of noise exposure or ototoxic medications. Local examination: Pinna, external auditory canal and tympanic membrane are normal in both sides. Tuning fork test: Rinne positive in both ears whereas Weber lateralize to the right. Fistula test: Negative. Q.1: What is the most probable diagnosis? Ans: Meniere’s disease. Meniere’s disease is characterized by episodic vertigo, fluctuating sensorineural hearing loss, tinnitus and aural fullness. Its peak incidence is at age 40 to 60 years. It can also be seen inchildren but less often than adults. Q.2: What is the pathogenesis of Meniere’s disease? Ans: Pathogenesis is endolymphatic hydrops—fluid distension of the endolymphatic spaces leading to rupture of the membranous labyrinth causes mixing of endolymph and perilymph leads to ionic disturbances, causes clinical manifestations. Hydrophs mainly affect cochlea (scala media), followed by saccule, utricle and semicircular canals. Least affected site is endolymphatic duct and sac. Q.3: What is Lermoyez syndrome? Ans: It is a variant of Meniere’s disease, where initially there is deafness and tinnitus, vertigo appears later, when deafness improves. Q.4: What are the characteristic audiometric findings in Meniere’s disease? Ans: • Pure tone audiogram typically shows low frequency sensorineural deafness initially. 59

60

Manual of Clinical Cases in ENT and Head–Neck Surgery

• Since it is a cochlear lesion, there is a high score in SISI indicating recruitment and absent tone decay. • Electrocochleography shows widening of SP/AP ration (Normal 30–40%). Q.5: What is the glycerol test and how it is useful in Meniere’s disease? Ans: There is improvement of hearing thresholds and SP/AP ratio after oral administration of glycerol, a dehydrating agent that brings down the hydrops. This is diagnostic and signifies a good prognosis for diuretic therapy and endolymphatic sac shunt surgery. Q.6: What is the medical management of Meniere’s disease? Ans: • Low salt diet • Labyrinthine sedatives • Diuretics to reduce endolymphatic accumulation • Stop smoking, limit alcohol and caffeine • Stress reduction Q.7: What is Meniett apparatus (Fig. 8.1)? Intermittent low pressure therapy is used in the pressure treatment of Meniere’s disease. After placing a grommet through myringotomy, the apparatus is palced in the canal, which thought to bring down hydrops by influencing the round window membrane pressure receptors and thus bringing down the endolymphatic pressure.

Fig. 8.1: Meniett apparatus

Q.8: What are the hearing conserving surgeries done in Meniere’s disease? Ans: Vestibular neurectomy, endolymphatic shunts surgery, endolymphatic sac decompression. Labyrinthectomy is the hearing destroying surgery. Q.9: What is the role of intratympanic gentamicin in Meniere’s disease? Ans: As gentamicin is selectively vestibulotoxic, locally instilled gentamycin in the middle ear can diffuse into the inner ear through the round window membrane, destroying the dark cells of vestibular labyrinth, thus bringing down vertigo. This is

Meniere’s Disease

61

also called medical/chemical layrinthectomy. Though theoretically there is chance of cochlear damage, usually it is negligible. Gentamicin may be instilled into the middle ear by microwick/microcatheter timed release system, kept through a grommet. Q.10: What are the differential diagnosis of vertigo, tinnitus and sensorineural deafness? Ans: • Meniere’s disease • Syphilitic/autoimmune labyrinthitis • Cogan’s syndrome • Transverse fracture temporal bone • Perilymph fistula • Acoustic neuroma/meningioma (CP angle lesion) • Multiple sclerosis • Hypothyroidism/hyperlipidemia Q.11: What are clinical profile of the Meniere’s disease? Ans: Cardinal symptoms • Episodic vertigo • Fluctuating deafness • Tinnitus Other symptoms • Aural fullness • Nystagmus • Headache • Anxiety state Pathology • Endolymphatic hydrops

CASE 2: MENIERE’S DISEASE

Presenting complaints: A 48-year-old male presented with sudden onset of rotatory vertigo associated with vomiting since one month. There was sudden onset of aural fullness and tinnitus in the right ear. History of present illness: There was a history of six episodes of rotatory vertigo during last one month. There was no loss of consciousness, no history of falls, visual symptoms or headache. History of past illness: The patient was not a known case of diabetic or hypertensive. Neurological examination: Normal. Caloric test: Hypoactive response on the right side. MRI of brain and inner ear: Normal.

62

Manual of Clinical Cases in ENT and Head–Neck Surgery

Q.1: What is the most possible diagnosis? Ans: Meniere’s disease. Q.2: What are the important clinical features of Meniere’s disease? Ans: The different clinical features are: • Age and sex—it occurs in the age group of 40–50 years, men are more commonly affected. • Unilateral ear involvement is more common (85%). • Acute exacerbations and remissions are characteristic. • Vertigo is sudden, severe and occurs in episodes. Patient is normal in between attacks. Sometimes Meniere’s disease shows Tullio’s phenomenon (noise induced vertigo). • Hearing loss—progressive, fluctuating sensorineural hearing loss, often exacerbates during attacks. • Intolerance to loudness (recruitment). • Diplacusis or distortion of sounds typically perceived by the patient while listening to a telephonic conversation. • Tinnitus—fluctating, low pitched tinnitus. It may be roaring or hissing type. • Fullness in ear. • Nausea and vomiting. Q.3: What is the genetic basis for Meniere’s disease? Ans: Meniere’s disease is autosomal dominant and is more seen in females. Q.4: What is the gold standard investigation used for diagnosis of Meniere’s disease? Ans: ECoG is the gold standard for diagnosis of Meniere’s disease. Q.5: What are current modes of management in Meniere’s disease? Ans: Dietary salt restriction (1.5–2 gm/day), vestibular suppressants for the acute crisis, alleviation of anxiety and diuretics. Q.6: What are the other treatment options in Meniere’s disease? Ans: Vestibular rehabilitation therapy enhances and expedites central compensatory mechanisms in the Meniere’s disease. Different surgical procedures have evolved for Meniere’s disease refractory to medical therapy. These can be in the form of conservative surgeries as in endolymphatic sac surgery or in the form of destructive procedures like labyrinthectomy and vestibular nerve section which result in deafferentation of the vestibular end organ.

63

Acoustic Neuroma

9

Acoustic Neuroma CASE 1: ACOUSTIC NEUROMA

Presenting complaints: A 39-year-old male comes to outpatient department of otolaryngology for left sided hearing loss since 3 months. Examination of ear: Both sides external auditory canal and ear drum are normal. Pure tone audiometry: It shows left sensorineural hearing loss and absent left corneal reflex. Imaging: CT scan shows a widened internal auditory meatus. Q.1: What is the most probable diagnosis? Ans: Acoustic neuroma. Q.2: What is the investigation of choice in acoustic neuroma? Ans: MRI brain with gadolinium enhancement is the investigation of choice in this case. Q.3: Most common symptoms in acoustic neuroma? Ans: Progressive bilateral sensorineural (retrocochlear) hearing loss (present in 95% patients) often accompanied with tinnitus (present in 65% patients). Q.4: Where is the origin of acoustic neuroma? Ans: It originates in the Schwann cells of the superior or inferior vestibular nerves at the transition zone (Obersteiner-Redlich zone) of the central and peripheral myelin. Q.5: What are the audiological findings in acoustic neuroma? Ans: It shows features of retrocochlear hearing loss. Recruitment is negative. Speech discrimination score is poor. BERA shows delay of >0.2 msec in wave V between the two sides. Acoustic reflex shows stapedial reflex decay. Q.6: What are the neurological tests findings in acoustic neuroma? Ans: • Reduced corneal reflex • Facial weakness • Cerebellar signs • CSF shows increased proteins • Fundus examination shows papilledema 63

64

Manual of Clinical Cases in ENT and Head–Neck Surgery

Q.7: What are the vestibular symptoms in acoustic neuroma? Ans: • Classical vertigo is not seen and vague imbalance may be a feature (although it arises from vestibular nerve, but because of slow growth and central adaptation, true vertigo does not occur). • Canal paresis is present (response to both hot and cold absent). • Nystagmus is of first degree. • Past pointing and positive Romberg’s test indicate involvement of cerebellum. • Ataxia occus, if tumor is very large one. Q.8: What are the radiological tests done in acoustic neuroma? Ans: • Plain X-ray of internal acoustic meatus not very helpful (Towne and Stenver’s view). • CT scan is useful for small to moderate size tumor. • MRI with gadolinium contrast is the best and gold standard investigation for very small size which may be intracanalicular type. • Vertebral angiography helps to make differential diagnosis from other CP angle tumors such as meningioma, arachnoid cyst, aneurysm or metastasis. CASE 2: ACOUSTIC NEUROMA

Presenting complaints: A 50-year-old man comes to ENT OPD with complaints of deafness, headache and tinnitus in the right ear since 6 months and deviation of angle of mouth and inability of to close right eye for last one month. Past history: There was no history of ear discharge. Tuning fork test: It showed moderate sensorineural hearing loss in the right ear. Q.1: What is the most appropriate investigation in this case? Ans: MRI brain with gadolinium enhancement. This clinical picture is suggestive of CP angle tumor involving VIIth and VIIIth cranial nerve. Acoustic neuroma is the most common CP angle tumor. MRI with gadolinium enhancement is the investigation of choice. Q.2: What are pathologies seen in CP angle? Ans: Acoustic neuroma (most common), meningioma, epidermoid, arachnoid cyst, schwannoma of other cranial nerves (e.g. CN V > VII > IX > X, XI), aneurysm, glomus tumor and metastasis. Q.3: What is the treatment for acoustic neuroma? Ans: Surgical removal of the tumor is the treatment of choice. Surgical approach will depend upon the size of the tumor. Different approaches are: Middle cranial fossa approach, translabyrinthine approach, suboccipital (retrosigmoid) approach and combined translabyrinthine—suboccipital approach.

Acoustic Neuroma

65

Different approaches • Intracanalicular tumor up to 8 mm size with reasonable hearing—middle cranial fossal approach. • Intracanalicular tumor with no hearing—translabyrinthine approach. • Medium size tumor up to 2.5 to 3.5 cm with or without trigeminal nerve involvement—translabyrinthine approach. • Large tumor of more than 2.5 cm with raised intracranial tension—approach wll be suboccipital decompression combined with translabyrinthine approach. • Bilateral medium to large tumor with good hearing—treatment would be retrolabyrinthine with preservation of labyrinth. Q.4: What is gamma knife surgery? Ans: Gamma knife surgery is not an actual surgical treatment but is done through linear accelerator or by cobalt-60 source. It is a type of stereotactic radiotherapy, which is applied in patients those are not fit for surgery or those refuse surgery. Head is fitted on a frame and high dose radiation is focused onto the tumor. Cyberknife is another modification over gamma knife and is more precise and accurate method and works through computer controlled robotics.

66

Manual of Clinical Cases in ENT and Head–Neck Surgery

10

The Deaf Child CASE 1: DEAF CHILD

Presenting complaints: A mother presented at the outpatient department of otolaryngology with her 2 years son who is not vocalizing. She also suspects that he is unable to respond to loud noises. The examination of the motor system is normal. Setting: OPD. Otoscopy: Bilateral tympanic membranes are normal. Q.1 What would be the most appropriate test in this child? Ans: Hearing test (BERA). Q.2: What is the absolute indication for a hearing aid? Ans: Absolute indication of hearing aid is congenital deafness, for satisfactory development of speech and language. Q.3: How hearing aid helps to deaf child? Ans: It amplifies sound reaching the ear. Best suited for patients with conductive hearing loss. In sensorineural deafness; there may be distortion of sound du to recruitment. Q.4: What are the measures are taken to reduce recruitment in hearing aids? Ans: Peak clipping, compression amplification and automatic gain control. Q.5: What is BAHA? Ans: Bone-anchored hearing aid. It acts by direct stimulation of cochlea bypassing external and middle ear, since it is anchored to the bone. It is indicated when mixed or conductive hearing loss not benefited by a conventional hearing aid, e.g. ear canal atresia and active CSOM, which precludes wearing of hearing aid. Q.6: What is Mondini’s dysplasia? Ans: Here the cochlea is 1.5 turns (only basal coil is present). There is incomplete partition between the scalae due to absence of osseus spiral lamina. This type of cochlear deformity is may be seen in Pendred syndrome, Waardenburg syndrome, Treacher-Collins syndrome, Wildervanck syndrome and Branchio-oto-renal syndrome. 66

The Deaf Child

67

Q.7: What are the methods for hearing assessment in infants and children? Ans: • Neonatal hearing screening procedures: Arousal test, auditory response test and ABR/OAEs. • Behavioal observation audiometry: Moro’s reflex, cochleopalpebral reflex and cessation reflex. • Distraction technique. • Conditioning methods: Visual reinforcement audiometry and play audiometry. • Objective tests: ABR, OAEs and impedence audiometry. Q.8: What are essentials for managing the deaf child? Ans: • Parental guidance • Hearing aids • Speech and language development • Education to the deaf child. • Vocational guidance to the deaf child. Q.9: What is hearing aid? Ans: Hearing aid is a device which amplify sounds reaching the ear. It consists of three parts: Microphone, amplifier and receiver. Microphone picks up sounds and convert it into electrical impulses. Amplifier magnifies electrical impulses and receiver converts electrical impulses back to sound. This amplified sound is then carried through the earmould to the tympanic membrane and middle ear. CASE 2: DEAF CHILD

Presenting complaints: A 3-year-old male child presented with severe sensorineural hearing loss was prescribed a hearing aid but showed no improvement. Setting: OPD. Hearing test: An infant is not responding to free field stimulation. BERA shows absent waveform in both ear. Q.1: What is the most possible treatment in above case? Ans: Cochlear implant. Absent waveform in both ear in BERA is suggestive of profound hearing loss in this infant. Cochlear implant is the treatment of choice in this case. Q.2: What is cochlear implant? Ans: It bypasses the cochlea and acts by stimulating the auditory nerve. It is used in severe to profound hearing loss, where there is irreversible damage to the hair cells. Q.3: In cochlear implants, electrodes are placed in? Ans: Commonest site of electrode placement in cochlear implant is the scala tympani of the cochlea. Round window is a route of access into the cochlea in those cases where a separate cocheostomy is not drilled out.

68

Manual of Clinical Cases in ENT and Head–Neck Surgery

Q.4: What are the parts of the cochlear implant? Ans: External components: It consists of external speech processor and a transmitter. The speech processor may be body worn or behind the ear type. Internal component: It is surgically implanted and comprises the receiver/stimulator package with an electrode array. Q.5: What are criteria of candidates for cochlear implant? Ans: Cochlear implant may be done in children and adults. The criteria for candidacy of cochlear implantation are: • Bilateral severe to profound sensorineural hearing loss. • Little or no benefit from hearing aids. • No medical contraindication for cochlear implant surgery. • Realistic expectation from cochlear implant. • Adequate cognitive function to be able to use the device. Q.6: What are the preoperative evaluations for cochlear implant? Ans: Different preoperative evaluations are: • Imaging of the temporal bone, cochlea, auditory nerve and brain is carried out by CT and MRI. • Audiological evaluation include pure tone audiometry, speech audiometry, tympanometry, otoacoustic emissions (OAE), auditory brainstem responses (ABR) and auditory steady state responses (ASSR). • All candidates must be fully vaccinated against meningitis particularly Haemophilus influenzae type B, Pneumococcus and Meningococcus. CASE 3: WAARDENBURG SYNDROME

Presenting complaints: The parent of a 4-year-male child brought the child to outpatient department of ENT with complaint of hearing loss since birth. On examination: The child has white forelock, vitiligo and heterochromia iridis. Pure tone audiometry: Bilateral severe sensorineural hearing loss. Q.1: What is the most possible diagnosis? Ans: Waardenburg syndrome. Q.2: What is Waardenburg syndrome? Ans: It is an autosomal dominant disorder where patient presents with unilateral or bilateral sensorineural hearing loss, white forelock, heterochromia iridis, vitiligo and dystopia canthorum. Q.3: What is Usher syndrome? Ans: Child present with sensorineural hearing loss, retinitis pigmentosa, night blindness. It is an autosoma recessive disorder. CASE 4: CROUZON SYNDROME

Presenting complaints: A 5-year-old male child presents with hearing loss in both ear, exophthalmos with divergent squint (frog eye), hypertelorism, parrot-

The Deaf Child

69

beak nose and mandibular prognatism since birth. He is also a mentally challenged child. On examination: This child has premature closure of the cranial sutures. Pure tone audiometry: Conductive or mixed hearing loss. Q.1: What is the most possible diagnosis? Ans: Crouzon syndrome. Q.2: What is Crouzon syndrome? Ans: It is an autosomal dominant disorder with conductive or mixed hering loss. The patient has frog eyes (exophthalmos with divergent squint). The child has parrot beak nose, hypertelorism, mandibular prognathism and premature closures of the cranial sutures.

70

Manual of Clinical Cases in ENT and Head–Neck Surgery

11

Hearing Loss CASE 1: HEARING LOSS

Presenting complaints: A 30-year-old man presents with hearing loss in right ear following an accident. Setting: OPD. On otoscopic examination: The tympanic membrane was normal and pure tone audiometry shows an air-bone gap of 50 decibel in the right ear with normal cochlear reserve. Q.1: What will be the type of tympanogram in above case and diagnosis? Ans: Ad type tympanogram. The diagnosis is ossicular disruption. Q.2: What are average hearing losses in different lesions of conductive apparatus? Ans: Complete obstruction of ear canal: 30 decibel. Perforation of tympanic membrane: 10–40 decibel. Ossicular interruption with intact ear drum: 54 decibel. Ossicular interruption with perforation: 10–25 decibel. Closure of the oval window: 60 decibel. Q.3: What are the types of tympanogram? Ans: Type A: Normal tympanogram. Type As: Low compliance tympanogram seen in otosclerosis. Type Ad: High compliance tympanogram seen in ossicular discontinuity or laxed tympanogram. Type B: Flat/dome-shaped tympanogram seen in middle ear fluid or thick tympanic membrane. Type C: Negative compliance tympanogram seen in retracted tympanic membrane. Q.4: What is the commonest cause of conductive hearing loss in children? Ans: Secretory otitis media (middle ear effusion) is the most common cause of conductive hearing loss in children. 70

Hearing Loss

71

Q.5: What are the causes of sensorineural hearing loss? Ans: Different causes of sensorineural hearing loss are: • Presbyacusis • Ototoxicity • Trauma • Noise induced hearing loss • Genetic • Systemic diseases like diabetes, syphilis and hypertension • Sudden idiopathic hearing loss • Meniere’s disease • Acoustic neuroma. Q.6: What are the lesions of external auditory canal causing conductive hearing loss? Ans: The lesions are: • Occlusion of lumen due to wax, fungus and foreign body. • Narrowing of external auditory canal due to exostosis, bony stenosis and congenital atresia. Q.7: What are the differences between conductive hearing loss and sensorineural hearing loss? Ans: Conductive hearing loss

Sensorineural hearing loss

• Disease is limited to external ear, middle ear • Disease is beyond the oval window up to footplate of stapes • Rinne test is negative

• Rinne test is positive

• Weber is lateralized to worse ear

• Weber is lateralized to better ear

• Absolute bone conduction (ABC) is normal

• ABC shortened

• Pure tone audiometry shows air bone gap

• Pure tone audiometry shows no air bone gap

• Low frequencies are involved

• High frequency hearing loss

• Hearing loss up to 50–60 dB

• Poor speech discrimination score

• Speech discrimination score is good (95–100%)

• Recruitment test is positive in cochlear lesions

• Recruitment is negative

• SISI score is more than 60% in cochlear lesions.

• Short increment sensitivity index (SISI) is around 15%

• A tone decay of 30 dB is seen in retrocochlear lesions

• No tone decay • Impedence audiometry is a useful test

• Impedence audiometry is not of much use

• BERA has not much use

• BERA is a very useful diagnostic tool

72

Manual of Clinical Cases in ENT and Head–Neck Surgery

Q.8: What are the differences between cochlear and retrocochlear types of sensorineural hearing loss? Cochlear SNHL

Retrocochlear SNHL

• Hair cells of cochlear are damaged

• Lesions in the 8th nerve or its central connections

• Recruitment is positive

• Recruitment is absent

• SISI score is high/positive

• SISI score is less/negative

• No significant tone decay

• Tone decay is significant

• Bekesy shows no gap between I and C tracing (type II)

• Bekesy shows wide gap between I and C tracing (type III)

• Speech discrimination is not highly impaired and no roll over phenomenon is seen

• Speech discrimination score is poor and roll over phenomenon is present

• Subjective feeling of hyperacusis, diplacusis or fullness in the ear

• No such sensation or feeling found

CASE 2: HEARING LOSS

Presenting complaint: A 32-year-old gentleman reports of decreased hearing in the right ear for last two years. Tunign fork test: On testing with a 512 Hz tuning fork the rinne’s test without masking is negative on the right ear and positive on the left ear. With the Weber’s test the tone is perceived as louder in the left ear. Q.1: The most likely patient has what type of hearing loss? Ans: Patient has right side severe sensorineural hearing loss. There is false negative Rinne test in the right side. Q.2: Why 512 Hz tuning fork is most commonly used for hearing assessment? Ans: 512 Hz tuning fork is used for hearing assessment as it has longer tone decay and sound is quite distinct from ambient noise. Tuning fork of low frequencies produce sense of bone vibration while those of higher frequencies have a shorter decay time and so not used routinely. The sound of frequency 512 Hz tuning fork falls in speech range. Q.3: Gelle’s test is done in _____? Ans: Otosclerosis. Gelle’s test was once a popular test to find out stapes fixation in otosclerosis but now has been superceeded by tympanometry. In this test, bone conduction is tested and at same time Siegel’s speculum compresses the air in the external auditory canal. If hearing is reduced when the air is compressed, test is positive. If there is no change in hearing, it is negative. Gelle’s test is positive in normal indiviuals and sensorineural hearing loss. Gelle’s test is negative in otosclerosis.

Hearing Loss

73

CASE 3: HEARING LOSS

Presenting complaints: A 28-year-old lady presenting with bilateral hearing loss since 4 years which worsened during pregnancy. Pure tone audiometry: Revealed bilateral conductive hearing loss Q.1: What is the type of impedence audiometry graph will be? Ans: Type As. Q.2: What is the diagnosis? Ans: Bilateral conductive hearing loss, female patient, 28 years of age and attenuation of hearing loss during pregnancy indicate the diagnosis of otosclerosis. Q.3: What are the different curves of impedence audiometry? Ans: Types of curves

Clincial conditions

A type curve

Normal

As type curve

Otosclerosis Tumors of middle ear Fixed malleus syndrome Tympanosclerosis

Ad type curve

Ossicular discontinuity Poststapedectomy

B type curve (flat or dome shaped curve)

Fluid in middle ear Tympanic membrane perforation Secretory otitis media Grommet in ear

C type curve

Retracted tympanic membrane Eustachian tube dysfunction

CASE 4: HEARING LOSS IN NEWBORN

Presnting complaints: A 6-month-old baby presented no response to sound. Parents of the baby brought the baby to ENT OPD for hearing assessment. Q.1: Which is the investigation of choice for assessement of hearing loss? Ans: Brainstem evoked response audiometry (BERA) Q.2: What is BERA? Ans: It is a non-invasive procedure which helps to find out the integrity of central auditory pathway through the VIII nerve, pons and midbrain. It is the most reliable audiological method of differentiating between cochlear and retrocochlear hearing loss. It is an objective test and can be done under sedation. It is used both as a screening test and as a definite test for hearing assessment in children. It is the best investigation for hearing in infants, mentally retarded child and malingering subjects. It is also used to identify the site of lesion in retrocochlear pathologies.

74

Manual of Clinical Cases in ENT and Head–Neck Surgery

Q.3: What is free field audiometry? Ans: This test is dependent upen the localizing powers of child, that develops at 6 months of age. It is assessed by placing the child in between 2 calibrated loudspeakers in a sound treated room and finding out whether the child is turning its head to direction from which the sound is coming. Q.4: What is behavior observation audiometry (BOA)? Ans: • This hearing test rely on judgment made by subjects are described as subjective and/or behavioral since they depend for reliability on responses of an alert, cooperative and instructed patients. • Here, audiometry signal presented to an infant produces a change in behavior e.g. alerting, cessation of an activity, widening of eyes or facial grimacing. Moro’s reflex is one of them and consists of sudden movement of limbs and extension of head in response to sound of 80–90 decibel. In cochleaopalpebral reflex, the child responds by a blink to a lound sound. In cessation reflex, an infant stops activity or starts crying in response to a sound of 90 decibe. CASE 5: HEARING LOSS AFTER HEAD TRAUMA

Presenting complaints: A patient has presented with unilateral conductive deafness. There is history of head trauma. Examination: The tympanic membrane is normal in appearance. Q.1: What is the etiology of the conductive hearing loss in this case? Ans: Ossicular chain disruption Q.2: What is the type of tympanogram in ossicular discontinuity? Ans: Type Ad. There will be high compliance at or near ambient pressure. Q.3: What is the quantity of conductive hearing loss in ossicular disruption? Ans: 38 to 54 decibel. The amount of conductive hearing loss in: • External auditory canal closure—30 decibel. • Perforated tympanic membrane—10 to 40 decibel. • Oval window closure—60 decibel. The maximum conductive deafness is 60 decibel. Q.4: Commonest cause of conductive hearing loss in adult? Ans: Wax Commonest cause of conductive hearing loss in children is secretory otitis media. Q.5: What is deafness and causes of deafness? Ans: Deafness is due to defective transmission or abnormal perception of the sound wave. The causes may be in the external, middle and inner ear mechanism which are given in Table. 11.1

75

Hearing Loss Table 11.1: Deafness: External, middle and inner ear causes External ear

Middle ear

Inner ear

Wax

Otosclerosis

Presbycusis

Atresia

Otitis media

Meniere’s syndrome

Otitis externa

Trauma

Noise trauma

Foreign body

Tumor

Viral infection

Trauma

Congenital

Ototoxicity

Tumor

Labyrinthitis Vascular Congenital Idiopathic

CASE 6: HEARING LOSS IN CHILDREN

Presenting complaints: A child of 6 years suffering from bilateral conductive deafness detected on school screening. Q.1: What is the commonest cause of hearing loss in this case? Ans: Glue ear or otitis media with effusion. Q.2: What are other names of the glue ear? Ans: The synonyms of the glue ear are: • Otitis media with effusion • Nonsuppurative otitis media • Chronic secretory otitis media • Chronic serous otitis media • Mucoid otitis media • Silent otitis media Q.3: What are the audiological profile in glue ear? Ans: • Tuning fork test: Conductive hearing loss • Pure tone audiometry: Conductive hearing loss • Imedence audiometry: B-type curve NB: Fluctuating deafness of conductive nature is seen in secretory otitis media, while fluctuating sensorineural hearing loss is a feature of Meniere’s disease. Pneumatic otoscopy (Siegalization) is the gold standard for the diagnosis of otitis media with effusion/glue ear. On gentle pressure on the bulb, tympanic membrane will not move, if an effusion is present. Presence of the middle ear fluid is mandatory for the diagnosis of the glue ear and that can be confirmed by pneumatic otoscopy or tympanometry. Q.4: What are the otoscopic pictures in the glue ear? Ans: • Tympanic membrane appears dull • Light reflex is absent

76

Manual of Clinical Cases in ENT and Head–Neck Surgery

• Tympanic membrane is retracted and restricted • Fluid levels and air bubbles are seen through tympanic membrane • Potbelly tympanic membrane is a feature of glue ear. Q.5: What are the treatment options in glue ear? Ans: Medical • Topical decongestants • Antiallergics • Antibiotics Surgical • Treatment of choice—myringotomy and grommet insertion • Surgical management of causative factors, i.e. adenoidectomy/tonsillectomy. CASE 7: TUNING FORK TEST

Presenting complaints: A 35-year-old man reports of decreased hearing in the right ear for last 2 years. Tuning fork test: With 512 Hz tuning fork, the Rinne’s test (without masking) is negative on the right ear and positive on the left ear. With the Webner’s test the tone is perceived louder in the left ear. Q.1: What is the most possible diagnosis? Ans: Right sensorineural hearing loss. Rinne’s test in this case is false negative. Q.2: What is false negative Rinne test? Ans: It is seen in severe unilateral sensorineural hearing loss. During performing Rinne test, patient does not perceive any sound of tuning fork by air conduction but responds to bone conduction testing. This response to bone conduction is, in reality, from the opposite side because of the transcranial transmission of the sound. In such cases, correct diagnosis can be done by masking the non-test ear with Barany’s noise box while testing for bone conduction. Weber test will further help as it gets lateralized to the better ear. Q.3: How to get degree of hearing loss from Rinne test? Ans: For knowing degree of hearing loss, three tuning forks are required such as 256,512 and 1024 Hz. • A Rinne test equal or negative for 256 Hz but positive for 512 Hz indicates airbone gap of 20–30 dB. • A Rinne test negative for 256 and 512 Hz but positive for 1024 Hz indicates air-bone gap of 30–45 dB. • A Rinne negative for all the three tuning forks of 256,512 and 1024 Hz indicates air-bone gap of 45–60 dB. CASE 8: TYMPANOGRAM

Presenting complaints: A 26-year-old lady suffering from bilateral hearing loss for 6 years which become profound with pregnancy.

Hearing Loss

77

Q.1: Which type of tympanogram will be obtained in this case? Ans: Bilateral hearing loss getting worse in pregnancy is due to otosclerosis. Tympanogram in this case would be As type. It is a normal tympanogram with reduced compliance due to stapes fixation. Q.2: What is type A tympanogram? Ans: It is high compliance at or near ambient pressure. This is seen in ossicular discontinuity or thin and lax tympanic membrane. Q.3: What is type B tympanogram? Ans: This is a flat or dome-shaped graph (Fig. 11.1). No change in compliance with pressure changes. It is seen in middle ear fluid or thick tympanic membrane. Q.4: What is type C tympanogram? Ans: Here, maximum compliance occurs with negative pressure in excess of 100 mm H2O. It is seen in retracted tympanic membrane and may show some fluid in the middle ear.

Fig. 11.1: Different types of tympanogram

CASE 9: NOISE INDUCED HEARING LOSS

Presenting complaints: A 35-year-old man work in a factory with exposure to noise and presented with hearing loss since one month. Pure tone audiometry: Audiogram showed bilateral symmetrical sensorineural hearing loss with noth at 4000 Hz. Q.1: What is the possible diagnosis? Ans: Noise induced hearing loss Q.2: What is the noise induced hearing loss? Ans: Noise induced hearing loss is a major cause of preventable sensorineural hearing loss (SNHL). SNHL may follow chronic exposure of noisy occupations, which are less intense sounds than the acoustic trauma. Q.3: What is acoustic trauma? Ans: A single brief exposure to a very intense sound such as an explosion, gunfire or a powerful cracker can result in permanent damage to hearing. Sudden loud sound has the potential to damage outer hair cells, organ of Corti, Reissner’s membrane, tympanic membrane and ossicular chain.

78

Manual of Clinical Cases in ENT and Head–Neck Surgery

Q.4: What are the pathology in noise induced hearing loss? Ans: Noise induced hearing loss damages hair cells, which begin at the basal turn of cochlea. Outer hair cells are affected earlier than inner hair cells. Q.5: What are preventable measures for noise induced hearing loss? Ans: Noise induced hearing loss is preventable. Persons who have to work at places where noise is above 85 decibel should have pre-employment and then annual audiograms for early detection. Ear protections by ear plugs or ear muffs should be used where noise levels exceed 85 decibel. They provide protection up to 35 decibel. If already hearing loss occur, patient may use rehabilitation with hearing aid. CASE 10: OTOTOXICITY

Presenting complaints: A 48-year-old man taking aspirin since two years is developing hearing loss for 2 months. He had no history ear discharge. On examination: Bilateral external auditory canal and tympanic membrane are normal. Pure tone audiometry: Bilateral moderate sensorineural hearing loss. Q.1: What is the most possible diagnosis? Ans: Ototoxicity. Q.2: What are the cochleotoxic drugs? Ans: • Dihydrostreptomycin • Kanamycin • Neomycin • Tobramycin • Amikacin • Furosemide • Ethacrynic acid • Bumetanide • Cisplatin • Nitrogen mustard • Salicylates • Quinine Kanamycin is cochleotoxic and most likely to cause unilateral hearing loss. Sensorineural hearing loss due to quinine, salicylates and furosemide is reversible if drug stopped early. Q.3: What are the vestibulotoxic drugs? Ans: Streptomycin, gentamicin and minocycline. • Streptomycin and gentamicin affect vestibular end organs, causing severe damage to sensory epithelium of cristae of all the semicircular canals. • Minocycline causes reversibe vestibular symptoms like gait disturbances, nausea and vomiting but no nystagmus. Q.4: How ototoxic drugs cause cochlear damage? Ans: In general, outer hair cells are more vulnerable to ototoxicity than the inner hair cells. In aminoglycoside ototoxicty, outer hair cells of basal turn are affected first and then damage spreads to apical turn. Once most of the outer hair cells are damaged, damage to inner hair cells proceeds in reverse order from apex to base.

79

Vertigo

12

Vertigo CASE 1: VESTIBULAR NEURONITIS

Case Presentation A 60-year-old housewife presented with complaints of severe rotator, spinning sensation since one week. She had nausea and vomiting during the episodes. She had also istability on walking since one week. After taking by a general practioner, she had an improvement in her symptoms. However, instability while walking persisted. She had no hearing loss or tinnitus. Past History • She is not diabetic and hypertensive. • She was not taking any ototoxic medication before attack. • She gave history of viral rhinitis about 15 days prior to presentation. Clinical Exmamination • • • • • • •

Both tympanic membranes are normal. Tuning fork tests are normal. All cranial nerves, motor, sensory systems and reflexes are found to be normal. No cerebellar signs seen. Horizontal nystagmus of first degree beating to the left. Unterberger’s stepping test: Abnormal deviation and rotation to the left. Gaze test, positional test and Dix-Hallpike test are normal.

Investigations • Pure tone audiometry is within normal limit. • Electronystagmography (ENG) done at the time of presentation showed spontaneous nystagmus to the left. Caloric test showed right sided hypoactive responses to warm and cold water irrigation. Q.1: What is the most possible diagnosis? Ans: This is a typical case of vestibular neuronitis. Patient presents with severe rotator vertigo along with nausea and vomiting lasting for a week. There are no ear symptoms like deafness, tinnitus, aural fullness, headache, diplopia or any sensorymotor loss. Classically it is preceded by an attack of upper respiratory tract infection, 79

80

Manual of Clinical Cases in ENT and Head–Neck Surgery

yet in most cases (70%), this history may not be present. So absence of history of upper respiratory tract infection does not rule out vestibular neuritis. Q.2: What is the treatment of above case? Ans: • The drug therapy for vestibular neuritis comprises vestibular sedatives and antiemetics, primarily to provide symptomatic relief. The patient may be started with antivertigo drugs. • The main stay of therapy for long term relief is early mobilization and vestibular rehabilitation exercises. CASE 2: BPPV

Case presentation: A 50-year-old male presented with repeated attacks of occasional rotator vertigo with severe nausea but no vomiting. Each attack lasting for about 1 minute for 2 weeks. Symptoms induced on turning usually in bed but sometimes even while sitting. Several times, the symptoms were elicited when the patients tried to get up from bed. He has no deafness, tinnitus, aural fullness and muscular weakness. Examinations Otoscopic examination—normal. Tuning fork test—normal. No cranial nerve palsy. No motor/sensory deficit. Plantar flexor and deep tendon reflex normal. No spontaneous nystagmus. Dix-Hallpike test showed definite rotatory left beating nystagmus (anti-clockwise direction) in left lateral position of head. Nystagmus started after about 10 seconds and accompanied by severe vertigo and mild nausea. The nystagmus and vertigo subsided after about 30–40 seconds. Repeating the position reduced the vertigosignifigantly. • Unterberger’s stepping test, standing test and cerebellar tests were all normal. • • • • • • •

Investigations: Pure tone audiometry—normal hearing. Q1: What is the most possible diagnosis? Ans: This is classical case of left side BPPV. Q.2: What are the causes of episodic short lived rotatory vertigo lasting for seconds? Ans: • Benign paroxysmal positional vertigo (BPPV) • Labyrinthine fistula • Caloric effect • Alternobaric vertigo • Vertebrobasilar insufficiency • Cervical vertigo

Vertigo

81

Q.3: What are the causes for episodic rotatory vertigo lasting less than 24 hours? Ans: • Meniere’s diseasase • Syphilitic labyrinthitis • Delayed hydrops • After middle ear surgery • Compensation of previous vestibular lesions Q.4: What are the causes for prolonged episodic rotatory vertigo lasting more than 24 hours? Ans: • Vestibular neuronitis • Head injury • Labyrinthectomy • Vestibular neurectomy • Labyrinthitis • Metastatic lesions at CP angle • Vascular lesions • Ear surgery CASE 3: MIGRAINE RELATED VERTIGO

Presenting Compaints A 40-year-old male executive reported with recurrent attacks of vertigo each attack persisting for 2 to 4 hours with no accompanying symptoms. There was no precipitating factors that the patient could recall on asking leading questions about any ear symptoms like tinnitus, deafness or aural fullness, the patient categorically denied any related ear symptoms. The vertiginous attacks had been occurring once every 2–3 months for the last 2 years but for the last 3 months it is much more frequent occurring once or twice every week. Important history noted • • • •

Recurrent attacks of vertigo with no accompanying symptoms. No ear symptoms noted. Duration of each symptom is 2–4 hours. Patient state in between two attacks—nothing specified.

Past history • Patient had undergone angioplasty 5 years back for cardiac problems. • He has a mild persistent pain in shoulder 2 years back diagnosed cervical spondylosis for which he is using cervical collar. • History of motion sickness in childhood. • Family history of migraine. • No other relevant past family history.

Manual of Clinical Cases in ENT and Head–Neck Surgery

82

On examination • • • •

Bilateral normal intact tympanic membrane. Rinne’s test positive in both sides. Weber central. ABC normal.

CNS examination • No motor/sensory loss. • No cranial nerve palsies. • No abnormal cerebellar signs. • Plantar flexor, knee jerk normal. • Corneal reflex—normal in both sides. Balance system • Stepping test—slight rotation and deviated to right side more or less within normal limits. • Gait—normal. • Romberg’s test—normal. • Head shaking test—normal. • No spontaneous, gaze and positional nystagmus. Investigations • Cardiac examinations—normal. • Routine blood test—normal. • Pure tone audiometry—within normal limit. • ENG, ECG, VEMP—normal. • MRI Brain—Normal. Q.1: What is the most possible diagnosis? Ans: Migraine related vertigo (MRV) as there is family history of migraine and a past history of motion sickness. Both these factors and the episodic vertigo with no aural symptoms suggest possibility of MRV. Q.2: What is the treatment of above case? Ans: Since there is no definite clinical test to confirm it, a therapeutic trial with prophylactic anti-migraine drugs like flunarizine, amitryptyline and propranolol was instituted. CASE 4: PERILYMPH FISTULA

Presenting Complaints A 45-year-old male patient met with a road traffic accident where he injured his head. CT head was normal. There was a transient loss of consciousness. There was no history of any ear bleeding. The patient was kept for observation overnight. He was discharged on the next day. He felt a little unstable but could walk. After 2 days he started feeling unsteady while walking. There was mild rotator vertigo with no nausea or vomiting. The ear examination was essentially normal. Fistula sign was negative. Valsalva response was not recorded. He had tinnitus on the left side and high frequency SNHL. Tympanogram showed a shallow A-type curve. High resolution computed tomography (HRCT) was repeated which showed no fracture lines or any damage to the otic capsule. He was initially treated on symptomatic treatment hoping

83

Vertigo

that he will recover of his vestibular symptoms. However, his symptoms persisted. During this period he had to travel by air. Though it was a short flight, his symptoms worsened and he had strong vertigo. He sought medical help and had to return by road. The sudden aggravation of symptoms raised suspicion of perilymph fistula. He was taken up for left exploratory tympanotomy. There was thin fluid noted in the middle ear. It was sucked clear but reaccumulated within few minutes. On close observation it was detected to be coming through fracture at the foot plate of stapes. The ossicles otherwise were intact. The fracture was not very obvious because of mucosal folds which were hiding the crack and the leak. The mucosa was swollen. It was divided and raised to either side. A small piece of tragal perichondrium was placed and mucosa replaced. It was not perfect cover by mucosa but enough to hold the graft position. A piece of gel foam under the crura through obturator foramen provided additional support to keep the tissues pressed against the foot plate at least for a few days. He was kept on bedrest with head elevated by 30° for 5 days. He was a muslim by religion but was asked to avoid bending down during his prayers for a period of 3 weeks. His vestibular symptoms started improving the next day and became almost vertigo free in 2 weeks. He was not allowed to fly for 1 month. His hearing showed no improved for the following 3 months. His tinnitus slowely reduced but did not go completely. Point to learn in this case • Post-head injury instability/vertigo can have varied presentations and causes including post-concussion syndrome. Some of them have confusion symptomatology. Often the investigations may not clinch the diagnosis. • The aggravation of symptoms subsequent to sustained pressure variations are strong suggestion towards the diagnosis of perilymph fistula. Probably while testing fistula sign the pressure variations are not sustained long enough. • Proactive approach towards exploratory tympanotomy should be the right way to manage these patients. Endomeatal tympanotomy is such a safe procedure that one can afford to have some negative explorations which should be acceptable. • Hearing improvement and response to tinnitus may be poor. However, some patients do improve significantly if intervened early. Q.1: What are the distinguishing characteristics of peripheral and central vertigo? Features

Peripheral vertigo

Central vertigo

Hearing loss and tinnitus Nausea and vomiting Direction of nystagmus Visual fixation Latency of the nystagmus Fatigable Head thrust sign Recovery Common causes

Common Usually present and severe One direction Inhibits Present Yes Present Begins within days Benign paroxysmal positional vertigo, vestibular neuritis, Meniere’s disease, labyrinthitis

Uncommon Varies Direction changing No change Absent No Absent Slow Migraine, vertebrobasilar insufficiency, multiple sclerosis, cerebrovascular accidents, brain tumors and cerebellar disorders

84

Manual of Clinical Cases in ENT and Head–Neck Surgery

CASE 5: ACUTE VERTIGO

Presenting Symptoms A 35-year-old lady complaints of vertigo spells 4–8 times in the last three days. She felt like the room was spinning and it lasted for few seconds. She experienced vertigo mostly when she changed positions. The vertigo spells seemed to resolve on its own after several seconds. Examination: • No associated neurologic symptoms. • Patient denied hearing loss, otalgia, otorrhea, aural fullness, tinnitus. • She also denied noise exposure, ear trauma, prior ear surgery, or family history of hearing loss. Q.1: What is the provisional clinical diagnosis? Ans: This is a case of BPPV with acute attack. Q.2: How the BPPV is diagnosed? Ans: Dix-Hallpike test. Q.3: What is the treatment of choice in BPPV? Ans: • In posterior semicircular canal BPPV, Epley’s maneuver, Brandt-Daroff positional exercises and Semont maneuver are useful. • Barbecue maneuver is useful treatment for horizontal canal BPPV. Q.4: What are the surgical treatment done in BPPV? Ans: Surgery is done for intractable cases not responding to particle repositioning procedures or vestibular rehabilitation. Surgical procedures for BPPV of the posterior semicircular canals are singular neurectomy and posterior semicircular canal occlusion. Q.5: What are the investigations are done in vertigo patients? Ans: • Blood investigations—full blood count, blood urea, serum creatinine, fasting blood sugar, thyroid function test, VDRL and autoimmune screening. • Audiogram—pure tone audiometry to rule out hearing loss. • MRI scan is helpful in case of brain tumor, strokes or multiple sclerosis. • ECG is done to rule out cardiac causes for dizziness such as myocardial infaraction.

II

Nose and Sinus 13. Diseases of the External Nose 14. Diseases of the Nasal Septum 15. Epistaxis 16. Allergic Rhinitis 17. Sinusitis 18. Nasal Polypi 19. Granulomatous Diseases of the Nose 20. Neoplasms of the Nose and Sinus 21. Headache 22. CSF Rhinorrhea 23. Foreign Body in Nose

13

Diseases of the External Nose CASE 1: CONGENITAL NASAL LESION (MENINGOENCEPHALOCELE)

Presenting complaints: Parents of a 2-year-old child complaining a mass bulging from the nasal cavity of their child since birth. Setting: Out patient department History of present illness: Since birth, child presenting a compressible mass coming from the left nostril. The size of the mass is more during crying. Q.1: A. C. Ans:

What is most possible diagnosis? Ethmoidal cyst B. AC polyp Vascular mass D. Meningoencephalocele D

Q.2: What is encephalocele or meningoencephalocele? Ans: It is herniation of brain tissue along with its meninges through a congenital bony defect. An external meningoencephalocele presents as a subcutaneous pulsatile swelling in the midline at the root of nose (nasofrontal type), side of nose (nasoethmoidal type) or on the anteromedial aspect of the orbit (naso-orbital type). Q.3: What are the investigations needed in the nasal encephalocele? Ans: CT scan with contrast, MRI brain and skull base with contrast, diagnostic nasal endoscopy. Biopsy should be never attempted. Q.4: What are the characteristic features of nasal encephalocele in endoscopy? Ans: Diagnostic nasal endoscopy reveals a soft, cystic, bluish, compressible and translucent mass with positive Furstenberg test. Q.5: What is the treatment of nasal encephalocele? Ans: Endoscopic transnasal excision and closure of the defect. External approach in case of frontoethmoidal meningoencephalocele. Craniotomy is done in case of extensive cranial defect. Q.6: What is glioma and how it differs from nasal encephalocele? Ans: Glioma is a nipped off portion of the encephalocele during embryonic development. There is no definite communication with the cranial cavity exists, except a stalk like connection to the meninges. Glioma does not contain any brain tissue. Clinically, impulse on coughing is negative. 87

88

Manual of Clinical Cases in ENT and Head–Neck Surgery

CASE 2: RHINOPHYMA

Presenting symptoms: A 32-year-old man presenting with an unsightly appearance of a mass at the tip of nose since 3 years. Local examination: Multinodular firm mass with marked fissures and nodular surface on the tip of the nose and adjacent dorsum (Fig. 13.1).

Fig. 13.1: Nodular appearance at the tip of nose

Anterior rhinoscopy: Normal. Q.1: What is the most possible diagnosis? Ans: Rhinophyma. It is an enormous progressive benign hypertrophy and hyperplasia of sebaceous glands as well as connective tissue at the tip of the nose. It is commonly seen among male by about 12 times. It is often called potato nose. Q.2: What is the pathogenesis of the rhinophyma? Ans: Rhinophyma is considered as final stage of acne rosacea. In the early stage of the disease, the dilation of blood vessels with increase blood supply which causes overproduction of connective tissue, forms lobulated mass. It develops progressively in 5 to 20 years. Q.3: What are the clinical symptoms in the rhinophyma? Ans: Cosmetic deformity, nasal obstructon in very large size and foul odor due to accumulation of sebum in between the lobules. Q.4: What is the treatment of rhinophyma? Ans: Different treatment options are surgical decortications, electrosurgical decortications, dermabrasion, excision with thick split thickness skin grafting, laser surgery and radiofrequency surgery. CASE 3: NASAL VESTIBULITIS

Presenting symptoms: A 63-year-old man complaining pain at the hairy part of the nasal cavity.

Diseases of the External Nose

89

Setting: OPD. On examination: Crusting, fissures, excoriation and ulceration of the vestibule (Fig. 13.2).

Fig. 13.2: Nasal vestibule showing crusting and ulcerations

Anterior rhinoscopy: Nasal septum and lateral wall of the nasal cavity were normal. Posterior rhinoscopy: Nasopharynx was normal. Q.1: What is the most possible diagnosis? Ans: Nasal vestibulitis. Q.2: What is nasal vestibulitis? Ans: It is the inflammation of the skin of the nasal vestibule. The common infective organism is Staphylococcus aureus. Q.3: What is the etiology of nasal vestibulitis? Ans: Repeated trauma due to nose picking causes ulceration of the local skin and infection. Excoriation of the skin is seen in persistent nasal discharge due to foreign body, chronic rhinosinusitis, etc. as this encourages infection. Q.4: What is the treatment of nasal vestibulitis? Ans: Treatment involves local cleansing and removal of all crusts gently, use of a topical antibiotic with a steroid combination, and correction of the underlying cause of nasal discharge. Q.5: What is nasal furunculosis? Ans: It is a bacterial infection of the hair follicles of the vestibule of the nose. It is caused by Staphylococcus aureus. Q.6: What are the predisposing factors for causing nasal furunculosis? Ans: Predisposing factors are diabetes mellitus, old age or trauma to the nasal vestibule caused by frequent nose picking or pulling of nasal vibrissae. The organism gain access to the roots of the hair follicles and cause infection.

90

Manual of Clinical Cases in ENT and Head–Neck Surgery

Q.7: Why nasal furuncle is dangerous? Ans: The nasal vestibule lies under the dangerous area of the face which is drained by the facial veins. These veins lack valves and communicate with the cavernous sinus via the ophthalmic veins and the pterygoid venous plexus. Thus infection from the nasal furuncle can easily spread to the cavernous sinus by thrombophlebitis, leading to cavernous sinus thrombosis. So furuncle should not be squeezed or picked. Q.8: What is the treatment of nasal furuncle? Ans: Treatment of furunculosis involves hot fomentation to cause localization of pus and reduce pain, systemic and topical antibiotics to combat the infection and analgesics to reduce pain. Most furunculsis subside with this treatment. Rarely incision and drainage can be done after the pus has localized. If furuncles occur repeatedly, the patient should be screened for diabetes mellitus. Q.9: What is nasal vestibule? Ans: It is a skin lined entrance to the nasal cavity. It contains hair follicles, hair (called vibrissae), sebaceous glands and sweat glands. Q.10: What is dangerous area of face? Ans: Dangerous area of face includes upper lip and anteroinferior part of nose include the vestibule. This area is freely communicates with the cavernous sinus through a set of valveless veins, namely anterior facial vein and superior ophthalmic vein. Any infection in this area can travel intracranially causing meningitis and cavernous sinus thrombosis. CASE 4:CHOANAL ATRESIA

Presenting complaints: A 10-day-old newborn baby presented with difficulty in breathing. The newborn baby does not breath through mouth. Examination: Presence of mucoid discharge in both nostrils. Absence of air bubbles in nasal discharge. Catheter could not pass in both nostrils to pharynx. Q.1: What is the provisional diagnosis? Ans: Bilateral choanal atresia Q.2: What is the etiology for choanal atresia? Ans: Choanal atresia is due to persistence of bucconasal membrane. Q.3: What are the types of the choanal atresia? Ans: Bony type of choanal atresia in 90% of cases and membranous in 10% of cases. Unilateral choanal atresia is more common than bilateral choanal atresia. Unilateral choanal atresia remains undiagnosed until adult life. Bilateral choanal atresia presents with breathing difficulty in newborn. Q.4: What are the diagnostic features for choanal atresia? Ans: • Nasal examination shows presence of mucoid discharge in nose and absence of air bubbles in nasal discharge.

Diseases of the External Nose

91

• Catheter fail to pass from nose to pharynx. • Putting a few drops of methylene blue dye into the nose and seeing its passage through the pharynx. • Installing radio-opaque dye into the nose and taking a lateral film. • CT scan of the paranasal sinus with axial view shows the choanal atresia if present. Q.5: What are treatment options available for bilateral choanal atresia? Ans: • McGovern’s technique—placement of a feeding nipple with a large hole. • Correction of atresia by transnasal or transpalatal approach. CASE 5: TRAUMA TO THE EXTERNAL NOSE

Presenting complaints: A 25-year-old boy presented with trauma to nose after a road traffic accident one day back. On examination: There is swelling over the nose and crepitus sound over the nasal bone on palpation. Q.1: What is the diagnosis? Ans: Nasal bone fracture. Q.2: What are the types of the nasal bone fracture? Ans: Classification of fracture of nasal bones by Lowenthal and modified by Gallon are: • Linear fracture: Simple fracture of the nasal bone without displacement. • Green stick fracture: Incomplete fracture of the one cortex and complete fracture of other cortex. • Simple depressed fracture of one nasal bone or frontal process into nasal fossa. • Lateral displacement fracture of the bony arch with lateral shift of the arch in fragments or as a unit. This is the commonest type and septum is commonly involved. • Severely depressed fracture from a frontal blow. There is severe depression of the dorsum of the nose with crumpling of the nasal septum. There may be impaction of the nasal bones within the frontal process. • Injury to nasal septum: Either cartilage or bone or both may be affected. • Tearing of lateral cartilage: From its attachment to the under surface of the nasal bone. Q.3: How you will manage this case? Ans: • Antibiotics • Analgesics • If lacerated—it should be repaired • Reduction and correction of fracture: Early case—if patient is seen, immediately before swelling of the soft tissues develop, the fracture may be reduced and corrected under local/general anesthesia; within 3–4 weeks time—swelling of the

92

Manual of Clinical Cases in ENT and Head–Neck Surgery

soft tissue usually subsides. Thereafter, the reduction and correction of farcture is performed under local/general anesthesia with help of Asche’s and Walsham forceps and nose is packed for two days and plaster cast is applied over nose; after 4 weeks onwards—Malunion takes place. So rhinoplasty/septorhinoplasty is the choice of treatment; nasal septum—if septum gets deformed or displaced should be corrected within 4 weeks time otherwise SMR/septoplasty should be performed. CASE 6:DERMOID CYST

Presenting complaints: A 5-year-old girl presenting with midline swelling at the dorsum of the nose and it was identified by parents 6 months back. On examination: The swelling is soft, nonpulsatile and fluctuating in nature (Fig. 13.3).

Fig. 13.3: Dermoid cyst

Q.1: What is the provisional diagnosis? Ans: Dermoid cyst Q.2: What are the types of dermoid cyst? • Simple dermoid: It occurs as a midline swelling under the skin but infront of the nasal bones. It does not have any external opening. • Dermoid with a sinus: It is found in infants and children and is represented by a pit or a sinus in the midline of the dorsum of nose. Hair may be found protruding through the sinus opening. There may be intracranial extension. Q.3: What are the differential diagnosis of the dermoid cyst? Ans: Sebaceous cyst, lipoma and meningocele. Q.4: Which investigation is required? Ans: CT scan is done to rule out intracranial extension.

93

Diseases of the External Nose

Q.5: What is the treatment? Ans: Enucleation with appropriate incision. A combined neurosurgical-otolaryngologic approach is needed in those extending intracranially so as to close simultaneously any bony defect through which fistulous tract passed. Q.6: What are the differences between dermoid cyst and sebaceous syst? Ans: Serial number

Dermoid cyst

Sebaceous cyst

1

Skin is free

Adherent skin

2

Puntum—absent

Puntum—present

3

Hair may be present

Usually no hair

4

No sebaceous material expressed

Sebaceous material expressed

5

Lined by squamous cell epithelium

Lined by superficial cell

6

Underlying bone is indented

Does not indent the bone

94

Manual of Clinical Cases in ENT and Head–Neck Surgery

14

Diseases of the Nasal Septum CASE 1: DEVIATED NASAL SEPTUM

Presenting complaints: Doctor, I have nasal obstruction since 6 months. Setting: Out patient department of otorhinolaryngology History of presenting illness: A 25-year-old man complaining left nasal obstruction since 5 years. History of past illness: He does take medication for high blood pressure. He had undergone sinus surgery 5 years back. General physical examination: The patient is well oriented with time, place and person. Pulse rate-90/minute, BP-110/80 mmHg. He has fever of 101°F. Anterior rhinoscopy: Nasal septum is deviated to the right. Examination of oral cavity and oropharynx: Normal. Posterior rhinoscopy: Choana is free. Examination of the neck: Normal. No swelling in the neck Examiantion of the ear, nasopharynx: Within normal limit. Q. 1: What is the diagnosis? What are the causes of unilateral nasal obstruction? Ans: Deviated nasal septum. The causes of unilateral nasal obstruction are deviated nasal obstruction, unilateral choanal atresia, antrochoanal polyp not occluding the nasopharynx, rhinolith, benign/malignant nasal masses and foreign body in the nasal cavity. Q.2: What is empty nose syndrome? Ans: After inferior turbinectomy or middle turbinectomy the patient has a feeling of nasal obstruction (paradoxical) and crusting. This inability to perceive nasal air results in hyperventilation by the patient. It is also seen that air impinging on the nerve ending of the turbinates, results in bronchodilator effect, which is missing in empty nose syndrome. Paradoxical obstruction can be explained by the fact that during surgery nerve endings are cut and so when air impinges on the nasal mucosa, it is not perceived by the person. Q.3: What are the constituents of nasal septum? Ans: A large septal cartilage (quadrilateral cartilage), perpendicular plate of ethmoid, vomer, anterior nasal spine of maxilla, nasal spine of frontal bone, maxillary crest, palatine crest and sphenoid rostrum. 94

95

Diseases of the Nasal Septum

Q.4: What are the types of deviated nasal septum? Ans: Anterior dislocation, C-shaped deflection, S-shaped deflection, spur and thickening of nasal septum (Fig. 14.1).

Fig. 14.1a: Anterior dislocation

Fig. 14.1b: C-shaped

Fig. 14.1d: S-shaped

Fig. 141c: Spur

Fig. 14. 1e: Thickening

Q.5: What is the treatment of deviated nasal septum? Ans: Minor degrees of septal deviation with no symptoms need no treatment. It is the only when deviated septum produces mechanical nasal obstruction or the symptoms need septoplasty or submucous resection (SMR) operation. Q.6: What are the causes of headache in DNS? Ans: The causes are: • Sinusitis (due to obstruction of osteomeatal complex). • Anterior ethmoidal nerve syndrome (Sluder’s neuralgia due to pressure effect of deviated septum on middle turbinate). • Vacuum headache (due to negative pressure created in the sinuses due to abnormal turbulence of inspiratory airflow). Q.7: What are the special tests used to known the patency of the nasal airway? Ans: The tests are: • Cold spatula test • Cottle’s test • Cotton wool test Q.8: What are the investigations done in DNS? Ans: Different investigations done in DNS are: • Diagnostic nasal endoscopy • X-ray paranasal sinuses • Rhinomanometry (not always done) • Routine blood tests (done before septoplasty) Q.9: What are the differential diagnoses of DNS? Ans: A deviated nasal septum itself may not result in nasal obstruction. So we should exclude other conditions for nasal obstruction.

Manual of Clinical Cases in ENT and Head–Neck Surgery

96

These are: • • • • • •

Hypertrophy of the turbinates Concha bullosa Chronic rhinosinusitis Allergic rhinitis Nasal septal edema Nasaopharyngeal pathology.

CASE 2: SEPTOPLASTY

Presenting complaints: A 12-year-old boy presenting with right nasal obstruction since 2 years. Setting: OPD. Anterior rhinoscopy: Gross DNS to the right side. Cold spatula test: No fogging on the tongue depressor in the right side. Q.1: What surgery can be planned in above case? Ans: Septoplasty. Septoplasty is done in all age group and is indicated in when deviation lie anterior to vertical line drawn from nasal process of frontal bone to nasal spine of maxilla (Cottle’s line). Q.2: What are the advantages of septoplasty? Ans: It can be done in children. Flapping of the septum does not occur. Perforation of the nasal septum rarely occur. Revision surgery if needed, can be done easily. Q.3: What are the differences between SMR and septoplasty? Ans: SMR

Septoplasty

Indications

Anterior deviation

Posterior deviation

Incision

Killian’s incision

Freer’s incision

Mucoperichondrium flap

Elevated in both sides

Elevated in only one side

Complications

More

Less

Revision

Difficult

Easy

Q.4: How DNS is diagnosed in infancy? Ans: DNS in newborn is associated with asymmetry of the nostrils. It is diagnosed by anterior rhinoscopy and the use of Gray’s struts. These struts are 4 mm wide and 2 mm thick, which is inserted into the nostrils and gently pushed backward. Normally the strut can be pushed up to 4–5 cm, but in case of deviation, a frank obstruction is seen usually 1.5–2 cm back from the nostrils. This is the most reliable test and well tolerated during infancy. Q.5: What is Cottle’s classification of DNS? Ans: It is classified into three types: Simple—mild deviation of the septum that does not obstruct; obstructed—severe deviation of the septum, which may touch the

Diseases of the Nasal Septum

97

lateral wall of nose, but on applying nasal decongestant, the turbinate shrinks and patient relieves from obstruction; impacted—marked septal deviation, no relieve of obstruction even after instillation of nasal decongestants. Q.6: What are the indications of septoplasty? Ans: The indications of septoplasty are: • DNS with nasal obstruction • DNS with headache • As part of rhinoplasty • To collect septal cartilage as graft material • DNS with epistaxis • Along with FESS to increase space for surgery • Before DCR surgery to get adequate space for surgery • For trans-septal trans-sphenoid approach for pituitary surgery • As part of sleep apnea surgery Q.7: What are different types of the surgical options for DNS? Ans: • Conventional septoplasty • Endoscopic septoplasty • Spurectomy (endoscopic removal of spur) • Submucosal resection of septum (SMR); it has currently almost replaced by the septoplasty. Q.8: What are the complications of septoplasty? Ans: The complications of septoplasty are: • Hemorrhage • Septal hematoma • Septal abscess • Incomplete correction of septal deviation • Recurrence of deviation of the nasal septum • CSF rhinorrhea • Synechia. Q.9: What are contraindications for septoplasty? Ans: • Uncontrolled diabetes and/or hypertension • Bleeding disorders • Acute bacterial or viral nasal infections However, the SMR is avoided before 17 years of age to prevent interference with nasal growth, though septoplasty can be done. Q.10: What is the percentage of newborns with deviation of nasal septum? Ans: 20%.

98

Manual of Clinical Cases in ENT and Head–Neck Surgery

CASE 3: SEPTAL HEMATOMA

Presenting complaints: A 20-year-old boy presented with bilateral nasal obstruction since 5 days. He has mild headache. History of past illness: History of trauma to face 5 days back. On examination: Anterior rhinoscopy shows a bilateral, dull, purplish, smooth, fluctuant, painless swelling of anterior part of nasal septum (Figs 14. 2 and 14.3).

Fig. 14.2: Septal hematoma

Fig. 14.3: Nasal cavity showing septal hematoma

Q.1: What is the most possible diagnosis? Ans: Nasal septal hematoma. Q.2: What are etiologies for septal hematoma? Ans: Trauma, septal surgery, bleeding disorders and idiopathic. Q.3: What are the complications of septal hematoma? Ans: Organisation of hematoma leading to fibrous tissue deposition and thickening, septal abscess and necrosis of septal cartilage causing saddle nose deformity. Q.4: What is the treatment of septal hematoma? Ans: Incision and drainage and bilateral nasal packing. Systemic antibiotics are given to prevent secondary infections. CASE 4: SEPTAL PERFORATION

Presenting complaints: A 29-year-old man presenting with whistling sound during breathing since 2 months. Setting: Out patient department. Past history: History of surgery for deviated nasal septum 6 months back. Anterior rhinoscopy: A small septal perforation at the anterior part of nasal septum (Fig. 14.4).

Diseases of the Nasal Septum

99

Fig. 14.4: Anterior rhinoscopy showing a small perforation in septum

Q.1: What is the most possible diagnosis? Ans: Nasal septal perforation. Q.2: What are traumatic causes for nasal septal perforation? Ans: Trauma is the most common cause for septal perforation. SMR surgery for deviated nasal septum, cauterization of the nasal septum, habitual nose picking. Deliberately perforation of nasal septum for use of ornament. Q.3: What are pathological factors causing septal perforation? Ans: Septal abscess, nasal myiasis, rhinolith, granulomatous diseases like lupus, TB, leprosy, Wegner’s granulomatosis. Q.4: What are the drugs causing septal perforation? Ans: Prolonged use of steroids, cocaine addicts, workers at chromium plating or soda ash. Q.5: What are the common clinical presentations of nasal septal perforations? Ans: The common clinical presentations are: • Small perforations cause whistling sound during inspiration or expiration. • Large perforations develop crusting which may obstruct the nose or cause severe epistaxis when removed. • On anterior rhinoscopy, 90% of perforations are seen anteriorly. • Posterior perforations are seen as result of previous surgery, syphilis or malignancy. • It may be asymptomatic in case of a large perforation. Q.6: What are the investigations done in nasal spetal perforation? Ans: • Hemogram and ESR • VDRL (veneral disease research laboratory), RPR (rapid plasma regain) and FTAABS (fluorescent treponemal antibody absorption test). • cANCA • Biopsy from the edge of perforation or nasal mucosa (sarcoidosis). • Nasal smear for lepra bacilli.

100

Manual of Clinical Cases in ENT and Head–Neck Surgery

Q.7: What are the treatments in nasal septal perforation? Ans: The different treatment options are: • An attempt should be made to find out the etiology before treating the perforation. • Asymptomatic cases do not need any treatment. • Granulations if present need biopsy. • To prevent crust formation, the nasal mucosa should be kept moist by douche. Irrigation of the nose with normal saline or diluted solution of sodium bicarbonate can also be done. • Inactive small perforations can be surgically closed by plastic flaps. Larger perforations are difficult to close. • Sometimes, a thin silastic button can be worn to get relief from the symptoms. Q.8: What are the surgical treatments for nasal septal perforation? Ans: Asymptomatic or posterior large perforations should be left alone and need no treatment. • Pedicled septal flap: Surgical closure of septal perforation can be done by pedicled septal advancement flaps by giving superior or inferior relaxing incisions. The flaps are then sutured at the site of the perforation. A temporalis fascia graft is used between the flaps to strengthen the area. • Sublabial mucosal flap: Closure of the septal perforation can also be done by sublabial mucosal flaps. They are raised from the mucosal surface if the lips and tunneled through the floor of the nose close to the nasal spine. The site of the septal perforation is accessed through Killian’s incision and transfixation or through external rhinoplasty incision. Q.9: Syphilis affects which part of the nasal septum? Ans: Syphilis affects the bony part of the nasal septum while tuberculosis affects the cartilaginous part of the nasal septum. Q.10: What is Sluder’s neuralgia? Ans: Sluder’s neuralgia or the anterior ethmoidal nerve syndrome is pain around the bridge of the nose radiating into the forehead. It is said to originate from the middle turbinate pressing on the septum. CASE 5: SEPTAL HEMATOMA

Presenting complaints: After laparoscopic appendectomy, patient had fall from the bed on her nose after which she had swelling in nose and slight breathing difficulty. Examination: There is smooth bilateral bulge of the septum blocking the nose on both sides. Q.1: What is the next step in management? Ans: This patient has probably septal hematoma which should be drained immediately under local anesthesia. It may result from injuries to nose. Septal hematoma is a collection of blood under the perichondrium or periosteum of the nasal septum.

Diseases of the Nasal Septum

101

Q.2: What are the complications of the septal hematoma? Ans: If infection supervenes the septal hematoma, it results in septal abscess formation giving rise to constitutional symptoms as well. If the hematoma is not drained in time, it may lead to saddle nose due to absorption of the cartilage. Q.3: What are the treatment options of the septal hematoma? Ans: Small septal hematoma can be aspirated with a wide bore sterile needle. Large septal hematoma are incised and drained. CASE 6: DNS WITH HYPOSMIA

Presenting symptoms: A 30-year-old male presented with decreased smell sensation since 6 months. He had nasal obstruction in both nostrils. However, he had no history of sneezinga and rhinorrhea. Anterior rhinoscopy: Gross DNS to the right side and inferior turbinate hypertrophy in left nostril. Q.1: What is the cause of the hyposmia? Ans: Because of the gross DNS to the right side and inferior turbinate hypertrophy in left side, inspired air fails to reach the olfactory region which results in hyposmia. Q.2: What is hyposmia and what are the causes of the hyposmia? Ans: Diminshed sense of smell is called hyposmia. The causes of the hyposmia are: • Deviated nasal septum • Old age (presbyosmia) • Tobacco smokers • Radiation therapy to the sinonasal area • Surgical removal of the sinonasal mucosa • Sinonasal polyposis Q.3: What is parosmia and what are the causes of the parosmia? Ans: Unpleasant change in sense of smell is called parosmia. The causes of parosmia are: • Skull fracture • Injury to the uncus of the temporal lobe • May follow administration of streptomycin or tyrothricin Q.4: What is anosmia and what are causes of anosmia? Ans: • Obstructive lesions in the nose and nasopharynx • Lesions of the sinonasal mucosa • Trauma—surgical or accidental

102

Manual of Clinical Cases in ENT and Head–Neck Surgery

• Neuritis • Central lesions in the brain Q.5: What is hyperosmia and what are the causes of hyperosmia? Ans: Exaggeration of the olfactory sensitivity is termed hyperosmia. The causes of the hyperosmia are: • Epilepsy • Pregnancy, hunger • Strychine poisoning • General paralysis of insane • Cystic fibrosis.

103

Epistaxis

15

Epistaxis CASE 1: EPISTAXIS

Presenting complaints: A 4-year-old boy presented with recurrent nasal bleeding since 2 months. Personal history: He is school going child and habit of nose picking. Setting: Out patient department. Anterior rhinoscopy: A blleding point at the left little’s area of nasal septum. Q.1: What is the cause of nasal bleeding in above child? Ans: Nose picking (digital trauma). Most common cause of epistaxis in child is nose picking. Commonest site of injury by digital trauma is little’s area. Q.2: What are other causes of nasal bleeding in children? Ans: Nasal injury, exanthematous fever, foreign body nose, diphteritic rhinitis and enlarged adenoids. Q.3: What is the treatment of above case? Ans: Advice the child to stop nose picking. Instil normal saline nasal drop for 10– 15 days. If there is active bleeding, anterior nasal pack is given to stop it. Bleeding parameters are tested to rule out any bleeding disorders. Q.4: What are the chemicals used for cauterization of bleeding points? Ans: Silver nitrate and trichloroacetic acid. Q.5: What are the four arteries contribute to Kiesselbach’s plexus? Ans: The most common site of epistaxis is the Little’s area (Kiesselbach’s plexus) which present in the anteroinferior part of the nasal septum. Septal branch of sphenopalatine artery which is the main artery of Kiesselbach’s plexus hence known as artery of epistaxis. Sphenopalatine artery is a branch from the maxillary artery which is a branch of external carotid artery. Septal branch of greater palatine artery which is also a branch of maxillary artery. Anterior ethmoidal which is a branch of ophthalmic artery, arising from internal carotid artery. Septal branch of superior labial artery which is a branch of facial artery which is a branch of external carotid artery. 103

104

Manual of Clinical Cases in ENT and Head–Neck Surgery

Q.6: What is the cause of unilateral epistaxis with obstruction in the same nostril? Ans: Most possible diagnosis is foreign body. CASE 2: HYPERTENSIVE EPISTAXIS

Presenting complaints: A 69-year-old man presents with nasal bleeding from both nostrils since one day. Setting: OPD. Anterior rhinoscopy: No active bleeding. Minimal blood clots in right nostril. General examination: The patient is hypertensive with BP 200/110 mm Hg. Q.1: What is the cause of epistaxis in above case? Ans: Hypertension. Q.2: What is next step in the management? Ans: Observation with hypertension control. Since there is no active bleeding, no targeted treatment is needed against bleeding. Q.3: What are the causes of epistaxis in elderly persons? Ans: Most common cause of epistaxis in elderly person is hypertension. Most common cause of epistaxis in non-hypertensive elderly person is malignant growth of nose, sinus and nasopharynx. Q.4: What are the common sites of epistaxis in the nasal cavity? Ans: Little’s area is the commonest site (90%) and this forms an arterial plexus with greatest contribution from the sphenopalatine artery, which is known as artery of epistaxis. Woodruff’s area forms a venou plexus under the posterior end of the inferior turbinate where sphenopalatine and posterior pharyngeal vessels meet. The other site of bleeding is browne’s area, located to the posterior end of nasal septum. Q.5: What is the first step in the management of epistaxis? Ans: Immediate aim of treatment is to locate the bleeding point and stop bleeding. Subsequently the cause of epistaxis is to find out and treat the cause. Q.6: What is Trotter’s method? Ans: In case of hypertensive epistaxis, Trotter’s method is sometimes useful. The mouth is kept open and patient is sitting upright and inclined slightly forward. The patient is instructed to breath quietly and spit out any blood in the pharynx. Swallowing of blood is discouraged. CASE 3: EPISTAXIS IN ADOLESCENT MALE

Presenting complaints: A 19-year-old boy presented with profuse nasal bleeding one day back. Presently he has no bleeding and coming for check up only. Past history: History of recurrent and profuse nasal bleeding. Anterior rhinoscopy: Normal. Posterior rhinoscopy: A polypoidal pinkish mass in the right side choana.

Epistaxis

105

Q.1: What is the most possible diagnosis? Ans: Nasopharyngeal angiofibroma. This is typical presentation of nasopharyngeal angiofibroma which is almost unique for adolescent males due to hormonal predilection. Q.2: What are the imagings useful in juvenile nasopharyngeal angiofibroma (JNA)? Ans: Lateral radiograpgh skull shows anterior bowing of posterior wall of maxilla due to a tumor enlarging in the pterygopalatine fossa, this is called antral sign or Holman-Miller sign. CT scan evidence of enhancing mass in the nose and pterygopalatine fossa with erosion of the sphenopalatine foramen at the root of the pterygoid plate forms the mainstay of diagnosis. MRI is complimentary to CT and characteristic finding is a ‘Bag of worms appearance’. Angiography may reveal typical blush in the nasopharynx. Q.3: How JNA is diagnosed? Ans: Diagnosis is done by the typical history and radiological signs. Biopsy is contraindicated due to the fear of torrential bleeding. Q.4: What are the treatments in JNA? Ans: Treatment of choice is surgery. Surgical approach of choice is mid-facial degloving approach to the nasopharynx. Others are transpalatal approach (Wilson’s), Lateral rhinotomy plus medial maxillectomy and craniofacial resection (intracranial extension). Presently endoscopic endonasal approach is getting popularity for treatment of JNA. CASE 4: EPISTAXIS FROM ONE NOSTRIL IN A CHILD

Presenting complaints: A 5-year-old child presenting intermittent bleeding from the right nostril since 10 days. He had also bad smell coming from the right nostril. Q.1: What may be the cause of the bleeding from one nostril in such child? Ans: Foreign body in the right nostril. Most common cause of unilateral epistaxis in children is foreign body. Q.2: What are the symptoms found in case of foreign body in a child? Ans: The child often presents with unilateral nasal discharge which is often foul smelling and occasionally blood stained. Q.3: What is the commonest cause of the epistaxis in children? Ans: Finger nail trauma. Q.4: What is TESPAL? Ans: Transnasal endoscopic sphenopalatine artery ligation (TESPAL). This procedure is done with rigid endoscope under topical anesthesia with sedation or general anesthesia. A mucosal flap is lifted in posterior part of the lateral nasal wall, sphenopalatine artery is localized as it exits the foramen and closed with a vascular clip. The distal branches of the artery can be additionally cauterized and the flap then

106

Manual of Clinical Cases in ENT and Head–Neck Surgery

reposited. Anterior ethmoidal artery can also be ligated by Lynch incision as an adjunctive procedure. Sphenopalatine artery ligation gives success in control of refractory posterior bleed. CASE 5: EPISTAXIS DUE TO FUNGAL INFECTION

Presenting complaints: A 48-year-old poorly controlled diabetic man presents with periorbital and perinasal swelling with bloody nasal discharge since one week. On examination: The nasal mucosa is black and necrotic. Ans: Rino-orbital mucormycosis. Q.1: What is mucormycosis? Ans: This is acute invasive fungal infection of the nose and paransal sinuses affecting the lateral nasal walls and quickly spreads to orbit, palate, face and brain. Fungal hyphae invade the blood vessels and cause ischemic necrosis. Treatment includes surgical debridement and amphotericin-B. CASE 5: CONGENITAL CAUSE OF EPISTAXIS

Presenting complaints: A 2-year-old child presenting with recurrent epistaxis from right nostril. On examination: There is telengiectatic lesions at the right side little’s area. There are aso multiple telangiectasia on the skin. Q.1: What is the possible diagnosis? Ans: Osler-Rendu-Weber’s syndrome or hereditary hemorrhagic multiple telangiectasia. Q.2: What are the treatment options in this case? Ans: Electrocautery or skin graft. Ethinyl estradiol daily may be helpful. CASE 5: NASAL ENDOSCOPY IN EPISTAXIS

Presenting complaints: A 25-year-old man presented with nasal bleeding from right nostril since 3 days. Diagnostic nasal endoscopy: Bleeding point found at the right sphenoplatine foramen area, i.e. just posterior-superior to posterior end of the right middle turbinate. Q.1: What is the use of nasal endoscopy in epistaxis? Ans: Nasal endoscopy helps to identify and treat specific sites of bleeding, especially posterior bleeding in the nose. Endoscopic ligation of the sphenopalatine artery below the posterior end of middle turbinate is another advantage. Q.2: How do you decide ligation of vessel in epistaxis? Ans: Regarding ligation of blood vessel, if bleeding is superior, anterior and posterior ethmoidal ligation is done; but if bleeding is inferior and posterior, ligation of internal maxillary artery is carried out.

Epistaxis

107

Q.3: What are the sites for ligation of blood vessel in epistaxis? Ans: Anterior and posterior ethmoidal arteries are ligated between inner canthus of eye and midline of nose. Internal maxillary artery is ligated by Caldwell-Luc approach through its posterior wall in pterygopalatine fossa. Q.4: What are the distances between optic nerve and ethmoidal artery? Ans: The distance between anterior lacrimal crest and anterior ethmoidal artery is 24 mm. The distance between the anterior ethmoidal artery and posterior ethmoidal artery is 12 mm. The distance between the posterior ethmoidal artery and optic nerve is 6 mm. Q.5: What is the Woodruff’s plexus? Ans: Woodruff’s plexus is a venous plexus in the lateral wall of inferior meatus posteriorly. Q.6: What are the five most important causes of nose bleed? Ans: Five important causes of nose bleed are trauma, hypertension, idiopathic, benign lesions and malignancy of nose and paranasal sinuses.

108

Manual of Clinical Cases in ENT and Head–Neck Surgery

16

Allergic Rhinitis CASE 1: ALLERGIC RHINITIS

Presenting complaints: A 19-year-old boy presented with intermittent watery nasal discharge and sneezing since 2 years. He has also on and off bilateral nasal obstruction. Setting: OPD. Anterior rhinoscopy: Bilateral pale colored inferior turbinate hypertrophy. Cold spatula test: Absence of fogging on tongue depresser from both nostrils. Q.1: What is the most possible diagnosis? Ans: Allergic rhinitis. Q.2: What is allergic salute? Ans: Repeated itching of the nose in allergic rhinitis might lead to lifting of the tip of the nose upwards with the palm, which looks like salute. Q.3: What are Darrier’s line and Dennie-Morgan line? Ans: Darrier’s line: Horizontal crease on the dorsum of the nose due to repeated allergic salute. Dennie-Morgan line: Transverse crease below the lower eyelid. Q.4: What are the investigations in allergic rhinitis? Ans: Blood count shows eosinophilia. Nasal smear show eosinophil. Nasal provocative test—allergen is sprayed into the nose and number of sneezing is noted. Patch test—allergen is placed on the skin and area of reaction is noted. Allergic skin test—prick test, scratch test or intradermal test. PRIST (Plasma reactive immune sorbent test)—it used for specific IgE estimation. RAST (radioallergo sorbent test)—it is used for specific IgE estimation. SET test (skin end point titration test)—it is the latest skin test for allergy and is more reliable. Q.5: What are the curative treatments of allergic rhinitis? Ans: Oral antihistamines, nasal decongestants, steroids (oral or intranasal), sodium chromoglycate. 108

Allergic Rhinitis

109

Q.6: What are the second generation antihistamines? Ans: Astemizole, terfenadine, cetirizine, levocetrizine, loratidine, ebastine and fexofenadine. Q.7: What are the skin tests used in the diagnosis of allergic rhinitis? Ans: Skin tests are performed by introducing the antigen into the skin and checking the site after 10–15 minutes for any immediate local allergic reactions namely erythema and edema. There are three types of skin tests. These are: • Prick test—subcutaneous injection of a small amount of antigen in the outer skin layer by lifting and pricking skin at 14° angles. • Scratch test—scratching the skin with a drop of allergen. • Intradermal test—a certain amount of antigen is injected into the dermis and reaction is noted. Positive skin tests do not always imply nasal allergy to the same allergen. They only give a guide to the degree of atopy. CASE 2: ALLERGIC RHINITIS WITH BRONCHIAL ASTHMA

Presenting complaints: A 25-year-old male presented with nasal itching, running nose, sneezing and nasal congestion from time to time. He has shortness of breath and early morning chest tightness associated with cough and thick sputum. Setting: OPD. Anterior rhinoscopy: Pale and edematous inferior and middle turbinates in both nostrils. Family/Personal/ Medical history: There was no family history of atopy. Patient was smoker and consumed alcohol occasionally. Medical history was negative for recent episodes of fever or infections. General examination: The patient was well oriented but appeared restless. No pallor, icterus, clubbing or cyanosis. Tachypnea (25 breath/minute). Chest examination showed bilateral expiratory wheezing. Pulmonary function test: Pattern of reversible airway obstruction. Predicted FVC was 91% and FEV1 was 76%. Q.1: What is the diagnosis of above case? Ans: Allergic rhinitis with bronchial asthma. Q.2: What is therapy in allergic rhinitis and bronchial asthma? Ans: Avoidance of allergens, pharmacologic therapy and allergen specific immunotherapy. Q.3: What is pharmacologic therapy in above case? Ans: Intranasal corticosteroids, oral montelukast and a second generation antihistamines. Intranasal corticosteroids constitute the mainstay of therapeutic management for mild to moderate cases of allergic rhinitis. Q.4: What is allergic rhinitis? Ans: Alergic rhinitis is an inflammatory disorder involving the upper respiratory airways, eyes and the nose. It produces troublesome symptoms such as rhinorrhea,

110

Manual of Clinical Cases in ENT and Head–Neck Surgery

sneezing and nasal congestion that negatively affect the daily performance, work productivity, cognitive function and psychosocial well-being. Q.5: What is Samter’s triad? Ans: It is a triad of nasal polyposis, bronchial asthma and aspirin intolerance. Q.6: How allergic rhinitis is classified? Ans: The ARIA (allergic rhinitis and its impact on asthma) panel in association with World Healt Organization recommended a revised classification of allergic rhinitis. This classification aims at better diagnosis, treatment and decrease in morbidity. The classification is: Intermittent

Persistent

Mild

Moderate to severe

Less than 4 days a week Or, less than 4 weeks

More than 4 days a week And more than 4 weeks

Normal sleep No impairment of daily activities Normal work and school No troublesome symptoms

Abnormal sleep Impaired daily activities Abnormal work and school Troublesome symptoms

CASE 3: VASOMOTOR RHINITIS

Presenting complaints: A 30-year-man complaints persistent nasal obstruction and running nose since 2 years. Setting: Outpatient department. History of present illness: He was apperantly alright 5 years back, to start with nasal obstruction. The nasal obstruction is more marked in night. The obstruction of nostrils alternates from side to side. He has also symptoms of sneezing and mucoid nasal discharge. History of past illness: No history of similar attacks 2 years back. No history of DM, TB, HTN or any infectious diseases. General examination: He is well oriented to time, place and person. Moderately body built and well nourished. No anemia, jaundice, cyanosis, clubbing. BP—126/mmHg, pulse—72/min. Respiratory rate—16/min. Anterior rhinoscopy: Bilateral inferior turbinates are hypertrophied and looks congested. Allergy tests: Negative. Q.1: What is the most possible diagnosis? Ans: Vasomotor rhinitis. Vasomotor rhinitis is non-allergic rhinitis and mimicks to the allergic rhinitis and patient presents with nasal obstruction, rhinorrhea and sneezing. It is also known as intrinsic rhinitis, idiopathic rhinitis, non-allergic noninfectious perennial rhinitis. Q.2: What is the cause of vasomotor rhinitis? Ans: It is due to autonomic dysfunction. Overactivity of parasympathetic system causes inferior turbinate hypertrophy and also causes excessive secretion from the nasal glands.

Allergic Rhinitis

111

Q.3: What are the medical treatments of VMR? Ans: Avoidance of provoking factors like dust, blasts of air, sudden change of temperature, humidity. Antihistamines, topical steroids are very useful. Tranquillizers may be advised in case of psychological disturbances. Q.4: What is vidian neurectomy? Ans: In VMR, excessive rhinorrhea, not corrected by medical treatment, can be cured by sectioning the parasympathetic secretomotor fibers (Vidian nerve) to nose. Q.5: What is NARES? Ans: When vasomotor rhinitis is associated with eosinophilia, it is known as NARES (non-allergic rhinitis with eosinophilia syndrome). Q.6: What are the differences between allergic rhinitis and vasomotor rhinitis? Ans: Allergic rhinitis

Vasomotor rhinitis

• It usually starts at young age

• It occurs in 4th or 5th decades of life

• Prior exposure to allergen would be present

• Exposure not necessary; even climate changes may trigger the phenomenon

• Itching, sneezing, conjunctival redness and rhinorrhea are present

• Sneezing and plain watery rhinorrhea alone are prominent

• Nasal smear shows eosinophils

• No

• Blood IgE levels increased

• No

• Skin prick tests often positive

• No

• Vidian neurectomy is not helpful

• Vidian neurectomy is helpful

CASE 4: RHINITIS MEDICAMENTOSA

Presenting complaints: A known case of allergic rhinitis was advised topical nasal decongestant 6 months back for nasal obstruction. He has been getting relieved his symptoms and using this topical nasal drops since 6 months. However, he is now complaining nasal obstruction not relieved by this medication now. Anterior rhinoscopy: Bilateral inferior turbinate hypertrophy. Q.1: What is the most probable diagnosis? Ans: Rhinitis medicamentosa. Q.2: What is the cause for rhinitis medicamentosa? Ans: Rhinitis medicamentosa results from injudicious and continuous use of nasal decongestants. Basic defect is failure of vasoconstrictor effect of nasal decongestant on nasal mucosa. Treatment includes withdrawing nasal decongestant. Q.3: What are the causes of bilateral nasal obstruction? Ans: Rhinitis medicamentosa, bilateral choanal atresia, ethmoidal polyps, allergic rhinitis, vasomotor rhinitis, atrophic rhinitis, chronic sinusitis, large adenoid blocking both choana, large antrochoanal polyp blocking both choana, S-shaped DNS, septal hematoma and abscess.

112

Manual of Clinical Cases in ENT and Head–Neck Surgery

CASE 5: ALLRERGIC RHINITIS WITH NASAL POLYPI

Presenting complaints: A 22-year-old man presented with sneezing, nasal discharge with bilateral nasal block since 3 months. Anterior rhinoscopy: Mild DNS to left side with hypertrophied inferior turbinate and multiple polypoidal masses in both nostrils. Cold spatula test: No fogging in both nostrils and occasional mild fogging in right side. Bilateral nasal obstruction, more in left side. Q.1: What is the provisional diagnosis? Ans: Bilateral nasal polypi with allergic rhinitis and DNS to left. Q.2: What are the investigations required in this case? Ans: • Diagnostic nasal endoscopy • X-ray PNS • CT PNS • Allergic tests • Culture and sensitivity of nasal discharge Q.3: Could there be fungal sinusitis? Ans: • Yes, there could be associated chronic fungal sinusitis which is confirmed by direct inspection while removing the disease during FESS and then stain the discharge with KOH mount and then culturing the material in Sabouraud-Glucose media. • CT scan preoperative is helpful to find out invasive or noninvasive form of fungal sinusitis usually caused by Aspergillus species. CT scan shows double density in fungal sinusitis. Q.4: What will be the management of this case? Ans: The best management would be FESS using microdebrider with or without septoplasty if DNS causing hinderance to passing of rigid endoscope.

113

Sinusitis

17

Sinusitis CASE 1: ALLERGIC FUNGAL SINUSITIS

Presenting complaints: Doctor, I have nasal obstruction since 6 months. Setting: Out patient department of otorhinolaryngology History of presenting illness: A 45-year-old man complaining nasal obstruction, repeated nasal discharge, intermittent facial pain and pressure over cheeks since 6 months. History of past illness: He does take medication for high blood pressure. He had undergone sinus surgery 5 years back. General physical examination: The patient is well oriented with time, place and person. Pulse rate—90/minute, BP—110/80 mmHg. He has fever of 101°F. Anterior rhinoscopy: The both side nasal cavities are filled with pale smooth masses. The masses are pale color with smooth surface not pulsatile, not sensitive to touch and does not bleed. Examination of oral cavity and oropharynx: Right tonsil is congested and swollen. Uvula is pushed to the left side. Posterior rhinoscopy: Choana is free. Examination of the ear and neck: Normal. No swelling in the neck Q.1: What is allergic fungal sinusitis (AFS)? Ans: AFS is a reaction, in part allergic, to aerosolized fungal spores, usually for the dematiaceous species. It is noninvasive and is present in immunocompetent hosts. It presents with sinonasal polyposis and mucin. The mucin contains eosinophils, Charcot Leyden crystals and fungal hyphae. Q.2: What is the CT picture in AFS? Ans: CT scan shows mucosal thickening with hyperdense areas. There may be expansion of the sinus or bone erosion due to pressure but no fungal invasion. Q.3: What is the treatment of AFS? Ans: Treatment is endoscopic clearance of sinuses with provision of drainage and ventilation. This is combined with pre- and postoperative systemic steroids. Q.4: What is fungal ball? Ans: It is the implantation of fungus in a healthy sinus where CT scan shows a hyperdense area with no evidence of bone erosion or expansion. Most commonly 113

114

Manual of Clinical Cases in ENT and Head–Neck Surgery

Aspergillus causes fungal ball. Maxillary sinus is the most commonly affected followed by sphenoid, ethmoid and frontal in order. Treatment is surgical removal of the fungal ball with adequate drainage of the sinus. No antifungal treatment is needed. CASE 2: SINUSITIS AND COMPLICATION

Presenting complaints: A 23-year-old female presenting with a long standing history of sinusitis complaining fever, personality changes and headache since 5 days. Anterior rhinoscopy: Mucopurulent discharge in bilateral nasal cavity. Fundus examination: Papilledema. Q.1: Most likely diagnosis is ____? Ans: Frontal lobe abscess. Typical features of frontal lobe abscess are triad of fever, headache and personality changes. Papilledema signifies raised intracranial tension. Q.2: What is Pott’s puffy tumor? Ans: Frontal bone osteomyelitis presents with doughy forehead swelling. Q.3: What are the intracranial complications of sinusitis? Ans: Meningitis, extradural abscess, subdural abscess, brain abscess, cavernous sinus thrombosis and encephalitis. Most common intracranial complication is meningitis. Commonest site of brain abscess is frontal lobe. Q.4: What are investigations done in above case? Ans: Diagnostic nasal endoscopy, CT scan of PNS and MRI brain. Q.5: What are the characteristic headaches in sinusitis? Ans: Headache showing diurnal variation is due to frontal sinusitis. Headache due to frontal sinusitis is more toward the morning (office headache). Headache due to maxillary sinusitis is more toward evening. Vertical headache with postnasal discharge is suggestive of sphenoid sinusitis. Ethmoidal sinusitis presents mainly as orbital symptoms. Q.6: What are the symptoms and signs in different sinusitis? Ans: Sinus affected Maxillary sinusitis

Symptoms Persistent cold, purulent nasal discharge, cheek pain, headache, toothache. Headache more in the afternoon

Signs Tenderness over cheek, pus in middle meatus, swelling of lower eylid, dental infection

Frontal sinusitis

Periodic frontal headache, more in the morning or office hour (office headache)

Tenderness of the floor of the frontal sinus, pus in the anterior part of middle meatus which reappears quickly after cleaning and swelling of upper eyelid

Ethmoidal sinusitis Retro-orbital pain, purulent nasal discharge

Pus in the middle meatus, associated polyps in nasal cavity and periorbital cellulitis as complication

Sinusitis

115

CASE 3: SINUSITIS AND CAVERNOUS SINUS THROMBOSIS

Presenting complaints: A 32-year-old man with sinus infection presents with chemosis of conjunctiva, bilateral proptosis and fever. Q.1: What is the possible diagnosis? Ans: Cavernous sinus thrombosis. Q.2: Which sinus infections may lead to cavernous sinus thrombosis? Ans: Cavernous sinus thrombosis usually result from infection of ethmoid and sphenoid sinuses. Q.3: What are the clinical manifestations of cavernous sinus thrombosis? Ans: • Onset is abrupt with chills and rigors • Involvement of IIIrd, IVth, Vth and VIth cranial nerves • Chemosis of conjunctiva • Proptosis of the eye with limited movements • Papilledema • Pupils are dilated and fixed • Decreased vision • Decreased sensation in the distribution of the Vth nerve (ophthalmic division) Q.4: What is the treatment of cavernous sinus thrombosis? Ans: High dose of IV antibiotics and attention to the focus of infection, drainage of the ethmoid or sphenoid sinus. Blood culture should be taken before starting antibiotic therapy. Role of anticoagulants is not clear. CASE 4: SINUSITIS IN PEDIATRIC PATIENT

Presenting complaints: An 8-year-old child presents with history of chronic sinusitis and multiple nasal polyi. He also suffers from recurrent chest infections and malabsorption syndrome. Q.1: Which investigation is useful for diagnosis of this disease? Ans: Sweat chloride. Q.2: What is the diagnosis of this disease? Ans: Presence of chronic sinusitis, multiple nasal polypi and malabsorption points towards cystic fibrosis and, therefore, sweat chloride test would be useful. CASE 5: ACUTE SINUSITIS

Presenting complaints: A 12-year-old boy presenting with facial pain, fever and malaise since 5 days. He was suffering from upper respiratory tract in fections since one week. Anterior rhinoscopy: Purulent discharge in both sides middle meatus. X-ray PNS: Bilateral maxillary sinusitis.

116

Manual of Clinical Cases in ENT and Head–Neck Surgery

Q.1: What is the most possible diagnosis? Ans: Acute sinusitis. Q.2: What are the signs and symptoms of acute sinusitis? Ans: Symptoms

Signs

Fever

Flushed eyelids

Malaise

Swollen eyelids

Headache

Tenderness

Nasal discharge

Pus in the nasal cavity

Anosmia/cacosmia Epistaxis Flat voice Pain Unpleasant taste

Q.3: What are the causes of facial pain? Ans: • Sinusitis • Migrainous headache • Anterior ethmoidal neuralgia • Facial migraine • Trigeminal neuralgia • Mandibular malpositioning • Nasopharyngeal malignancy • Psychological • Brainstem lesions • Multiple sclerosis Q.4: What are the treatments of acute sinusitis? Ans: Medical treatment

Surgical treatment

Bedrest

Drainage of the sinuses

Decongestant nasal drops

Endoscopic sinus surgery

Steam inhalation Analgesics Local fomentation Antibiotics (amoxycillin or amoxycillin clavulanate)

117

Nasal Polypi

18

Nasal Polypi CASE 1: ETHMOIDAL POLYPOSIS

Presenting complaints: Doctor, I have nasal obstruction since 6 months. Setting: Out patient department of otorhinolaryngology History of presenting illness: A 35-year-old man complaining bilateral nasal obstruction since 6 months. History of past illness: He does take medication for high blood pressure. He had undergone sinus surgery 5 years back. General physical examination: The patient is well oriented with time, place and person. Pulse rate—90/minute, BP—110/80 mm Hg. He has fever of 101°F. Anterior rhinoscopy: The both sides nasal cavity are filled with pale smooth masses. The masses are pale color with smooth surface not pulsatile, not sensitive to touch and does not bleed. Examination revealed pale grape like masses in both nasal cavities (Fig. 18.1).

(a)

(b)

Fig. 18.1a and b: Ethmoidal polyps

Examination of oral cavity and oropharynx: Right tonsil is congested and swollen. Uvula is pushed to the left side. 117

118

Manual of Clinical Cases in ENT and Head–Neck Surgery

Posterior rhinoscopy: Choana is free. Examination of the neck: Normal. No swelling in the neck Examiantion of the ear, nose, paranasal sinuses and nasopharynx: Within normal limit. Q.1: What is the commonest site of origin of polyps in the nose? Ans: Commonest site is osteomeatal complex. Polyps arise from the upper part of nose around the ethmoids. It can also arise from the sinuses and middle turbinate. Q.2: What is the possible diagnosis in above case? Ans: Ethmoidal polyposis. Ethmoidal polyposis arises from the multiple air cells of the ethmoidal labyrinth. They present as multiple, bilateral and grape like masses. Q.3: What are the characteristic features of ethmoidal polyposis in anterior rhinoscopy? Ans: Small multiple grayish whitish mass (grapes like masses), soft in consistency, do not bleed on touch, insensitive to touch. Q.4: What are the investigations in the nasal polyposis? Ans: X-ray paranasal sinuses, diagnostic nasal endoscopy and CT scan of osteomeatal complex. Q.5: What are the factors contributing for obstruction of the osteomeatal complex? Ans: Anatomical anomalies 

Deviated nasal septum

Mass lesions 

Nasal polyp, foreign body and neoplasms

Middle turbinate abnormality 

Concha bullosa and paradoxical middle turbinate

Abnormality in lateral nasal wall 

Enlarged uncinate process

Mucosal edema 

Allergy, infections, vasomotor rhinitis, rhinitis medicamentosa

Q.6: Why ethmoidal polyps are usually bilateral? Ans: The ethmoidal polyps are usually bilateral because • Ethmoidal polyps are usually associated with nasal allergy, which is a part of systemic disease and it affects both sides of nasal cavity. • Ethmoid bone is a single midline bone. So the disease often affects both side of the same bone. Q.7: What are the differential diagnoses of ethmoidal polyps? Ans: The differential diagnoses are: • Hypertrophic inferior turbinates • Concha bullosa • DNS • Septal abscess, septal hematoma • Rhinosporidiosis • Angiofibroma

Nasal Polypi

119

• Meningocele/meningoencephalocele • Neoplasms of the nasal cavity • Old foreign body in nasal cavity/rhinolith. Q.8: Why antrochoanal polyp is usually unilateral? Ans: The causes are: • Antrochoanal polyp is not usually associated with nasal allergy. • Maxillary bones on either side are separate bones. Q.9: What are etiologies for ethmoidal polypi? Ans: • Chronic rhinosinusitis. • Bronchial asthma. • Aspirin intolerance. Samter’s triad—nasal polypi, asthma and aspirin intolerance. • Cystic fibrosis. • Allergic fungal sinusitis. • Kartagener’s syndrome—bronchiectasis, sinusitis, situs inversus and ciliary dyskinesis. • Young’s syndrome—sinopulmonary disease and azoospermia. • Churg-Strauss syndrome (asthma, fever, eosinophilia, vasculitis and granuloma). • Nasal mastocytosis. CASE 2: ANTROCHOANAL POLYP

Presenting complaints: A 6-year-old boy presents with right nasal obstruction since 3 months. He has also nasal discharge. Anterior rhinoscopy: Nasal discharge filling the right nostril. Posterior rhinoscopy: Not done as child was not co-operated. Diagnostic nasal endoscopy: A pale polypoidal mass in the posterior part of right nasal cavity. Oral cavity and oropharynx examination: A polypoid mass hanging from nasopharynx (Fig. 18.2).

Fig. 18.2: AC polyp seen through oral cavity

120

Manual of Clinical Cases in ENT and Head–Neck Surgery

Q.1: What is the most possible diagnosis? Ans: Antrochoanal polyp. Q.2: What is Killian’s polyp? Ans: It is synonym of antrochoanla polyp. It is usually unilateral and arises from the maxillary sinus and grows posteriorly to the choana. Q.3: Why does AC polyp go posteriorly? Ans: • The accessory maxillary sinus ostium is posterior. • The cilia beat posteriorly. • The slope of the floor of nose is towards the choana. Q.4: How do you differentiate a polyp from a turbinate? Ans: • Polyps are insensitive/turbinates are sensitive. • Polyps are soft/turbinate is hard. • Polyps do not shrink with decongestants. • Enlarged turbinates shrink with decongestants. • Probe can be passed all around a polyp, whereas turbinate has a lateral attachment. Q.5: Crescent sign is seen in _____? A. Juvenile nasopharyngeal angiofibroma B. Ethmoidal polyp C. Antrochoanal polyp D. Hemangioma of nasal cavity Ans: (C) Antrochoanal polyp Explanation: Crescent sign refers to crescent of air between the mass and posterior pharyngeal wall. It is present in antrochoanal polyp. Q.6: What is the treatment of choice in AC polyp? Ans: Functional endoscopic sinus surgery. Q.7: What are the differences between antrochoanal polyp and ethmoidal polypi? Ans: Age Etiology Number Laterality Origin Growth Size and shape Recurrence Treatment

AC polyp

Ethmoidal polypi

Common in children Infection Single Unilateral Maxillary sinus Grows backward to choana Trilobed Uncommon FESS

Common in adults Allergy or multifactorial Multiple Bilateral Ethmoidal sinuses Grows anteriorly Grape like masses Common FESS

Nasal Polypi

121

CASE 3: FUNCTIONAL ENDOSCOPIC SINUS SURGERY

Presenting complaints: A 25-year-old lady presents with sudden swelling of right eye after endoscopic sinus surgery done for chronic sinusitis. Q.1: What are the complications of FESS? Ans: Periorbital ecchymosis, orbital hematoma and optic nerve injury. Intracranial complications are CSF leak, meningitis, direct injury to brain, carotid artery injury. Other complications are injury to nasolacrimal duct, anosmia and synechia. Q.2: What is the importance of ethmoidal artery bleeding during FESS? Ans: Anterior ethmoidal artery is a branch of ophthalmic artery and is frequently encountered during endoscopic sinus surgery inside a canal between the roof of ethmoids and lamina papyracea of the orbit. The bony canal may be thinned out or dehiscent; inadvertent breach during surgery leads to retraction of vessels into the orbit to form a retro-orbital hematoma, which if sufficiently large can compress the orbit nerve and cause visual loss. Q.3: What are the major complications of endoscopic sinus surgery? Ans: Orbital hemorrhage, loss of vision, diplopia, CSF leak, meningitis, brain abscess, anosmia, massive hemorrhage, intracranial hemorrhage, injury to internal carotid artery, injury to nasolacrimal duct and death. Q.4: What are the minor complications of endoscopic sinus surgery? Ans: Periorbital ecchymosis, periorbital emphysema, epistaxis, postoperative infections, synechia, stenosis of maxillary or frontal sinus opening, hyposmia and dental pain. Q.5: What are the contraindications of endoscopic sinus surgery? Ans: Disease inaccessible by endoscopic procedures like lateral frontal sinus d s e a s e and stenosis of frontal sinus ostium, osteomyelitis, threatened intracranial or intraorbital complications, inexperience and lack of proper instrumentations. CASE 4: MENINGOENCEPHALOCELE

Presenting complaints: A 2-year-old child presenting with right side nasal mass which show more bulging on coughing or crying. The mass is reducible in nature. This mass is found since birth. Examination: The surface of the mass is smooth and pale in color An impulse can be noted in coughing, crying and straining. Q.1: What is the most possible diagnosis? Ans: Meningoencephalocele. Q.2: What is meningoencephalocele? Ans: Meningoencephalocele or encephalocele is herniation of brain tissue along with its meninges through a congenital bony defect. An extranasal meningoencephalocele

122

Manual of Clinical Cases in ENT and Head–Neck Surgery

presents as a subcutaneous pulsatile swelling in the midline at the root of nose (nasofrontal variety), side of nose (nasoethmoid variety) or on the anteromedial aspect of the orbit (naso-orbital variety). Q.3: What are the clinical characteristics of meningoencephalocele? Ans: Nasal mass/swelling show cough impulse and may be reducible. Patient presents with a pedunculated smooth mass at the roof of the nose. An impulse can be noted on increasing intracranial pressure by coughing, crying, straining, etc. Q.4: How the meningoencephalocele is managed? Ans: Diagnosis is confirmed by CT scan. Treatment warrants closure of the bony defect, in consultation with or preferably in the hands of a neurosurgeon. Q.5: What is Glioma? Ans: It is anipped off part of the encephalocele during embryonic development. There is no definite communication with cranial cavity exists, except a stalk like connection to the meninges. It does not contain any brain tissue. Clinically, impulse on coughing is negative. Majority of them are extranasal and present as firm subcutaneous swelling on the bridge, side of nose or near the inner canthus. Some of them are purely intranasal and very few cases are both intra- and extranasal. Treatment is surgical excision. Extranasal gliomas are encapsulated and can be easily removed by external nasal approach. CASE 5: MENINGOENCEPHALOCELE

Presenting complaints: A 3-year-old child presents with a unilateral single polyp. It does not bleed. Q.1: What will be your line of treatment? Ans: Simple nasal polyp below 5 years of age are rare. Meningocele, meningoencephalocele or a glioma can present as unilateral polyp an biopsy would risk meningitis. Such a child requires CT scan/MRI to establish the intracranial commmuincation and later a neurosurgical approach for removal. CASE 6: FUNCTIONAL ENDOSCOPIC SINUS SURGERY

Presenting complaints: A 35-year-old man with bilateral ethmoidal nasal polyposis attended the ENT outpatient department. He was already taking some medications but failed to relieve nasal block. Q.1: What is the surgical treatment of choice? Ans: Functional endoscopic sinus surgery (FESS). Q.2: What is the principle of the FESS? Ans: It improves the mucociliary clearance pathways of the osteomeatal unit. The obstruction to the drainage of the ostia is removed in the osteomeal complex. The ostia of the sinuses are made functional in the FESS.

Nasal Polypi

123

Q.3: What are the advantages of the FESS? Ans: • Good illumination and visualization of the areas • Absence of the external scar • Minimum hospitalization • Good for documentation and teaching purposes • Postoperative morbidity is minimal Q.4: What are the contraindications for FESS Ans: • Acute sinus infections associated with intracranial complications or orbital cellulitis with visual defects as well as osteomyelitis. • Infiltrative lesions. • A markedly stenosed frontonasal duct. • Aggressive invasive fungal infections of the paranasal sinuses such as mucormycosis will need an external approach. • Advanced malignancy. Q.5: What is mini FESS? Ans: It is a procedure in which only uncinectomy needs to be done because of the disease is limited. Q.6: What is balloon sinuplasty? Ans: It is the procedure where the natural ostium of the paranasal sinuses are dilated by using a balloon catheter and a sinus guide cannula and guide wire for entry into the particular sinus. It is a safe and less time consuming with minimal morbidity. Q.7: What is navigational FESS? Ans: It is also called image guided FESS. In this, preoperative loading of CT scan is done which can be visualized while doing surgery. This helps to do surgery in difficult areas such as frontal or sphenoidal areas or in revision cases.

124

Manual of Clinical Cases in ENT and Head–Neck Surgery

19

Granulomatous Diseases of the Nose CASE 1: RHINOSPORIDIOSIS

Presenting complaints: A 20-year-old boy presented with trickling of blood from the nose since 10 days. Setting: Out patient department History of present illness: The patient presenting with bleeding from left nostril since 10 days. It has occurred several times. In each episode, the bleeding consists of frank blood and was mostly unprovoked. He has also nasal blockage, which does not respond to nasal decongestant drops. No history of cold, sneezing, wheezing. No loss of smell and no headache. History of past illness: No similar attack of excessive bleeding in the past. No history of diabetes, hypertension, tuberculosis, drug allergy or food allergy. No history of any ENT surgery in the past. General physical examination: Weight—60 kg, height—170 cm, pulse—70/min, respiratory rate—20/min, temp—afebrile, blood pressure—12/80 mm Hg. Anterior rhinoscopy: A pinkish to reddish fleshy mass occupying almost the entire left nasal cavity (Fig. 19.1). The surface resembles a strawberry, with granular surface and whitish spots. It is not sensitive to touch. On touching the mass, bleeding occurs. On brobing, mass is attached to the lateral wall. The right side is free of any mass.

Fig. 19.1: Pinkish to reddish mass inside the left nasal cavity 124

Granulomatous Diseases of the Nose

125

Posterior rhinoscopy: The left choana is found to be filled with the same pinkish mass. The right choana, the lateral and posterior walls of the nasopharynx are free. Ears, oral cavity, throat, neck: Normal Q.1: What is the provisional diagnosis? Ans: Rhinosporidiosis Q.2: What is the etiology of rhinosporidiosis? Ans: It is caused by rhinosporidium seeberi, which is thought to be fungus. Now it is proved to be an aquate protistan parasite. It is included in the class Mesomycetozoea. This class consists of several fish parasites, and is also called DRIP clade, or Ichthyosporea. Because the organisms could not be cultured easily in any media, this controversy could not be settled. Q.3: Why is it common in people who take bath in pond? Ans: Common in people where animals and men take bath together. Water gets contaminated by spores through animals. Q.4: What are the types of rhinosporidiosis? Ans: (A) Benign: Nasal, nasopharyngeal, mixed, bizarre (conjunctival/tarsal); (B) malignant (generalized rhinosporidiosis). Q.5: What is the epidemiology of rhinosporidiosis? Ans: About 88% of the cases in the world are from India and Sri Lanka. Diseases are unknown in Jammu Kashmir, Punjab and Haryana in India. People with blood group ‘O’ are more susceptible. Q.6: How do you diagnose rhinosporidiosis? Ans: The diagnosis is based on typical clinical features and on gross appearances. Histopathological examination of the lesion shows a fibrous stroma with a lot of chronic inflammatory cells, namely macrophages and lymphocytes. A large number of sporangia and spores are seen in the mass. Q.7: What is the treatment of rhinosporidiosis? Ans: Treatment is complete excision of the mass with cauterization of the base. Recurrence may occur after surgical excision. Dapsone has been tried in some cases. Q.8: What is malignant rhinosporidiosis? Ans: When rhinosporidiosis spreads (systemic spread) by blood and deposit in different parts of the body like skin or bone, then it is called malignant rhinosporidiosis. CASE 2: RHINOSCLEROMA

Presenting complaints: A 35-year-old lady presented with bilateral nasal obstruction since 6 months. Anterior rhinoscopy: It shows granulomatous bluish red rubbery nodules in both nasal cavity. Ala nasi is stiff on palpation. The lesion originates at the mucocutaneous junction in the nasal vestibule.

126

Manual of Clinical Cases in ENT and Head–Neck Surgery

Q.1: What is the most possible diagnosis? Ans: Rhinoscleroma. Rhinoscleroma is a chronic progressive granulomatous disease of nasal mucosa extending into the nasopharynx, oropharynx and even in the larynx, trachea and bronchi. It is caused by bacillus Klebsilla rhinoscleromatis (Frisch bacilli). Because of cicatrization, nose attains a hard feel, described as woody nose/Hebra nose. It mostly occurs in poor and low socioeconomic people. In India, it is common in north than the south and often seen in MP, Mumbai, Delhi, Punjab and UP. Q.2: What are the clinical manifestations in rhinoscleroma? Ans: Clinical features varies in different stages of the rhinoscleroma. Atrophic stage: Congestion and infiltration with symptoms and signs of nasal catarrh. Subsequent changes occur in the nasal cavity mucosa resembling atrophic rhinitis but nose is not roomy and no anosmia. Nasal vestibule is some what stuffy. Crusting and fetor are present. Nodular stage: Bluish-red nodules appear at mucocutaneous junction which is rubbery in consistency. Later on these nodules become pale and hard. A cartilaginous feel of the nose is typically woody. Stenotic stage (cicatrization or fibrotic stage): There is fibrosis, adhesion formation, gradually nasal cavity becomes narrow, stenosed with distortion of the anatomy. Q.3: How the diagnosis of the rhinoscleroma is done? Ans: Biopsy (histopathological examination), culture of the bacilli from the biopsy material and antibodies estimation. Q.4: What is the characteristic histopathological feature in rhinoscleroma? Ans: Characteristic histological features include foam cells with vaculated cytoplasm and eccentric nucleus (Mikulicz cell) and transformed plasma cells (Russell bodies). Q.5: What are the treatment options for rhinoscleroma? Ans: A long term treatment with antibiotics (3 months) are required because—frisch bacilli have thick mucopolysaccharide wall, intracellular situation of bacillus, fibrosis around the lesion does not allow blood and drug to reach the lesion. Systemic antibiotics like ciprofloxacin, rifampicin, tetracycline, co-trimoxazole, streptomycin are given till 2 cultures negative from biopsy sample. Applications of 2% acriflavine dye or rifampicin locally and local irradiation have been tried. Q.6: What are Hebra nose deformity and Tapir nose deformity? Ans: In granulomatous or nodular stage of rhinoscleroma, rubbery nodules occur in the nose. There is subdermal infiltration of the lower part of the external nose and upper lip giving a woody feel, called Tapir nose. In cicatrical stage, scarring, enlargement and disfigurement of the nose occurs. This is called Hebra nose. CASE 3: RHINITIS SICCA

Presenting complaints: A 30-year-old man, gold smith in profession presenting with nasal obstruction and occasional nasal bleeding since one month.

Granulomatous Diseases of the Nose

127

Setting: Out patient department. Anterior rhinoscopy: Crusts in anterior part of nasal cavity. Posterior rhinoscopy: Normal. Examination of ear and throat: Normal. Q.1: What is the most possible diagnosis? Ans: Rhinitis sicca. Q.2: What is rhinitis sicca? Ans: It is a crust forming disease of the nasal cavity and seen among patients those work in hot, dry and dusty environment, e.g. bakers, iron and goldsmiths. This condition is confined to the anterior third of nasal cavity particularly nasal septum. The ciliated columnar epithelium of the nasal cavity undergoes squamous metaplasia with atrophy of seromucinous glands. Crusts are seen anterior part of septum and their removal causes ulceration and epistaxis and may cause septal perforation. Q.3: What is the treatment of rhinitis sicca? Ans: Avoid dry and dust surroundings. Application of bland ointment or one with antibiotic and steroid to the affected part. Nor pricking and forcible of crusts from nasal cavity should be avoided. Nasal douching or normal saline spray are useful. Q.4: What is rhinitis caseosa? Ans: It is also known as nasal cholesteatoma. This is a rare clinical condition where cheesy material drains into the nasal cavity from the maxillary sinus (usually). It probably results from failure of resolution of sinusitis, when the contents become inspissated and lining mucosa granulomatous. The medial wall of the maxillary sinus is often eroded. Often there is polypoid changes in the nasal mucosa. The treatment is removal of the cheesy material and granulation tissue from antrum by sublabial approach with drainage of sinus by antrostomy. The polypoidal mass of the nasal cavity is removed and all inspissated cheesy masses are sucked out. CASE 4: SINONASAL MUCORMYCOSIS

Presenting complaints: A 62-year-old diabetic male presents with blackish foul smelling discharge from nose since 3 months. Anterior rhinoscopy: Black necrotic mass was seen in the region of inferior turbinate. Q.1: Most possible diagnosis is___? Ans: Mucormycosis. Q.2: What is mucormycosis? Ans: Mucormycosis is common among elderly, uncontrolled diabetic and immunocompromised states. It is a fulminant infection (due to angioinvasion) and spreads very rapidly to orbit and intracranial cavity. Clinical examination shows a necrotic black mass inside the nasal cavity. Q.3: What is the treatment of mucormycosis? Ans: This patient need immediate hospitalization, debridement of mucormycotic tissues, antifungal (amphotericin B) and control of diabetes.

128

Manual of Clinical Cases in ENT and Head–Neck Surgery

CASE 5: SINONASAL MUCORMYCOSIS WITH ORBITAL INVOLVEMENT

Presenting complaints: A 22-year-old male patient with diagnosis of diabetes attended the ENT OPD with fever, swelling over right upper eyelid and right cheek since 7 days. He had occasional purulent nasal discharge and suuden loss of vision since 2 days. Anterior rhinoscopy: Grayish white, slough covered middle turbinate with mucopurulent secretion in the right nasal cavity whereas left nasal cavity patent. Examination of eye: Proptosis of the right eye inferolaterally, upper eyelid edema, drooping of upper eyelid, decreased movement of the right eye in upwards and inwards direction. Fundus examination of the right eye revealed hyperemic disc, full cup, dilated veins, two cotton wool patches and macular edema. CT scan: CT scan of orbit and PNS showed irregular infiltrating soft tissue density mass in the superomedial aspect of the right orbit with inferolateral displacement of the right globe suggestive of orbital pseudotumor. KOH stain of the nasal secretion: Broad ribbon like hyphae without septation. Q.1: What is the diagnosis? Ans: Sinonasal orbital mucormycosis. Q.2: What is the causative organism of the mucormycosis? Ans: The causative organisms are members of the family mucoracea. Rhizopus oryzae is the predominant pathogen and accounts for 90% of rhinocerebral case of mucormycosis. Q.3: What is the treatment of sinonasal orbital mucormycosis? Ans: Treat the diabetics. Immediate endoscopic debridement of the fungal mass from nose and sinus. Start intravenous administration of amphotericin-B in dose of 1– 1.5 mg/kg/day. CASE 6: WEGENER’S GRANULOMATOSIS

Presenting complaints: A 55-year-old male presents with sinus pain and bloody nasal discharge since one year. Anterior rhinoscopy: It shows a septal perforation with crusts in nose. Urine analysis: It showed proteinuria, hematuria and red cell casts. Histopathological examination: A biopsy from the edge of septal perforation reveals granulomatous inflammation with necrosis. Blood tests: It showed the presence of c-ANCA. Q.1: What is the most possible diagnosis? Answer: Wegener’s granulomatosis. It is an autoimmune condition associated with systemic manifestations involving the nasal cavity and respiratory tract. It is associated with generalized vasculitis and a focal form of glomerular nephritis. Q.2: What are the Lab investigations in this case? Ans: The cytoplasmic antineutrophil cytoplasmic antibody (C-ANCA) has high sensitivity and specificity. Lab evaluation shows raised ESR, raised blood urea and serum creatinine. Urine is positive for red cell casts, albumin. Diagnosis is established by nasal biopsy and renal biopsy.

Granulomatous Diseases of the Nose

129

Q.3: What is the treatment of Wegener’s granulomatosis? Ans: Treatment includes steroids and cyclophosphamide. Q.4: What are the differences between Wegener’s granulomatosis and nonhealing midline granuloma? Ans: Table 19.1: Comparison between Wegener’s granulomatosis and non-healing midline granuloma Wegener’s granulomatosis

Non-healing midline granuloma

• Distribution of lesion

Focal and localised

Diffuse

• Onset

Gradual

Explosive

• Progress

Gradual

Rapid

• Ear, trachea and renal

Involvement common

Involvement very uncommon

• Histology

Vasculitis

Polymorphic lymphoid infiltrate

• C-ANCA

Diagnostic

Negative

• EBV-RNA

Absent

Detected

• Immuno-histochemistry

No role

Diagnostic

• Treatment

Immunosupression

Radiotherapy

CASE 7: ATROPHIC RHINITIS

Presenting complaints: A 28-year-old lady complaints with nasal obstruction, loss of smell perception and nasal discharge since one year. Setting: OPD Anterior rhinoscopy: Nasal cavity is roomy, filled with foul smelling discharge and greenish crusts (Fig. 19.2). The nasal mucosa is dry and atrophic. Turbinates are atrophic and insensitive to touch.

Fig. 19.2: Greenish crusts with atrophic changes inside the nasal cavity

Posterior rhinoscopy: Dry mucosa with few greenish crusts near the choana. Examination of the throat: Dry mucosa.

130

Manual of Clinical Cases in ENT and Head–Neck Surgery

Q.1: What is the most possible diagnosis? Ans: Atrophic rhinitis. Atrophic rhinitis is a chronic inflammatory disease of the nose, characterized by progressive atrophy of the nasal mucosa and underlying turbinate bones, resulting in a roomy nasal cavity and yellowish green crusting with foul smell. It is often called Ozoena. Q.2: What are the points in favor of your diagnosis? Ans: • Nasal obstruction • Loss of smell • Crusting • Recurrent nasal bleeding • Stench complained by relatives and friends • Nasal cavity is roomy due to atrophy of nasal turbinates • Nasal cavity is filled with greenish crust Q.3: What is Merciful anosmia? Ans: Merciful anosmia is a pathognomonic feature of Atrophic rhinitis. A foul smell emanates from the patient’s nose and make her/him a social outcaste. But the smell is not perceived by the patient herself/himself due to atrophy of olfactory mucosa. Q.4: What are the investigations advised in atrophic rhinitis? Ans: • Routine blood tests • VDRL • Mantoux test • Chest X-ray • X-ray of paranasal sinus • Proof puncture and lavage Q.5: What are the possible differential diagnoses of atrophic rhinitis? Ans: • Chronic sinusitis • Rhinitis sicca Q.6: What are etiology of primary atrophic rhinitis? Ans: • Heredity—runs in family. • Endocrine factor—it starts in puberty and mostly affects female. • Racial factor—white and yellow races are commonly affected. • Nutritional deficiency—vitamin A, D and iron deficiency. • Infections—Klebsiella ozaenae, diptheroids, proteus, E. coli, etc. • Autoimmune.

Granulomatous Diseases of the Nose

131

Q.7: What is the secondary atrophic rhinitis? Ans: Secondary atrophic rhinitis is secondary to trauma (major nasal surgery), irradiation, chronic nasal infection (tuberculosis, leprosy, scleroma, syphilis) and other granulomatous diseases (midline Stewart’s granuloma, sarcoidosis, fungal granuloma). Q.8: What are the complications of atrophic rhinitis? Ans: Atrophic pharyngitis, atrophic laryngitis, Maggots in nose, sinusitis, middle ear infections, depression due to foul smelling nose, meningitis and perforation of nasal septum. Q.9: What is the aim of the treatment in atrophic rhinitis? Ans: Reducing crust, reducing foul smell and reduce dryness of the nasal mucosa by making moist. Alkaline nasal douche helps to remove the crust from the nose, to reduce the foul smell and to control epistaxis. The alkaline nasal douche solution contains sodium chloride (for isotonic), sodium bicarbonate (for loosening the crust) and sodium bicarbonate (borax) is act as antiseptic. Q.10: What are medical treatment of atrophic rhinitis? Ans: • Nasal irrigation with alkaline solution. • 25% anhydrous glucose in glycerine nasal drop. • Antibiotics to control infection. • Estradiol and vitamin D3. • Streptomycin. • Oral potassium iodide. • Placental extract injection in submucous layer. Q.11: What is the of 25% glucose in gylcerine in atrophic rhinitis? Ans: 25% glucose in glycerine: Glycerine prevents drying of the nasal mucosa and glucose gets fermented into lactic acid, which prevents growth of the proteolytic organisms. Q.12: How the nasal douching is done in atrophic rhinitis? Ans: Nasal douching is done with specific alkaline solution. It contains 280 ml of water in which one part of soda bicarbonate, one part of sodium biborate and two parts of sodium chloride are added. This solution loosens and dislodges the crusts. Some thick crusts may have to be removed manually during endoscopic examination. Douching is preferably done 2 to 3 times a day. Q.13: What is Wittmack’s operation and Lautenslager’s operation? Ans: Wittmack’s operation: Transfer of stensen duct to the maxillary sinus for increasing the moisture in the nose. Lautenslager’s operation: Medialisation of the lateral wall of the nose. CASE 8:RHINOSCLEROMA

Presenting complaints: A 25-year-old female from Uttar Pradesh presents with nasal block and crusting of nose since 2 months.

132

Manual of Clinical Cases in ENT and Head–Neck Surgery

On examination: Revealed an infiltrating lesion involving the nasal vestibule and upper lip with broadening of nasal dorsum. Q.1: What is most likely diagnosis? Ans: Rhinoscleroma. Rhinoscleroma is common in northern states of India. It is caused by Klebsiella rhinoscleromatis—a capsulated gram-negative coccobacillus. Q.2: What are the common presentations by rhinoscleroma? Ans: In rhinoscleroma, the nose is the most commonly involved and the lesion begins at the mucocutaneous region of nasal vestibule and infiltrates the nasal vestibule, septum and upper lip resulting in broadening of nose and woody hard infiltartaion of lip.

133

Neoplasms of the nose and Nose

20

Neoplasms of the Nose and Sinus CASE 1: INVERTED PAPILLOMA

Presenting complaints: A 55-year-old man seen by his primary care physician over several months with complaints of difficulty breathing through his right nasal cavity for more than a year along with frequent nasal discharge and occasional bleeding. He was thoroughly treated for chronic sinusitis with antibiotics and nasal steroid sprays and had no resolution of his symptoms. Past history: He had undergone surgery for deviated nasal septum and right nasal polyp five years back but has not since been seen by an otolaryngologist. Personal history: The patient is a social drinker with no smoking history. Examination of the nose and paranasal sinuses: On examination he appears to have a decreased movement of air through his right nasal cavity. Anterior rhinoscopy is essentially clear but on nasal endoscopy a fair large polypoidal mass (Fig. 20.1) can be seen projecting from beneath the right middle turbinate. It appears polypoid and friable. The mass does not appear to be vascular. Endoscopic examination of the left nasal cavity is clear showin no evidence of polyps or osteomeatal disease.

Fig. 20.1: Polypoidal mass inside the nasal cavity

Q.1: What is the most possible diagnosis? Ans: Inverted papilloma. Inverted papilloma is the most common benign epithelial tumor of the nose and paranasal sinuses. Incidence is in the range of 0.2 to 0.6% per 133

134

Manual of Clinical Cases in ENT and Head–Neck Surgery

100,000 people per year, making up 0.5 to 4% of primary nasal tumors. The male to female ratio is 3:1, and predominance in white patients is suggested in the literature. The most common sites of origin include the lateral nasal wall at the root of the middle turbinate and the maxillary sinus. Q.2: What is the importance of Inverted papilloma? Ans: It is a premalignant lesion. There is 2–5% chance of developing squamous cell carcinoma. Malignancy may co-exist in 5–8% cases. There is high possibility of recurrence after excision as it may be multicentric. Q.3: What are the other names of inverted papilloma? Ans: Ringertz tumor or transitional cell papilloma or Schneiderian papilloma. Q.4: What is the etiology for inverted papilloma? Ans: There is no widely accepted cause, but there may be an association with HPV. Various types, including 6, 11, 16 and 18 have been found in these lesions, but no clear causative factor has been found. Q.5: Where is the origin of inverted papilloma inside nasal cavity? Ans: Inverted papilloma is a lesion arises from the Schneiderian membrane. This is the transition between the endoderm-derived respiratory epithelium and the ectodermally derived squamous epithelium. Other lesions can arise from this membrane, including fungiform papillomas, which are often found on the nasal septum and are human papilloma virus (HPV) related, and cylindrical papillomas, which appear different but act similar to inverted papillomas. Debate exists as to whether these are different entities or expressions of the same process in different locations. Q.6: What is the histopathological characteristic of inverted papilloma? Ans: Inverted papilloma histopathologically have an epithelium that inverts into the underlying connective tissue stroma (Fig. 19.2). They often appear exophytic, polypoid, more vascular than an inflammatory polyp, and with a gray to pink appearance. No mucus cells, koilocytosis, or eosinophils are present. This is in contradistinction to inflammatory nasal polyps. Although invasion into surrounding structures such as occurs with malignancy is not present, pressure and mass effect can cause bone remodeling and destruction.

Fig. 20.2: Histopathological picture of inverted papilloma

Neoplasms of the nose and Nose

135

Q.7: What is the surgical treatment of inverted papilloma? Ans: Lateral rhinotomy with medial maxillectomy was the standard technique for removing the tumor completely, with recurrence rates less than 15%. Complications include epiphora, a misplaced medial canthus and a large facial scar. With advent of endoscopic aurgical techniques, many inverted papillomas are now removed by this method where recurrence rates have been comparable to traditional approaches. CASE 2: ESTHESIONEUROBLASTOMA

Presenting complaints: A 45-year-old male with a history of progressive nasal obstruction, headache and occasional nasal bleeding since 6 months. Anterior rhinoscopy: Irregular fleshy granular mass arising from roof of the nasal cavity. On probing, the mass bleeds. Head and neck examination: Normal. Vision: Normal. Biopsy under local anesthesia: Esthesioneuroblastoma. Q.1: What is esthesioneuroblastoma? Ans: Esthesioneuroblastoma (also called olfactory neuroblastoma) is a rare slow growing malignant neoplasm of neuroectodermal origin that begins in neuroepithelial cells of the olfactory membrane in the roof of the nasal fossa. Q.2: What is the radiological investigation for above tumor? Ans: High resolution CT scan and magnetic resonance imaging can be used to delineate the extent of the tumor and define the involvement of the cribriform plate, fovea ethmoidalis, anterior cranial fossa, lamina papyracea and retromaxillary space. Q.3: What is the Kadish classification for esthesioneuroblastoma? Ans: Stage A: Tumors confined to the nasal cavity; Stage B: Tumor involving the nasal cavity and extends to the paranasal sinuses; Stage C: Tumor extension beyond the nasal cavity and paranasal sinuses. Q.4: What is the treatment for esthesioneuroblastoma? Ans: Treatment recommendation ranges from a minimally invasive approach to craniofacial resection combined with readiotherapy. CASE 3: SINONASAL TUMORS

Presenting complaints: A 52-year-old male presents with swelling on the right side of the cheek region since one year. History of presenting illness: Hitory of progressive right side nasal obstruction and recurrent episodes of right nasal blood stained discharge. Anterior rhinoscopy: A red fleshy mass in the right nasal cavity arising from the right lateral nasal wall (Fig. 20.3). Posterior rhinoscopy: Normal. Oral cavity and oropharynx: Right side palatal bulging is present and others are normal.

136

Manual of Clinical Cases in ENT and Head–Neck Surgery

Fig. 20.3: Right nasal mass with palatal bulging

Q.1: What is the provisional diagnosis? Ans: Right maxillary tumor. Neoplasms of nose and paranasal sinuses are very rare and constitutes only 0.5% of head and neck malignancy. In nasal cavity 50% are benign and 50% are malignant. In paranasal sinuses malignant tumors are common. Common sites of neoplasma of paranasal sinuses are in order maxillary sinus (70%), ethmoidal (20%), sphenoid (3%) and frontal (1%). Q.2: What are the etiological factors for paranasal sinus malignancy? Ans: Nickel workers are prone for squamous cell carcinoma, wood workers are prone for adenocarcinoma, leather shoe workers are prone for adenocarcinoma, industrial fumes, leather tanning, mustard gas, radium paint, hydrocarbons and alcohol consumption. Q.3: What are the clinical profiles of maxillary carcinoma? Ans: Nasal obstruction, epistaxis, nasal discharge, trismus, pain, epiphora, diplopia, proptosis, paresthesia, swelling and neck node enlargement. Q.4: What are the investigations for maxillary carcinoma? Ans: Diagnostic nasal endoscoy and biopsy, FNAC from enlarged neck node, CT scan, MRI and PET scan. Q.5: What are the earliest suspicion clinical features of sinonasal cancer? Ans: Unexplained epistaxis, unexplained epiphora, dental neuralgia, frequent change of denture, flattening of dental arch and non-healing socket. Q.6: What is Ohngren’s classification? Ans: An imaginary oblique line extending from the medial canthus of the eye to the angle of mandible and a second line passing vertically through the pupil. Neoplasms posterior to these lines are dangerous and poor prognosis in comparision to anterior tumors. Posterior tumors have tendency to spread superiorly and posteriorly. Tumors below the line is easily resected. Q.7: What is Lederman’s classification? Ans: Two horizontal lines passing through floor of the orbit and floor of the nasal cavity divided into three parts like suprastructers, mesostructures and infrastructures.

Neoplasms of the nose and Nose

137

Two vertical lines passing through medial orbital margin again divided each region into three like middle, right and left. There are total nine segments formed. Q.8: What is the treatment of maxillary carcinoma? Ans: Surgery can be curative if complete resection is done. Postoperative radiotherapy is given for: Large tumor, positive margins, perineural or perivascular invasion and lymph node metastasis. Q.9: What are the contraindications for surgery in maxillary carcinoma? Ans: Distant metastasis, inoperable metasatic nodes, base of skull involvement, involvement of nasopharynx and spread of tumors beyond midline. CASE 4: DENTIGEROUS CYST

Presenting complaints: A 14-year-old boy presenting with slow growing painless mass in the right upper jaw since 6 months. Setting: OPD. On examination: Palpation giving feeling of egg-shell cracking. Tooth is missing at the site of swelling. Q.1: What is the provisional diagnosis? Ans: Dentigerous cyst. Dentigerous cyst is developmental odontogenic cyst arising from the enamel organ (epithelium) in the crown of a permanent developing tooth (after amelogenesis has been completed). The expanding cyst is coming from the follicle (consisting of an inner epithelial layer and oute connective tissue covering, so it is also called follicular cyst). The root remains unerupt and remained impacted in the cyst or its wall. It constitutes 95% of follicular cysts and 34% of the ontogenic cysts. Q.2: What are common tooth affected by dentigerous cyst? Ans: The third molar in the mandible and premolar (cuspid) in the maxilla. Q.3: What are the features of odontogenic cyst in imaging? Ans: A well circumscribed radiolucent area is seen with clearly defined margin and unerupted tooth in the cavity. Occasionally, there may be a peripheral scale of sclerosis surrounding the translucent area. At times pseudo-trabecular pattern (Soap bubble appearance) in translucent area is seen. Q.4: What are the complications of dentigerous cyst? Ans: Transformation into ameloblastoma and pain due to pressure in the root of adjacent tooth. Q.5: What is the treatment of dentigerous cyst? Ans: Enucleation along with the origin of the tooth. Q.6: What are other different odontogenic cysts? Ans: Radicular cyst: It occurs due to infection of the root canal following long chronic inflammation in periapical granuloma. Primordial cyst: It occurs as a cystic degeneration occurring prior to maturation of enamel organ.

138

Manual of Clinical Cases in ENT and Head–Neck Surgery

21

Headache CASE 1: CLUSTER HEADACHE

Patient complaints: A 30-year-old male presents with recurrent episodic severe pain in the right eye associated with swollen conjunctiva and redness in eye. Usually it subsides in about an hour or after taking analgesics. Most of the episodes are at night. Setting: OPD Examination: Ear, nose and throat are normal. Q.1: Most probable diagnosis is___? Ans: Cluster headache. Q.2: What are the other names of cluster headache? Ans: Horton’s headache, periodic migrainous neuralgia, erythrocephalgia, histamine cephalgia or suicide headache. Q.3: What are the classical clinical presentations of cluster headache? Ans: Unilateral headache often lasting for less than one hour and rarely more than four hours. It is commonly seen in young persons. Headache is characterized by excruciating pain behind or around the eye radiating to the temporal region, jaws, chin or teeth. Headache is associated with unilateral sweating, sclera congestion, rhinorrhea, unilateral nasal obstruction and facial flushing. Q.4: What are the treatment options for cluster headache? Ans: The cluster headache can be ended with inhalation of 100% oxygen for 10 minutes or administration of ergotamine or dihydroergotamine. Sumatriptan works best if taken subcutaneously. Calcium channel blockers like nifedipine or verapamil are useful for prophylactic medications. CASE 2: TENSION HEADACHE

Patient complaints: A 40-year-old office going executive presents with dull aching headache with a band like sensation over the forehead. Headache increases as the day progresses. Examination of nose and sinus: Normal. Examination of ear and throat: Normal. 138

Headache

139

Q.1: Most likely diagnosis is___? Ans: Tension headache. Q.2: What are the characteristics of tension headache? Ans: Headache with a band like sensation and increase in severity as the day progress is characteristic of tension headache. Tension headache usually lasts 30 minute to 70 days. Q.3: What is the cause for tension headache? Ans: Stress and emotional disturbances are the important causes for tension headache. Patients with tension headache have depression and anxiety. Q.4: What is the treatment for tension headache? Ans: Reassurance to the patient and change of life style to make him. Her tension free. Tablet amitriptyline is effective when given for a period of six weeks with a starting dose of 10 mg at bedtime. If found effective, the drug is continued for a period of six months. CASE 3: MIGRAINE

Patient complaints: A 25-year-old female presents with episodes of left sided throbbing headache more in premenstrual period and at times associated with nausea and vomiting. Patient says that sharp light and loud music aggravates the pain and taking rest in a dark room is relieving. Her mother also had a similar history of headache. Examinations of nose and sinus: Normal. Examinations of ear and throat: Normal. CT scan and brain: Normal. Q.1: What is the most probably diagnosis? Ans: Migraine. Q.2: What are the diagnostic criteria for migraine? Ans: Migraine without aura—(A) At least five attacks fulfilling B–D; (B) Headache has at least 2 of the following characteristics: (a) Unilateral, (b) pulsating quality, (c) moderate to severe intensity, (d) aggravation by or causing avoidance of routine physical activity. (C) During headache at least one of the following: (a) Nausea and/ or vomiting, (b) photophobia and phonophobia. (D) At least one of the following: (a) History and physical do not suggest headache secondary to organic or systemic metabolic disease, (b) history and/or physical and/or neurological examination do suggest such disorder, but is ruled out by appropriate investigations, (c) such disorder is present, but migraine attacks do not occur for the first time in close temporal relation of the disorder. Migraine with aura—(A) Atleast two attacks fulfilling B–D. (B) Aura consisting of at least one of the following, but no motor weakness: (a) Fully reversible visual symptoms including positive features (e.g. flicker lights, spots or lines) and/or negative features (loss of vision), (b) fully reversible sensory features including positive features (e.g. pins and needles) and/or negative features (numbness), (c) fully reversible dysphasic speech disturbance. (C) At least two of the following: (a) Homonymous visual symptom and/or unilateral sensory

140

Manual of Clinical Cases in ENT and Head–Neck Surgery

symptoms, (b) at least one aura symptom develops gradually over five minutes and/ or different aura symptoms occur in succession over five minutes. (D) Headache fulfilling criteria B–D for migraine without aura begins during the aura or follows aura within 60 minutes. (E) No attributed with another disorder. Q.3: What are the drugs used for prevention of migraine attacks? Drug

Dose available

Target doe

Topiramate Divalproex

ANTICONVULSANTS 25 mg 250 mg

100–200 mg daily Sedation, weight loss, kidney stone 1500–2000 mg daily Weight gain, polycystic ovary

ANTIDEPRESSANTS Amitryptyline 10 mg Pizotifen 0.5–1.5 mg

30 mg daily

Propanolol Timolol Metaprolol

BETA BLOCKERS 10 mg 2.5–10 mg 25 mg

Nifedipine Verapamil

CALCIUM CHANNEL BLOCKERS 30 mg 60 mg 40 mg 80–320 mg

Naproxen

NSAIDs 250–500 mg

Side effect

Sedation Weight gain and urinary retention

120 mg daily 20 mg daily 50–100 mg daily

1500 mg

CASE 4: TEMPORAL ARTERITIS

Patient complaints: A 27-year-old lady presents with right sided throbbing headache for lasy 2 years. On examination in temporal region bead like appearance is felt. Setting: OPD. ENT examinations: Normal. Lab investigations: It showed raised ESR. Q.1: Most probable diagnosis? Ans: Temporal arteritis. Q.2: What are the characteristic features in temporal arteritis? Ans: It is common in female and in age group over fifties. Temporal artery is tender on palpation. Visual loss is seen in 40% of cases due to involvement of ophthalmic artery. Q.3: What are the investigations done in temporal arteritis? Ans: ESR raised, C-reactive protein levels are elevated, plasma viscosity is raised and biopsy is diagnostic. Q.4: What is treatment for temporal arteritis? Ans: Predinsolone is the drug of choice and should be given in high daily dose of 60 mg per day.

Headache

141

CASE 5: EAGLE’S SYNDROME

Patient complaints: A 42-year-old female presents with episodic sharp stabbing pain in the upper part of the neck with radiation to preauricular area, ear and lower down neck. It worsens during swallowing. She has no complaints of difficulty in swallowing, cough with expectoration. Setting: OPD. Examination of oropharynx: On palpation in the tonsillar fossa hard structures is felt . Q.1: What is the probable diagnosis? Ans: Stylagia or Eagle’s syndrome. Q.2: What is the cause for stylalgia? Ans: It is due to elongated styloid process or calcification of stylohyoid ligament. Q.3: What are the clinical manifestations if Eagle’s syndrome? Ans: Patient complains of pain in the tonsillar fossa and upper neck which radiates to the same side ear. Diagnosis is made by transoral palpation of the styloid process in the tonsillar fossa and a radiograpgh such as anteroposterior view with open mouth or lateral view of skull. Q.4: What is the treatment of Eagle’s syndrome? Many persons may have elongated styloid process but remain asymptomatic and do not need treatment. Symptomatic patient need treatment. Treatment is transoral excision of styloid process with tonsillectomy. Cervical approach can also done alternative to transoral approach. CASE 6: TRIGEMINAL NEURALGIA

Patient complaints: A 45-year-old lady presented with on and off stabbing pain on one side of face for last 6 months. It aggravates on touching tha nasal ala. There is no history of vomiting, unconsciousness and ear discharge. Setting: OPD. ENT examinations: Normal. Imaging: CT scan head is within normal limits. Q.1: What is the most probable diagnosis? Ans: Trigeminal neuralgia. Q.2: What are the characteristic features in trigeminal neuralgia? Ans: Brief paroxysms of severe stabbing pain with normal intense pain phases. The pain is unilateral and may be associated with sensory or motor loss. Pain is precipitated by touch, face wash, shaving, brushing and even loud noise. Q.3: What are the causes for trigeminal neuralgia? Ans: The cause is unknown in majority cases. In some cases, compression of nerve or Gasserian ganglion by central causes including aberrant blood vessels or aneurysm, tumor, cholesteatoma, multiple sclerosis, neurosyphilis and meningitis.

142

Manual of Clinical Cases in ENT and Head–Neck Surgery

Q.4: What is the treatment for trigeminal neuralgia? Ans: Carbamazepine 100–200 mg twice daily. Phenytoin can be given. Intracranial decompression of trigeminal pathway is done if any compression on trigeminal nerve. Q.5: A 20-year-old boy presents with frontal headache since last 3 months. He is also complaining of nasal obstruction and nasal discharge for last 3 months. Examination showed DNS to left side with mucoid secretions in the nose. What is the most possible cause of headache in this patient? Ans: Sinusitis secondary to DNS causing headache.

143

CSF Rhinorrhea

22

CSF Rhinorrhea CASE1: CSF RHINORRHEA

Presenting complaints: A 36-year-old lady presenting with clear watery discharge from the nostril (Fig. 22.1) since 10 days.

Fig. 22.1: Watery discharge from the nostril

History of past illness: No history of head injury in past. Anterior rhinoscopy: Watery fluid coming from the right nostril and the flow is more during bending head forward. Q.1: What is the most possible diagnosis? Ans: CSF rhinorrhea. Leakage of CSF into the nose is called CSF rhinorrhea. It may be clear fluid or mixed with blood as in case of head injury. Q.2: What are the causes of CSF rhinorrhea? Ans: Trauma (accident or surgical), inflammation, neoplasms, congenital lesions and idiopathic. 143

144

Manual of Clinical Cases in ENT and Head–Neck Surgery

Q.3: What are the differenc between the fluid of CSF rhinorrhea and nasal secretions? Ans: CSF fluid

Nasal secretion

History

History of head injury, nasal surgery

Sneezing, running nose or nasal obstruction

Side

Usually unilateral

Usually bilateral

Flow of fluid

Flows in drops or drips. It cannot be sniffed back

Continous flow with sneezing or obstruction and can be sniffed back

Character

Clear and watery

Mucoid

Head position

Flow is more with head bend forward No relation to head bending

Stiffen or not

Does not stiffen in handkerchief

Stiffen in handkerchief

Biochemistry

Same as CSF. Sugar more than 30 mg/cc.

Sugar less than 10 mg/cc.

B2 transferrin

Always present

Absent

Smear

No cellular element

Eosinophil may be present

CT scan with contrast

Show leak area in cribriform plate

No leakage area in cribriform plate

Q.4: What is double target sign? Ans: CSF rhinorrhea after head injury is often mixed with blood and shows double target sign when collected on a piece of filter paper. It shows central red spot (blood) and peripheral lighter halo. CASE 2: CEREBROSPINAL FLUID RHINORRHEA

Presenting complaints: Doctor, I have watery nasal discharge since one month. Setting: Out patient department of otorhinolaryngology History of presenting illness: A 26-year-old male involved in low-speed motor vehicle collision as a front passanger and was brought urgently to emergency department for evaluation. On ENT examination, showed both nasal bone fracture with bloody rhinorrhea that had failed to resolve spontaneously despite direct pressure and application of topical nasal decongestant. General physical examination: The patient is well oriented with time, place and person. Pulse rate—90/minute, BP—110/80 mmHg. He has fever of 101°F. Oral cavity and oropharynx: Right tonsil is congested and swollen. Uvula is pushed to the left side. Neck: Normal. No swelling in the neck Ear, nose, paranasal sinuses and nasopharynx: Within normal limit. Q.1: What is the normal CSF pressure and other parameters related to CSF? Ans: • CSF is formed in the choroid plexus within the lateral, third and fourth ventricle of the brain at a rate of 20 ml per hour.

CSF Rhinorrhea

145

• The total volume of CSF in an adult is 90 to 150 ml. • Normal CSF pressure is 140 mm H2O (10–15 mmHg) in adults and 40 mm H2O in children, although pressure varies with the cardiac and respiratory cycles, activity level and changes in head position. • Maintaining CSF pressure need a delicate balance between CSF secretion and resorption. Q.2: What are the common locations for CSF rhinorrhea? Ans: The most common location for CSF rhinorrhea is the cribriform plate (35%), followed by the sphenoid sinus (26%), anterior ethmoid (18%), frontal sinus (10%), posterior ethmoid (9%) and inferior clivus (2%). The site of leak is important in determining the best surgical approach for repair when surgery is indicated. Q.3: What are the differential diagnoses of CSF rhinorrhea? Ans: • Allergic rhinitis • Vasomotor rhinitis • Senile rhinorrhea • Spontaneous rupture of a mucous retention cyst inside maxillary antrum Q.4: What is CSF otorhinorrhea? Ans: CSF leak from middle or posterior cranial fossa into the middle ear and mastoid air cells and then through the eustachian tube into the nasopharynx and nose. Q.5: What is the commonest cause of CSF rhinorrhea? Ans: Commonest cause was head injury but in all recent series iatrogenic trauma has replaced head injury as the commonest cause. Common surgeries causing CSF leak or transsphenoidal hypophysectomy, intranasal naked eye surgeries, FESS and skull base surgeries. Q.6: What are the investigations done in CSF rhinorrhea? Ans: Diagnostic nasal endoscopy, intrathecal flurescein injection, CSF glucose estimation (30 mg/dl),  2 transferrin estimation, high resolution CT scan, CT cisternography and MR cisternography. Q.7: What are the treatments for CSF rhinorrhea? Ans: Bedrest, lumbar drain, antibiotics, diuretics, endoscopic repair of CSF leak and intracranial approach in case of larger defect.

146

Manual of Clinical Cases in ENT and Head–Neck Surgery

23

Foreign Body in Nose CASE 1: FOREIGN BODY IN NOSE

Presenting complaints: A 5-year-old boy presented with history of purulent nasal discharge from left nostril since 15 days. Examination: It showed a grayish white mass with purulent discharge in left nostril (Fig. 23.1). X-ray nose (lateral view): Radio-opaque foreign body inside the nostril.

Fig. 23.1: Foreign body inside the nostril

Q.1: What is the most probable diagnosis? Ans: Foreign body nose. Q.2: What are the complications of foreign body in the nose? Ans: Nasal infections, sinusitis, rhinolith formation and inhalation of foreign body into the tracheobronchial tree. Q.3: What are the causes of unilateral nasal discharge? Ans: Old foreign body in the nose, unilateral chronic sinusitis, unilateral choanal atresia, antrochoanal polyp, rhinosinusitis, rhinolith and nasal or nasopharyngeal or paranasal sinus masses. 146

Foreign Body in Nose

Q.4: What are the causes of bilateral nasal discharge? Ans: • Allergic rhinitis • Vasomotor rhinit • Bilateral choanal atresia • Ethmoid polyps • Atrophic rhinitis • Adenoid hypertrophy • Granulomatous rhinitis Q.5: What are the common causes for bilateral nasal obstruction? Ans: • Congenital bilateral choanal atresia • Ethmoidal polyps • Allergic rhinitis • Vasomotor rhinitis • Bilateral chronic sinusitis • Large antrochoanal polyp obstructing both choanal • Atrophic rhinitis • Septal hematoma and septal abscess • Bilateral rhinosporidiosis • Nasopharyngeal angiofibroma and other neoplasms • Rhinitis medicamentosa Q.6: What are the common causes for unilateral nasal obstruction? Ans: • Foreign body • Antrochoanal polyp • Congenital unilateral choanal atresia • Congenital atresia of anterior nares • Unilateral sinusitis • Deviated nasal septum • Synechia • Hypertrophy of inferior turbinate • Unilateral rhinosporidiosis • Neoplasms of nose, paranasal sinus or nasopharynx Q.7: What are common causes of nasal obstruction in children? Ans: • Foreign body • Common cold • Adenoid hypertrophy • Choanal atresia

147

148

• • • •

Manual of Clinical Cases in ENT and Head–Neck Surgery

Nasal allergy Catarrhal rhinitis Nasopharyngeal angiofibroma Neoplasms

Q.8: What are the common causes for fetor/foul smell from nose? Ans: • Foreign body in nose • Atrophic rhinitis • Sinusitis • Nasal or paranasal sinus neoplasms • Nasal diphtheria • Tuberculosis of the nasal cavity • Gumma • Nasal cholesteatoma CASE 2: MYIASIS OF NOSE

Presenting complaints: A 65-year-old lady presenting with nasal bleeding from both nostril and pain in the nose since 15 days. She has also bilateral nasal obstruction and foul smelling from nose. On examination: Anterior rhinoscopy showed crawling of the maggots in both nostrils. Blood pressure is 130/80 mm Hg, pulse is 76/min. Q.1: What is the diagnosis? Ans: Myiasis of nose (maggots). Myiasis is an infestation of the nose by maggots which are larvae of a house fly. Q.2: What are the etiologies of the myiasis of nose? Ans: Bad hygienic conditions like atrophic rhinitis or infective sinusitis. Q.3: What are the clinical features of the myiasis of nose? Ans: • Diffuse swelling around nose and eyes • Nasal obstruction • Fever and toxemia • Foul smelling • Maggots coming out of the nose Q.4: What is the treatment of the myiasis of nose? Ans: • Removal of maggots by instillation of drops containing paraffin, olive oil with chloroform • Alkaline nasal douching • Turpentine oils in the nose • Antibiotics and supportive treatment

Foreign Body in Nose

149

CASE 3: RHINOLITH

Presenting complaints: A 34-year-old man presented with right nasal block since 2 months and nasal discharge from the right nostril since 15 days. On examinations: A greyish mass (Fig. 23.2) in the floor of the right nasal cavity. On probing, it was felt like stony concretion.

Fig. 23.2: Rhinolith

Q.1: What is the diagnosis? Ans: Rhinolith. These are stomy concretions forming in the nose. The nucleus of the rhinolith is foreign body, blood or pus. Q.2: What are the compositions of the rhinolith? Ans: Phosphates and carbonates of calcium and magnesium often arranged in layers and form the stony concretions. They look like whitish or greyish white in color. Q.3: What are the clinical symptoms of the rhinolith? Ans: • Nasal obstruction • Nasal discharge • External nasal swelling • Pain • Stony mass found in the nasal cavity Q.4: How to manage the rhinolith? Ans: • Radiology—soft tissue X-ray of nose shows radio-opaque shadow which is existence of the rhinolith. • Removal of the rhinolith is done under general anesthesia. • Occasionally lateral rhinotomy is done for removal of the foreign body.

150

Manual of Clinical Cases in ENT and Head–Neck Surgery

CASE 4: FOREIGN BODY INSERTION

Presenting complaints: A 3-year-old boy inserted a portion of chalk into his right nostril. Immediately the parent of the child brought him to outpatient department of otorhinolaryngology. On examination: Showed a whitish chalk piece is inside the right nostril (Fig. 23.3).

Fig. 23.3: Chalk piece in the right nostril

Q.1: How this foreign body was removed? Ans: The foreign body was removed in awake with help of nasal foreign body hook. Q.2: What are the common foreign bodies inserted into the nasal cavity by children? Ans: Common foreign bodies may be organic such as wood, paper, cotton, foam or rubber and inorganic such as metal, button, beads and plastic.

III

Head and Neck Diseases 24. Oral Cavity 25. Tonsils and Adenoids 26. Tumours of Oral Cavity and Oropharynx 27. Neck Mass 28. Vocal Cord Paralysis 29. Nasopharynx 30. Tumors of Hypopharynx 31. Tracheostomy 32. Congenital Lesions of Larynx 33. Infections of Larynx 34. Foreign Body in Airway 35. Hoarsness of Voice 36. Laryngeal Growthj 37. Esophagus 38. Neck Abscess

24

Oral Cavity CASE 1: APHTHOUS STOMATITIS

Presenting complaints: A 16-year-old boy complaining painful swallowing since 2 days. Setting: Out patient department History of present illness: He was apparently alright 2 days back. To start with pain in the mouth during taking food, which is more during taking spicy foods. History of past illness: He had fever 5 days back and after taking certain medicines from local doctor, relieved. On examination: In oral cavity mucosa, multiple small shallow necrotic ulcers with a clean base and surrounding erythema (Fig. 24.1).

Fig. 24.1: Aphthous ulcers

Q.1: What is the diagnosis? Ans: Aphthous stomatitis. Q.2: What is the etiology of aphthous stomatitis or aphthous ulcer? Ans: Aetiology is not known but may be due to vitamin deficiency, viral, psychogenic and autoimmune factors. These may be associated with Behcet’s syndrome or Sutton’s disease, in which there are major recurrent aphthous ulcers. 153

154

Manual of Clinical Cases in ENT and Head and Neck Surgery

Q.3. What is the Behçet’s syndrome? Ans: Behçet’s syndrome is characterized by: Ulceration of buccal mucosa, ulcers on genitals, ocular lesions such as iridocyclitis and corneal ulcers. Its cause is not known but may be due to virus infection or autoimmune causes. Q.4: What is the treatment of aphthous stomitis? Ans: Mentain oro-dental hygiene, supplement of vitamin B complex and vitamin C, topical steroid applications and analgesic lozenges. Q.5: What are other ulcers found in oral cavity? Ans: • Tuberculosis ulcer is painful from the beginning. • Malignant ulcers commonly cause referred pain in the ear. • Aphthous ulcers and dental ulcers are quite painful. • Syphilitic ulcer: It is painless and situated at the dorsum. It has punched out edges and wash leather slough. Q.6: What is Fordyce’s spot? Ans: They are aberrant sebaceous glands present under the buccal mucosa and shine through it as yellowish or yellow brown spots. CASE 2: ORAL SUBMUCOUS FIBROSIS

Presenting complaints: A 41-year-old man complaining progressive restricted opening of the mouth since six months. He had also burning sensation in mouth during taking chilies and spicy food. Personal history: He has habit of betel nut chewing since 10 years and also addicted to gutkha since 5 years. Oral cavity examination: Blanching, fibrosis and increased hardness of mucosa of soft palate and buccal mucosa (Fig. 24.2). Uvula is reverted.

Fig. 24.2: Oral submucosal fibrosis

Oral Cavity

155

Q.1: What is the most possible diagnosis? Ans: Oral submucosal fibrosis. It is a chronic insidious disease of the oral cavity characterized by juxtraepithelial deposition of fibrous tissue in the oral cavity and occasionally in the pharynx. Q.2: What is the history behind the oral submuosal fibrosis (OSMF)? Ans: OSMF was first described in India by Joshi in 1953. Desa described the condition in detail with sequence of events and so, it often called Desa syndrome. Q.3: What are the clinical manifestations in OSMF? Ans: Reduced mouth opening, soreness in mouth, burning sensation in mouth on taking chilies or spicy food, ulcerations in oral cavity, restriction in protruding the tongue. The common sites of oral cavity involved are soft palate, faucial pillars and buccal mucosa. Q.4: What are the causative factors in OSMF? Ans: Chewing betel nut, chewing tobacco or smoking, taking excessive chilies and spicy food. Dietary deficiency like vitamin B complex, vitamin A and iron. This disease is common in Indian subcontinent. The prevalence rate varies from 2 to 5 per 1000. Q.5: What are the treatment options in OSMF? Ans: Avoid betel nuts, pan, tobacco and spicy food. Supplentations of vitamins (B complex and A), topical injection of steroids, topical steroid and hylase injection. Surgical treatment options are laser excision of the fibrous bands. Excision of the fibrous bands and skin grafting. Excision of fibrous bands and tongue flap reconstruction. Selecting tooth extraction. Forced opening of jaws using screw obturator between upper and lower teeth. CASE 3: EPULIS

Presenting complaints: A 45-year-old lady presented with a pinkish nodule at the lateral surface of the gum near premolar tooth since 8 months. Setting: OPD. On examination: A nodular mass which is firm, nontender and pedunculated attached to the gum (Fig. 24.3).

Fig. 24.3: Nodular mass in the gum

156

Manual of Clinical Cases in ENT and Head and Neck Surgery

Q.1: What is the most possible diagnosis? Ans: Epulis. Epulis means upen the gum. Any swelling on the gum is called epulis. Q.2: What are the types of epulis? Ans: Fibrous epulis (it is firm, pinkish, nodular, nontender swelling arising from the gum and does not bleed on toush), myeloid epulis (it is firm, purple coloured rapidly growing mass), granulomatous epulis (it is soft red swelling, bleeds on touch and is related to caries tooth) and carcinomatous epulis (it is actually fungating growth. The swelling may be painful, friable and has bleeding tendency. Regional lymph nodes are enlarged). Q.3: What are the pathological features of the epulis? Ans: It arises from periodontal membrane at the neck of the incisor or premolar teeth. Pathologically it originates from interdental papilla or in response to local irritation (from sharp margin of carious cavity). It is more common in women and among young adults. Q.4: What is the treatment of epulis? Ans: Excision of the epulis with cautery of the base to stop bleeding. CASE 4: VERRUCOUS CARCINOMA

Presenting complaints: A 72-year-old male who has been chewing the tobacco for the past 50 years present with a six months history of large, fungating, soft papillary lesions in the oral cavity. Examination: Lymph nodes of the neck are not palpable. Biopsy: Two biopsies were taken from the lesion where histopathological examination showed benign appearing papillomatosis with hyperkeratosis and acanthosis infiltrating the adjacent tissues. Q.1: The most probable diagnosis? Ans: Verrucous carcinoma. Q.2: What is the histopathological features of the verrucous carcinoma? Ans: Histologically verrucous carcinoma are characterized by marked acanthosis, hyperkeratosis often with broad bullous process showing central columns of keratin. There is no cytological evidence of malignancy. Q.3: The commonest site of oral cancer among Indian population is___? Ans: Alveobuccal complex. • Most common site of oral cancer in the world is tongue. • Most common histological carcinoma of lip is squamous cell carcinoma. • Most common histological carcinoma of upper lip is basal cell carcinoma. • Most common malignant tumor of adult males in India is oral cancer. • Most common malignant tumor of females in India is carcinoma cervix. • Most common malignant tumor of female in world is carcinoma breast. • Most common malignant tumor of adult males in world is lung carcinoma.

Oral Cavity

157

CASE 5: LEUKOPLAKIA

Presenting complaints: A 58-year-old man presenting with a white patch or plaque on the right buccal mucosa since 3 months. He had habit of gutkha chewing. Q.1: What is the most possible diagnosis? Ans: Leukoplakia Q.2: What is leukoplakia? Ans: It is defined as a white patch or plaque that cannot be characterized clinically or pathologically as any other disease. Q.3: What are the common sites and age of oral cavity leukoplakia? Ans: It is most commonly found on the buccal mucosa and oral commissures. It is most commonly found among men older than 50 years. Q.4: What are the common predisposing factors for development of the oral cavity leukoplakia? Ans: • Tobacco use • Chronic cheek biting • Chronic hyperplastic candidiasis • Electrogalvanism among alloys used in dental work • Syphilitic glossitis • Plummer-Vinson syndrome • Submucosal fibrosis Q.5: What is erythroplakia? Ans: It is defined as a lesion which presents as a bright red, velvety plaque, which cannot be characterized clinically or pathologically as any other recognizable condition. CASE 6: TONGUE TIE

Presenting complaints: A 12-year-old boy presenting with dysphonia/speech defect since birth. The hearing of the child is normal in both ears. On examination: The child is unable to protrude the tongue beyond the lower incisors. There is shortening of the frenulum of the tongue (Fig. 24.4).

Fig. 24.4: Tongue tie

158

Manual of Clinical Cases in ENT and Head and Neck Surgery

Q.1: What is the most possible diagnosis? Ans: Tongue tie or ankyloglossia Q.2: What is the treatment of the tongue tie? Ans: Treatment of any significant tongue tie is transverse release and vertical closure. CASE 7: GLOSSOPHARYNGEAL NEURALGIA

Presenting complaints: A 35-year-old patient complains of pain in the throat which is aggravated on swallowing and radiates to the ear and posterior part of the tongue. Q.1: The likely diagnosis is? Ans: Glossopharyngeal neuralgia Q.2: What is glossopharyngeal neuralgia? Ans: Glossopharyngeal nerve supplies base of tongue, tonsillar fossa and the ear. In glossopharyngeal neuralgia, pain radiates to these areas and is precipitated by swallowing or yawning. It is relieved by anesthesing the area. CASE 8: HEMANGIOMA OF TONGUE

Presenting complaints: A 35-year-old lady presented with large size tongue which progreeses its size since 6 months. Examination: The tongue swelling is bluish colored and compressible and refill rapidly on withdrawal of pressure (Fig. 24.5). It is non-pulsatile.

Fig. 24.5: Hemangioma of tongue

Q.1: What is the possible diagnosis? Ans: Cavernous hemangioma of the tongue. Q.2: What is hemangioma? Ans: Hemangioma is a benign tumor which occurs in the endothelial lining of the blood vessels. It is often found in the skin or internal organs.

Oral Cavity

159

Q.3: Which investigation is required in this case? Ans: Arteriography may help to detect the feeding vessels in large lesions. Q.4: What are the treatment options for cavernous hemangioma? Ans: • Injection of sclerosing agents • Cauterization • Laser Q.5: What are the commonly used sclerosing agents? Ans: • Boiling water • Hypertonic solution • Sodium morrhuate (3%) • Bleomycin CASE 9: PLUMMER-VINSON SYNDROME

Presenting complaints: A 30-year-old lady presented with dysphagia since 3 months. On examination: She has koilonychia, angular chelitis and anemia. Q.1: What is the most probable diagnosis? Ans: Plummer-Vinson syndrome Q.2: What is Plummer-Vinson syndrome? Ans: It is also called Paterson-Brown-Kelly syndrome. It consists of iron-deficiency anemia, dysphagia (due to esophageal web), koilonychia (spooning of nails) and angular cheilits. It is a risk factor for postcricoid carcinoma. It predominantly affect female. Q.3: What are the investigations required in this case? Ans: Barium swallow will detect the postcricoid web and same can be seen on esophagoscopy. It is due to subepithelial fibrosis in this region. Q.4: What are the treatment options for Plummer-Vinson syndrome? Ans: Anemia is corrected by oral/parentral iron. Serum levels of iron are more important than hemoglobin level. Associated vitamin B 12 and B 6 deficiency should also be corrected. Dilatation of the webbed area should be done by esophageal bougies. CASE 10: BEHÇET’S SYNDROME

Presenting complaints: A 30-year-old man presented with multiple aphthous ulcers in the oral cavity, genital ulcers and uveitis. Q.1: What is the most possible diagnosis? Ans: Behçet’s syndrome (oculo-orogenital syndrome)

160

Manual of Clinical Cases in ENT and Head and Neck Surgery

Q.2: What are the important causes of ulcers in oral cavity? Ans: Important causes of ulcers in the oral cavity are infective (viral-HIV, bacterial, fungal), traumatic, neoplastic, allergic, blood disorders, skin disorders, nutritional disorders and hormonal imbalance. Q.3: What is trench mouth? Ans: Vincent’s angina is also called trench mouth. It occurs due to gram-negative fusiform bacilli and a spirochaete (Borellia vincentii) Q.4: What is tonsillolingual sulcus? Ans: Tonsillolingual sulcus is also called surgeon’s graveyard because growth/ ulcerative lesion/malignancy in this region may be overlooked or missed. Q.5: What is hand, foot and mouth disease? Ans: It is a viral infection affecting the children. Oral lesions are seen on the palate, tongue and buccal mucosa. Vesicles also develop on the skin of the hands, feet and sometimes buttocks.

25

Tonsils and Adenoids CASE 1: ACUTE TONSILLITIS

Presenting complaints: Doctor, I have throat pain and fever for last 5 days. Now I am unable to swallow the food since one day. Setting: Out patient department History of presenting illness: A 10-year-old boy presented with severe throat pain, fever and odynophagia for 5 days. Now he is unable to swallow his saliva since one day. He has severe pain in right ear. General physical examination: The patient is well oriented with time, place and person. Pulse rate—90/minute, BP—110/80 mmHg. He has fever of 101°F. Examination of oral cavity and oropharynx: Bilateral tonsils are congested and swollen. Multiple whitish patches are present over medial surface of tonsils (Fig. 25.1).

Fig. 25.1: Appearance of oropharynx of the patient

Examination of the neck: Bilateral jugulodiagastric nodes are enlarged and tender. Examiantion of the ear, nose, paranasal sinuses and nasopharynx: Normal. Q.1: What is probable diagnosis? Ans: Acute follicular tonsillitis. 161

162

Manual of Clinical Cases in ENT and Head and Neck Surgery

Q.2: What are the signs in acute tonsillitis? Ans: • Inflammed tonsils, anterior pillars, soft palate, uvula. • Bilateral jugulodigastric lymph nodes are enlarged and tender. • Tongue may be red, i.e. strawberry tongue. Q.3: What are the types of acute tonsillitis? Ans: • Acute catarrhal tonsillitis. • Acute follicular tonsillitis. • Acute parenchymal tonsillitis. • Acute membranous tonsillitis. Q.4: What are the differential diagnoses of acute tonsillitis? Ans: • Scarlet fever • Diphtheria • Agranulocytosis • Infectious mononucleosis Q.5: How acute follicular tonsillitis is differentiated from acute diphtheria? Ans: Acute follicular tonsillitis

Acute diphtheria

• Common age—children of 6–20 years or more. • History of recurrent tonsillitis. Acute onset.

• Children under 10 years (2–5 years). • History of exposure to diphtheria. Slow onset. • Fever (up to 102°). • Mild throat pain. • Toxemia—present. • Pulse rate is out of proportion to fever. • Membrane formation extensive on tonsil, difficulty and leaves a raw bleeding surface and it reforms after removal. • Throat swab shows Corynebacterium. • Massive cervical lymphadenopathy (bull’s neck). • Schick’s test is positive. • Albuminuria is present. • Treatment is with antidiphtheritic serum (ADS) and antibiotics.

• • • • • • • • • •

Fever (102°–105°F). Severe throat pain. Toxemia—absent. Pulse rate is proportionate to rise of body temperature. Dirty white pseudomembrane limited to tonsil only. Throat swab shows bacteria of Streptococcus, Staphylococcus or Haemophilus influenzae. Jugulodiagastric nodes are enlarged and tender. Schick test is negative. Albuminuria is absent. Treatment with antibiotics.

CASE 2: CHRONIC TONSILLITIS

Presenting complaints: Doctor, I have recurrent attacks of throat pain since 3 years. Setting: Out patient department.

Tonsils and Adenoids

163

History of presenting illness: A 13-year-old male presented with recurrent attacks of throat pain since 3 years, approximately 5 to 6 attacks per year. During throat pain he presented with odynophagia and fever. General examination: The patient is well oriented with time, place and person. Blood Pressure—122/80 mm of Hg, Pulse—80/minute. Examination of oral cavity and iropharynx: • Both tonsils are congested and enlarged. • Flushing of anterior pillars. • Pressing the anterior pillars of tonsils with tongue depressure, pus come out from tonsils. Examination of the neck: Enlarged and non-tender jugulodigastric lymph nodes. Q.1: What is the most probable diagnosis? Ans: Chronic parenchymatous tonsillitis.

Fig. 25.2: Chronic parenchymatous tonsillitis

Q.2: What are the types of chronic tonsillitis? Ans: Chronic follicular tonsillitis, chronic parenchymatous tonsillitis and chronic fibroid tonsillitis. Q.3: What is Irwin Moore’s sign? Ans: Pus from the crypts of the tonsils on squeezing, called Irwin Moore sign. This is the diagnostic sign of chronic tonsillitis. Q.4: What are the points in favor of your diagnosis (chronic tonsillitis)? Ans: • Recurrent throat pain and difficulty in swallowing. • Tonsils are enlarged and egorged. • Flushing of the anterior pillars. • Pus comes out of the tonsillar crypts on pressing the anterior pillars. • Jugulodigastric lymph nodes are palpable and non-tender.

164

Manual of Clinical Cases in ENT and Head and Neck Surgery

Q.5: What are the investigations are done in chronic tonsillitis? Ans: Investigations are done for diagnostic and routine preoperative purpose. These are: • Complete blood count with coagulation parameters and serology. • ASO titer. • Throat swab for culture and sensitivity. • Chest X-ray. • ECG. Q.6: What are the complications of chronic tonsillitis? Ans: • Peritonsillar abscess • Parapharyngeal abscess • Tonsilloliths • Tonsillar cysts • Rheumatic heart disease • Glomerulonephritis Q.7: What are the differential diagnoses for tonsillar patch? Ans: • Membranous tonsillitis • Diphtheria • Vincent’s angina • Infectious mononucleosis • Agranulocytosis • Leukemia • Aphthous ulcer • Candida infection Q.8: How the tonsillar enlargement is graded? Ans: Brodsky grading scale • Grade 0 tonsil—tonsils are within the tonsillar fossa. • Grade 1—tonsils just outside of the tonsillar fossa and occupy 25% of the oropharyngeal width. • Grade 2—tonsils occupy 26–50% of the oropharyngeal width. • Grade 3—tonsils occupy 51–75% of the oropharyngeal width. • Grade 4—tonsils occupy >75% of the oropharyngeal width. Q.9: What is absolute contraindication of tonsillectomy? Ans: Epidemic poliomyelitis is only one absolute contraindication for tonsillectomy.

Tonsils and Adenoids

165

CASE 3: PERITONSILLAR ABSCESS

Presenting complaints: Doctor, I have severe throat pain and fever for last one week. Now I am unable to swallow the food since one day. Setting: Out patient department. History of presenting illness: A 16-year-old male presented with severe throat pain, fever and odynophagia for 7 days. Now he is unable to swallow his own saliva since one day. He has severe pain in right ear. General physical examination: The patient is well oriented with time, place and person. Pulse rate—90/minute, BP—110/80 mmHg. He has fever of 101°F. Examination of oral cavity and oropharynx: Right tonsil is congested and swollen. Uvula is pushed to the left side (Fig. 25.3).

Fig. 25.3: Findings of oropharynx

Examination of the neck: Right side jugulodigastric node is enlarged and tender. Examiantion of the ear, nose, paranasal sinuses and nasopharynx: Normal. Q.1: What is the most probable diagnosis of this case? Ans: Right peritonsillar abscess. Q.2: What is treatment of choice in peritonsillar abscess? Ans: Treatment includes hospitalization, incision and drainage and broad spectrum parentral antibiotics. Q.3. What is interval tonsillectomy? Ans: Interval tonsillectomy is done after 6 weeks of peritonsillar abscess. Q.4: What is abscess or hot tonsillectomy? Ans: Tonsillectomy done in peritonsillar abscess without performing incision and drainage, called abscess or hot tonsillectomy. Hot tonsillectomy has the risk of rupture of the abscess during anesthesia and excessive bleeding during the surgery.

166

Manual of Clinical Cases in ENT and Head and Neck Surgery

Q.5: What are the complications of peritonsillar abscess? Ans: Parapharyngeal abscess, edemal of the larynx, septicemia, jugular vein thrombosis, pneumonitis or lung abscess and spontaneous hemorrhage from carotid artery or jugular vein. Q.6: What are the causes of unilateral tonsillar enlargement? Ans: • Lymphoma (NHL and Hodgkin’s lymphoma) • Squamous cell carcinoma • Leukemia • Metastasis • Plasmacytoma • Tuberculosis/sarcoidosis • Quinsy • Infective (candidiasis, actinomycosis) • Parapharyngeal abscess • Parapharyngeal tumors CASE 4: INFECTIOUS MONONUCLEOSIS

Presenting complaints: Doctor, I have sore throat and fever since seven days. Setting: Out patient department. History of presenting illness: A 15-year-old boy presenting with sore throat and fever since last seven days. He was taking some medication (ampicillin) from local doctor after which he developed skin rash. General physical examination: The patient is well oriented with time, place and person Pulse rate: 92/minute Blood pressure: 110/78 mm Hg. Oral cavity and oropharyngeal examination: Erythematous congested tonsils with grayish white membrane (Fig. 25.4). Neck examination: Bilateral jugulodigastric node enlarged and tender.

Fig. 25.4: Oropharynx of the patient

Tonsils and Adenoids

167

Q.1: What is the most possible diagnosis? Ans: Infectious mononucleosis. This often affects young adults. In this case, both tonsils are very much enlarged, congested and covered with membrane. Local discomfort is marked. Lymph nodes are enlarged in the posterior triangle of the neck along with splenomegaly. Q.2: What is the etiology of infectious mononucleosis? Ans: Epstein-Barr virus. Q.3: What are the complications of untreated infectious mononucleosis? Ans: Splenomegaly and myocarditis. Q.4: How infectious mononucleosis is diagnosed? Ans: The diagnosis is established by positive monospot test and abnormal lymphocytes in peripheral blood smear. Blood smear may show more than 50% lymphocytes, of which about 10% are atypical. White cell count may be normal in the 1st week but rises in the 2nd week. Pau-Bunnell test (monospot test) will show high titer of heterophil antibody. Q.5: What is the treatment of infectious mononucleosis? Ans: Aspirin and steroids. CASE 5: ADENOID HYPERTROPHY

Presenting complaint: Doctor, my 4 years son has mouth breathing during sleep and nasal discharge since two months. Setting: Out patient department History of presenting illness: A 4-year-old male child presenting with history of mouth breathing during sleep associated with snoring. He has nasal discharge from both nostrils along with bilateral nasal obstruction since two months. General physical examination: The mental condition of the child is subnormal. Blood pressure—110/70 mm Hg, pulse—90/m. Anterior rhinoscopy: Bilateral purulent nasal discharge. Posterior rhinoscopy: Adenoid is grossly hypertrophied blocking the both choanae (Fig. 25.5).

Fig. 25.5: Picture of nasopharynx

168

Manual of Clinical Cases in ENT and Head and Neck Surgery

Q.1: What is the most probable diagnosis? Ans: Adenoid hypertrophy. Q.2: Differential diagnosis of adenoid hypertrophy? Ans: • Nasopharyngeal angiofibroma. • Antrochoanal polyp. • Nasopharyngeal rhinosporidiosis. • Other nasopharyngeal tumors. Q.3: What is adenoid facies? Ans: A child with adenoid hypertrophy has characteristic facial expression, called adenoid facies. The characteristic features are: • Half open mouth • Dull facial expression • Elongated face • Pinched nostrils • Prominent and croweded upper teeth • Prominence of upper jaw • Recession of lower jaw • Loss of nasolabial fold • Hitched up upper lip • High arched palate • Coated tongue Q.4: What are the differences between tonsils and adenoids? Ans: Tonsils

Adenoids

Paired

Single

Crypts are seen on the medial surface

Vertical furrows are seen

Lined by stratified squamous epithelium

Ciliated columnar epithelium

Does not regress by 7 or 8 years of age

Regress

False capsule is present at lateral surface

No capsule

Q.5: What are the common indications of adenoidectomy in children with adenoid hyperplasia? Ans: • Nasal obstruction/mouth breathing • Recurrent otitis media • Otitis media with effusion Q.6: In which condition, the adenoidectomy is avoided? Ans: Adenoidectomy should be modified or avoided in cases of submucous cleft palate.

Tonsils and Adenoids

169

CASE 6: ADENOID HYPERTROPHY

Presenting complaints: A 7-year-old boy presenting with recurrent upper respiratory tract infections, mouth breathing, nasal obstruction and decreased hearing since 3 months. Setting: Out patient department History present illness: He was apparently alright 3 months back. To start with he developed mouth breathing, nasal block and decreased hearing since 3 months. He had history of recurrent common cold. Examination: He has elongated face, high arched palate, over crowded teeth. Anterior rhinoscopy—mucoid secretions in both nostrils; cold spatula test—no fogging in both nostrils. Q.1: What is the possible diagnosis? Ans: The symptoms of nasal obstruction, mouth breathing and hearing impairment favor the diagnosis adenoids. Q.2: What are the investigations required in this case? Ans: Diagnostic nasal endoscopy, X-ray nasopharynx (lateral view) (Fig. 25.6), CT scan of the nose, paranasal sinuses and nasopharynx, pure tone audiometry and tympanometry.

Fig. 25.6: X-ray of nasopharynx (lateral view) showing Grade 4 adenoid

Q.3: What are the treatments required in this case? Ans: As the child has adenoid hypertrophy, adenoidectomy would be required. As the child suffers from hearing loss (due to otitis media with effusion), a myringotomy with grommet insertion will be required. Q.4: What is the disadvantage for doing only myringotomy? Ans: Simply doing myringotomy will lead to recurrence unless the cause (adenoids) is removed.

170

Manual of Clinical Cases in ENT and Head and Neck Surgery

CASE 7: TONSILLECTOMY

Presenting complaints: A 5-year-child scheduled for tonsillectomy. On the day of surgery presenting with running nose, fever and cough. Setting: Out patient department History of present illness: A 5-year child presents with fever, nasal discharge and cough on the day of surgery. Examination of oral cavity and oropharynx: Bilateral tonsils are enlarged and congested Q.1. A. B. C. D.

Which should be the most appropriate decision on the surgery? Surgery should be cancelled Can proceed with surgery if there is no wheeze and chest is clear Get a chest X-ray before proceeding for surgery Cancel surgery for 3 weeks and put the patient on antibiotics

Ans. D: Cancel surgery for 3 weeks and put the patient on antibiotics Recent URTI is a relative contraindication for tonsillectomy and hence the surgery should be postponed preferable for 6–8 weeks and the patient put on antibiotics. CASE 8: COMPLICATIONS OF TONSILLECTOMY

Presenting complaints: Doctor, I have bleeding from my mouth with sudden onset. Setting: Emergency department History of present Illness: A 15-year-old boy presents with one episode of hemorrhage from mouth today. He had undergone tonsillectomy one day back. General physical examination: The patient is well oriented with time, place and person. Blood pressure is 110/70 mm Hg, pulse rate is 92/minute. Examination of oral cavity and oropharynx: Active bleeding from right tonsillar fossa. Q.1: A. B. C. D.

What is the best treatment for this patient? External gauze packing Irrigation with saline Antibiotics and mouth gargle Reoperate immediately

Ans. D: Reoperate immediately Bleeding within 24 hours of tonsillectomy is the diagnosis of reactionary hemorrhage, which is the most dangerous complication of tonsillectomy. It needs immediate exploration of the tonsillar fossa under GA for locating and ligating the bleeding point. Q.2: What is the commonest postoperative complication of tonsillectomy? Ans: Hemorrhage. Q.3: What are the types of postoperative tonsillectomy hemorrhage? Ans: It is of 3 types. • Primary hemorrhage: Bleeding during surgery—due to recent infection, undetected bleeding diathesis, poor technique.

Tonsils and Adenoids

171

• Reactionary hemorrhage: Bleeding within 24 hours.This is the most dangerous complication of tonsillectomy. It is due to slippage of ligatures, postoperative retching by patient or increase in BP opening up collapsed vessels. It needs immediate re-exploration under GA. • Secondary hemorrhage: Bleeding occurs around 5–7 days. It is due to infections of the tonsillar fossa. Q.4: What is the average blood loss during tonsillectomy? Ans: 50–80 ml whereas it is 80–120 ml in combined adenotonsillectomy. CASE 9: TONSILLECTOMY

Presenting complaints: A 6-year-old child presenting with recurrent respiratory tract infection, mouth breathing and decreased hearing. Setting: Out patient department. Q.1: What is the treatment? Ans: This child is probably suffering from adenotonsillar hypertrophy which is causing recurrent respiratory tract infections and mouth breathing. Hypertrophied adenoids/tonsils might have lead to eustachian tube blockade with a resultant SOM which explains decreased hearing. So the treatment is myringotomy with grommet insertion along with adenotonsillectomy in the same sitting. Q.2: What is rose position? Ans: Tonsillectomy is done in rose position. The advantage of this position is that blood collects in nasopharynx and not in throat. Q.3: What is tonsillectomy position? Ans: Tonsillectomy position is made after tonsillectomy. It is left lateral position of the patient. CASE 10: TONSILLECTOMY

Presenting complaints: A 5-year-old child is planned for tonsillectomy. On the day of surgery he had running nose, fever and dry cough. Examination of the oral cavity and oropharynx: Bilateral tonsils are enlarged and congested. Q.1: Should we go for surgery? Ans: Surgery should be cancelled for 3 weeks and patient to be on antibiotic. As tonsillectomy is an elective operation, so should not be undertaken in presence of respiratory tract infections. During upper respiratory tract infections, patient may develop laryngospasm with airway manipulation. This complication carries the risk of significant morbidity and even mortality if taken for surgery under general anesthesia.

172

Manual of Clinical Cases in ENT and Head and Neck Surgery

Q.2: What are other contraindications of tonsillectomy? Ans: The contraindications for tonsillectomy are: • Hemoglobin levels less than 10 gm%. • Children under 3 years of age. • Overt or submucous cleft palate. • Bleeding disorders. • At the time of epidemic polio. • Uncontrolled systemic diseases like diabetes, cardiac diseases, hypertension or asthma. • Tonsillectomy is avoided during the period of menses. Q.3: What are the absolute indications for tonsillectomy? Ans: Huge hypertrophic tonsils causing oropharyngeal obstruction and causing obstructive sleep apnea, suspected malignancy of the tonsils for biopsy and treatment. Q.4: What are the postoperative cares for tonsillectomy patients? Ans: • Patient is kept in tonsillar position, where head is kept low and the patient lies in lateral position to prevent aspiration of blood. • Nil orally for six hours. • Strict watch kept over the temperature, pulse and respiration every hour for first 4 to 5 hours. • Broad spectrum antibiotics are given for 5–7 days. • Feeding: First 48 hours—cold food like ice cream, ice cubes, cold drinks without gas/soda. After 48 hours—lukewarm diet. Cold drinks and ice cream are given for first 24–48 hours as the nerve endings are exposed causing cold anesthesia. Warm liquids are started after 24–48 hours as the inflammation starts and warmth is always soothing for the inflammation. • Gargles: Weak hydrogen peroxide gargles. Hydrogen peroxide gargles are helpful, because when it comes in contact with the slough, there is release of nascent oxygen which helps in the contraction of blood vessels expelling small clots thereby causing closure of mouth of blood vessel thus stopping bleeding. CASE 11: REACTIONARY HEMORRHAGE

Presenting complaints: A 16-year-old male patient presents with hemorrhage 4 hours after tonsillectomy. On examination: A large blood clots over the right tonsillar fossa. General examination: BP is 120/76 mm Hg and pulse is 90/min. Patient is pallor. Q.1: What is the most possible diagnosis? Ans: It is reactionary hemorrhage. Reactionary hemorrhage is common among: • Adults • Males

Tonsils and Adenoids

173

• Patients with history of infection • Recurrent acute tonsillitis • Infectious mononucleosis. Q.2: What are reasons for reactionary hemorrhage after tonsillectomy? Ans: Slippage of knot tied at the time of surgery, rebound hypertension on recovery from general anesthesia causes blood vessels to open and bleeds. Q.3: What is the treatment of reactionary hemorrhage after tonsillectomy? Ans: Arrange blood if required. Correct fluid and electrolyte if any. If any blood clots in the tonsillar fossa, it should be removed as clot prevents clipping action of superior constrictor muscle on vessels. If bleeding is not controlled by above methods, ligation/electrocoagulation of bleeding vessels are done under general anesthesia. Q.4: What are the causes of primary hemorrhage in tonsillectomy? Ans: Primary hemorrhage is caused by recent infection, undetected bleeding diathesis, poor technique. It is controlled with pressure, electrocoagulation and ligature. Q.5: What is the cause of seconday hemorrhage after tonsillectomy? Ans: It seen between 5 and 10 postoperative day. It is often caused by infection. It is an emergency. Q.6: What is the treatment of secondary hemorrhage? Ans: Hospitalization. Broad spectrum intravenous antibiotics. Transfusion of blood depending on blood loss. Under general anesthesia, bleeding vessels are ligated or electrocauterization done. If bleeding is not controlled, approximation of pillars is done with mattress sutures or even external carotid artery ligation may be needed if bleeding control failed by above methods. Q.7: What is the incidence of reactionary hemorrhage after tonsillectomy? Ans: 0.5 to 1%. CASE 12: TONSILLAR CARCINOMA

Presenting complaints: Doctor, I have throat pain, foul breath, odynophagia and right sided ear pain since 3 months. Setting: Out patient department History of presenting illness: A 60-year-old farmer came to ENT OPD with complaints of throat pain, foul breath, odynophagia and right sided otalgia since 3 months. He has smoking habit with 10 cigarette per day since 20 years and drinks socially. He has no trismus. General physical examination: The patient is well oriented with time, place and person. Pulse rate: 74/minute Blood pressure: 140/88 mm Hg.

174

Manual of Clinical Cases in ENT and Head and Neck Surgery

Oral cavity and oropharyngeal examination: He has a friable, ill-defined, ulcerated mass in the right tonsillar area that extends into the soft palate and base of tongue mucosa. The tumor is 4 cm in greatest dimension. Neck examination: He has a 3 cm single, enlarged, firm upper jugular lymph node (Level II). Q.1: What is the best possible diagnosis? Ans: Right tonsillar malignancy. But biopsy is essential for confirmation. Q.2: What is the staging of tonsillar carcinoma? Primary Tumor (T) T1: Primary tumor 4 cm in greatest dimension. T4: Primary tumor involving surrounding structures like bone, soft tissue of the neck and tongue. Regional lymph nodes (N) NX: Nodal status cannot be determined. N0: No evidence of nodal metastasis. N1: Metastasis to a single ipsilateral node 3 cm in greatest dimension. N2a: Metastasis to a single ipsilateral node 3–6 cm in greatest dimension. N2b: Metastases to multiple ipsilateral nodes, none >6 cm. N2c: Metastases to bilateral or contralateral nodes, none >6 cm. N3: Metastasis to a lymph node >6 cm. Distant metastasis (M) MX: Distant metastasis cannot be determined. M0: No distant metastasis. M1: Distant metastasis. Q.3: What are the imagings done in this case? Ans: A computed tomography (CT) scan of the head and neck with intravenous contrast is a common first line imaging. A chest radiography is needed to evaluate for potential lung cancer or metastasis. MRI is useful in evaluating skull-base, skeletal and neurovascular involvement. PET-CT should be considered for patients with advanced regional diseases who have risk of distant metastasis. Q.4: What is the staging of tumor in our case? Ans: T2N1M0. Q.5: What is the treatment option in our case? Ans: Combined modality consisting of surgery and radiotherapy. Lesions invading bone requires wide surgical excision with hemimandibulectomy and neck dissection (commando operation).

175

Tonsils and Adenoids

CASE 13: OBSTRUCTIVE SLEEP APNEA

Presenting complaints: A 45-year-old man presented with snoring during sleep since 3 years. Setting: OPD. History of present illness: History of apneic episodes during sleep. He has also history of day time sleepiness. Muller’s maneuver: A flexible endoscope is passed through the nose and the patient asked to inspire vigorously with nose and mouth completely closed. There was collapse of the soft tissues at the level of tongue and just above the soft palate. Q.1: What is the most possible diagnosis? Ans: Obstructive sleep apnea. Q.2: What is snoring? Ans: It is noisy breathing during sleep. It is estimated that 25% of adult males and 15% of adult females snore. Its prevalence increases with age. Q.3: What are the clinical features of obstructive sleep apnea? Ans: Excessive daytime sleepiness, morning headache, general fatigue, memory loss, irritable, depression, decreased libido, increased risk of road accidents. Q.4: What are the types of sleep apnea? Ans: Three types. Obstructive: There is collapse of the upper airway resulting in cession of airflow. Central: Airways are patent but brain fails to signal the muscles to breathe. Mixed: It is combination of both types (obstructive and central). Q.5: What is Epworth sleepiness scale? Ans: Situation

Score (0 to 3)

Sitting and reading Watching television Sitting inactive in a public place Being a passanger in a car for 1 hour without break Lying down to rest in afternoon when circumstances permit Sitting and talking to somebody Sitting quietly after a lunch without alcohol Sitting in a car while stopped in traffic for a few minutes

0—never dozing off, 1—slight chance of dozing off, 2—moderate chance of dozing off, 3—high chance of dozing. Q.6: What are the consequences of obstructive sleep apnea? Ans: • Congenital heart failure/cor pulmonale • Polycythemia

Manual of Clinical Cases in ENT and Head and Neck Surgery

176

• • • • • • •

Hypertension Atrial and ventricular arrhythemias and left heart failure Angina attack Snoring spouse syndrome Loss of memory Decreased libido Road traffic accident

Q.7: What is the gold standard investigation for OSA? Ans: Polysomnography. It records EEG, ECG, EOG, EMG, pulse oximetry, nasal and oral airflow, sleep position, blood pressure. Q.8: What are the nonsurgical treatments of OSA? Ans: • Weight reduction • Avoidance of alcohol, sedatives, and smoking • Sleeping in lateral position • Intraoral devices like mandibular advancement device and tongue retention device • CPAP or BiPAP or APAP Q.9: What are the surgical treatments of OSA? Ans: Adenotonsillectomy in children (in case of adenotonsillar hypertrophy), septoplasty, turbinate reduction, polypectomy, uvulopalatopharyngoplasty, Advancement pharyngoplasty, tongue base surgery, tongue base radiofrequency reduction, mandibular osteotomy with genioglossus advancement, hyoid myotomy and suspension, maxillomadibular osteotomy and advancement. Tracheostomy is the gold standard treatment. Q.10: What is CPAP? Ans: It is continuous positive airway pressure, useful in treatment of obstructive sleep apnea. It provides pneumatic splint to airway and increases its caliber. The optimum airway pressure for device is to open the airway and is usually kept at 5–20 cm H2O. If CPAP is not tolerated by patient, a BiPAP device is used. It provides positive pressure at two fixed levels, i.e. a higher inspiratory and a lower expiratory pressure. CASE 14: QUINSY

Presenting complaints: A 25-year-old man presents with worsening sore throat. On examination: He has trismus and unilateral enlargement of the right tonsil. Q.1: What is the most possible diagnosis? Ans: Quinsy (right aide) Q.2: What feature of quinsy make it different from acute tonsillitis? Ans: Causative organisms of quinsy involved are mixed flora (GABHS, Staphylococcus, gram-negative and anaerobes) as opposed to Group A beta

Tonsils and Adenoids

177

hemolytic Streptococcus (GABHS) alone in acute tonsillitis. Quinsy is usually unilateral whereas tonsillitis is bilateral. Bilateral quinsy has been reported in infectious mononucleosis. Patients present with odynophagia in quinsy as opposed to sore throat in acute tonsillitis. Patient develops a plummy quality of voice called hot potato voice; has referred otalgia and sometimes trismus all of which are not seen in tonsillitis. On clinical examination, the tonsils are pushed medially and the uvula deviates to the opposite side. CASE 15: TONSILLAR LYMPHOMA

Presenting complaints: A 58-year-old lady is noted to have unilateral tonsillar enlargement. She denies sore throat. Q.1: What is the most possible diagnosis? Ans: Tonsillar malignancy/lymphoma. Q.2: How the diagnosis is confirmed? Ans: Biopsy and histopathological examination. Q.3: Which type of lymphoma is seen in tonsil? Ans: Both Hodgkin and non-Hodgkin lymphomas arise from the tonsil. These are seen in young adults and sometimes in children. Enlarged cervical nodes may coexist. CASE 16: STYALGIA (EAGLE’S SYNDROME)

Presenting complaints: A 30-year-old man presents with pain in the right tonsillar fossa and upper part of the neck in the right side since 3 months. The pain is aggravated on swallowing. Transoral palpation: Feeling of the styloid process in the right tonsillar fossa during transoral palpation. Q.1: What is the possible diagnosis? Ans: Styalgia (Eagle syndrome) Q.2: Which investigation confirms the diagnosis? Ans: X-ray skull (anteroposterior view with open mouth or lateral view of skull) or CT scan with 3D reconstruction. Q.3: What is the normal lengh of the styloid process? Ans: It is 25 mm; if greater than 30 mm, it is considered as elongated. Q.4: What is the etiology for styalgia? Ans: It is due to elongated styloid process or calcification of stylohyoid ligament. Q.5: What is the treatment of the Eagle’s syndrome? Ans: Many patients may have elongated styloid process but remain asymptomatic and do not need treatment. Symptomatic styloid process can be excised by transoral or cervical approach.

178

Manual of Clinical Cases in ENT and Head and Neck Surgery

CASE 17: SECONDARY HEMORRHAGE

Presenting complaints: A 12-year-old boy came to casuality for hemorrhage from the mouth. Past history: He underwent tonsillectomy 5 days back. On examination: Bleeding from the right tonsillar fossa. Q.1: What is the diagnosis? Ans: Secondary hemorrhage following tonsillectomy Q.2: What is the incidence of the secondary hemorrhage? Ans: The incidence is usually 1% and occurs on 5th to 7th day following surgery. It is mainly due to infection. Tonsillar fossa may show the unhealthy slough. Q.3: How to manage the secondary hemorrhage? Ans: • Broad spectrum antibiotics and coagulants • Patient should be admitted in the hospital • Blood transfusion and intravenous fluids to maintain complete hydration • Local pressure at the tonsillar fossa • If local pressure fails, stitching of the anterior and posterior pillars • Ligation of feeding blood vessel or external carotid artery rarely required CASE 18: CHRONIC ADENOTONSILLITIS WITH CHRONIC BILATERAL SECRETORY OTITIS MEDIA

Presenting complaints: An 8-year-old boy presenting with long standing attacks of sore throat, nasal obstruction, nasal discharge and diminished hearing since three month. Exmaination: Chronic adenotonsillitis, bilateral jugulodigastric lymph node enlargement. Bilateral tympanic membrane dull and air fluid levels visible through the tympanic membrane. Tuning fork test: Bilateral conductive hearing loss in both sides. Q.1: What is most possible diagnosis? Ans: Chronic adenotonsillitis with chronic bilateral secretory otitis media. Q.2: What are the cardinal signs of chronic tonsillitis? Ans: • Congestion of anterior pillars • Pus coming from the medial surface of the tonsil after pressing the anterior pillar (Irwin Moore sign) • Enlarged but not tender jugulodigastric nodes Q.3: How to manage this case? Ans: Adenotonsillectomy with bilateral myringotomy and grommet insertion under general anesthesia.

Tonsils and Adenoids

Q.4: What are the sequelae of chronic secretory otitis media? Ans: • Atrophic tympanic membrane and atelectasis of the middle ear. • Tympanosclerosis. • Ossicular necrosis. • Retraction pockets and cholesteatoma. • Cholesterol granuloma.

179

180

Manual of Clinical Cases in ENT and Head and Neck Surgery

26

Tumors of Oral Cavity and Oropharynx CASE 1: RANULA

Presenting complaints: A 10-year-old boy presents with swelling in the floor of oral cavity for last 6 months. His main complaint was bluish cystic transilluminant swelling is seen on right side of frenulum lingue. Setting: Out patient department. History of presenting complaints: He was alright 6 months back. To start with gradual swelling in the floor of the mouth. He had no voice change and no dysphagia. Oral cavity examination: A tense spherical swelling in the floor of the mouth underneath, the tongue, in the midline, measuring about 4 cm in diameter, blue color appearance, smooth cystic and semitransparent (Fig. 26.1).

Fig. 26.1: Smooth and cystic swelling in floor of the mouth

Q.1: What is the most possible diagnosis? Ans: Ranula. Q.2: What is ranula? Ans: It is a large mucos cyst in the floor of the mouth, occurs due to obstruction of sublingual salivary glands duct. It presents as a brilliantly translucent cystic mass. Ranula resembles to the belly of little frog, hence the name ranula given by Hippocrates. 180

Tumors of Oral Cavity and Oropharynx

181

Q.3: What is the plunging ranula? Ans: It is the extension of simple ranula into the neck through the dehiscence into the mylohyoid muscle behind the submandibular gland. Q.4: What is the diagnostic characteristic of ranula? Ans: Bimanula palpation shows a cystic swelling in the floor of the mouth as well as in the submandibular or submental region. Q.5: What are the differential diagnoses of the ranula? Ans: Sublingual dermoid, sebaceous cyst, suprahyoid bursitis, Ludwig’s angina, lipoma and lymphadenitis. Q.6: What is the treatment of ranula? Ans: Treatment is marsupialization. It consists of excision of roof along with overlying mucosa and apposition of cut edges of the mucous membrane of the floor of the mouth with cut edge of the remaining wall of mucosa. Thus bottom of ranula forms the floor of the mouth. CASE 2: CARCINOMA OF THE TONGUE

Presenting complaint: A 65-year-old man presenting an ulcerative growth on the left lateral marging of the tongue (Fig. 26.2) since 3 months.

Fig. 26.2: Growth at lateral border of tongue

Oral cavity examination: Ulcerative growth (2 × 2 cm) with rolled out and everted edge. Indurated base and slough in the floor of the ulcer. Q.1: What is the provisional diagnosis? Ans: Carcinoma of the tongue. Carcinoma of tongue is a common head and neck cancer. High incidence can be found in areas of high consumption of tobacco, betel squid and alcohol. Q.2: What are the premalignant conditions in the oral cavity? Ans: Leukoplakia, erythroplakia, oral submucosal fibrosis.

182

Manual of Clinical Cases in ENT and Head and Neck Surgery

Q.3: What are the different sites of tongue carcinoma? Ans: Lateral border at middle one-third of tongue (50%), posterior one-third tongue (20%), tip of tongue (10%), dorsum towards the midline (10%) and undersurface of the tongue (10%). Q.4: What are the clinical manifestations in tongue carcinoma? Ans: Painless ulcer, excessive salivation, referred otalgia, halitosis, ankyloglossia, defective articulation, trismus and pain due to involvement of lingual nerve. Q.5: What are the treatment options in carcinoma of tongue? Ans: Surgery, radiotherapy. Surgery includes excision of the tumor with 1–2 cm healthy margin plus neck dissection. Postoperative radiotherapy is advised. Q.6: How the carcinomatous lesions of tongue are staged? Ans: The lesion/tumor of the carcinoma tongue is: Tx: Primary tumor cannot be assessed. T0: No evidence of primary tumor. Tis: Carcinoma in situ. T1: Tumor 2 cm in greatest dimension. T2: Tumor >2 cm but 4 cm in greatest dimension. T3: Tumor >4 cm in greatest dimension. T4: Tumor invades adjacent structures (e.g. cortical bone, into deep extrinsic muscle of tongue, maxillary sinus, skin; superficial erosion of bone/tooth socket by gingival primary is not sufficient to classify as T4). Q.7: What is the choice of imaging in carcinoma of tongue? Ans: MRI is the most accurate scanning method to assess the tumor size. MRI has also advantage of taking images at three dimensional planes which helps in the planning of resection. Q.8: What are the levels of lymph nodes in the neck? Ans: • Level IA: Submental nodes • Level IB: Submandibular nodes • Levels II, III and IV: Upper, mid and lower jugulodigastric nodes • Level V: Posterior triangle • Level VI: Below the hyoid in the anterior neck • Level VII: Anterior mediastinum CASE 3: OROPHARYNGEAL MASS

Presenting complaints: A 48-year-old male presents with pain in throat since 2 years, difficulty in opening his mouth since 6 months, ulceration on the left side of the posterior tongue since 6 months and left side neck swelling since 4 months. History of present illness: Pain in the throat is insidious in onset; gradually progressive in intensity. He has mild impairment of tongue.

Tumors of Oral Cavity and Oropharynx

183

History of past illness: No history of diabetes mellitus, hypertension, syphilis, dental problems. Family history: No significant family history. Personal history: Patient is smoker and has been chewing betel nut since 20 years. He is not an alcoholic. Indirect laryngoscopy: An ulcerative growth in lateral part of base of tongue (Fig. 26.3). Larynx and hypopharynx are normal.

Fig. 26.3: An ulcerative growth at base of tongue

Examination of the neck: Level II neck node enlargement with 5 cm in greatest dimension, hard and mobile. Q.1: What are the investigations are needed in above case? Ans: CT scan and MRI to look for site, size and extent of the primary tumor, neck node status, bony invasion to maxilla or mandible and potential spread to parapharyngeal space, supraglottic, posterior pharyngeal wall. Chest X-ray to know the pulmonary metastasis or synchronous primary in the chest. FNAC of the neck node. PET scan for stage III and IV. Biopsy from the primary lesion. Q.2: What are the common sub-sites of the oropharynx where malignancies occur? Ans: Faucial tonsil and pillar (50%), posterior third of tongue (40%), soft palate, posterior pharyngeal wall (10%). Q.3. What is the staging of oropharyngeal malignancy? Ans: T1: 2 cm or less in greatest dimension. T2: More than 2 cm, but not greater than 4 cm in greatest dimension. T3: More than 4 cm or extension to lingual surface of epiglottis. T4a: Invades larynx, extrinsic muscles of tongue, medial pterygoid, hard palate and mandible. T4b: Lateral pterygoid muscle, pterygoid plate, lateral nasopharynx, skull base or encasing carotid artery. Q.4: What is the treatment of base of tongue maliganancy? Ans: Tumors which are radiosensitive such as anaplastic carcinoma, lymphoepithelioma or lymphoma are treated by radiotherapy to the primary and neck nodes. For T1 and

184

Manual of Clinical Cases in ENT and Head and Neck Surgery

T2 squamous cell carcinoma with N0 or N1 neck, block dissection of the neck is done and if neck dissection specimen shows a stage more than N1, postoperative radiotherapy is advised. T3 and T3 lesions need surgical excison with mandibular resection, neck dissection and postoperative radiotherapy. T4 lesion which extend into anterior third of the tongue or vallecula need extensive surgery with total glossectomy and neck dissection. CASE 4: VERRUCOUS CARCINOMA

Presenting complaints: A 72-year-old man presenting with large, fungating and soft papillary lesions in the oral cavity since 6 months. He has been addicted to tobacco chewing for past 50 years. Examination: The lesion has penetrated into the mandible. Neck nodes are not palpable. Biopsy and histopathological examination: It showed benign appearing papillomatosis with hyperkeratosis and acanthosis infiltrating the adjacent tissues. Q.1: What is the most likely diagnosis? Ans: Verrucous carcinoma. CASE 5: ORAL LEUKOPLAKIA

Presenting complaints: A 30-year-old man presenting with whitish patch on the right side buccal mucosa since 3 months. He has habit of cigarette smoking since 10 years. Q.1: What are classical histopathological findings of the verrucous carcinoma? Ans: Histologically verrucous carcinoma is characterized by marked acanthosis, hyperkeratosis often with broad bullous processes showing central column of keratin. There is no cytological evidence of malignancy. Q.2: What is the most probable diagnosis? Ans: Leukoplakia. Q.3: What is leukoplakia? Ans: Most common premalignant condition of oral cavity is leukoplakia. Leukoplakia is a clinical white patch which cannot be characterized clinically or pathologically as any other disease. Q.4: What are the common locations for oral cavity leukoplakia? Ans: Most common sites for oral cavity leukoplakia are buccal mucosa and oral commissures. Q.5: What are the features suggestive change in leukoplakia? Ans: Induarion, speckled or nodular appearance of the lesions. Chances of malignant changes in leukoplakia increase with increase in age of the lesion and age of the patient. All the lesions must be biopsied and send for histopathological examination. Q.6: Which premalignant lesion of the oral cavity has highest risk for oral cancer? Ans: Erythroleukoplakia.

Tumors of Oral Cavity and Oropharynx

185

Q.7: What are the lesions of the oral cavity associated with increased risk of malignancy? Ans: Premalignant conditions • Leukoplakia (most common) • Erythroplakia • Chronic hyperplastic candidiasis Conditions increasing risk of malignancy • Oral submucosal fibrosis • Syphilitic glossitis • Sideropenic dysphagia Conditions with doubtful risk • Oral lichen planus • Discoid lupus erythematosus • Dyskeratosis congenita CASE 6: LINGUAL THYROID

Presenting complaints: A 15-year-old girl presents with asymptomatic red mass over the dorsum of the tongue near the foramen cecum. Examinations: Examination of the oral cavity and neck are within normal limit. Q.1: What is the provisional diagnosis? Ans: Lingual thyroid Q.2: What is lingual thyroid? Ans: Lingual thyroid gland is a rare clinical entity that is due to failure of descent of the thyroid gland anlage early in the course of embryogenesis. Q.3: What are the common clinical symptoms by lingual thyroid? Ans: It may present with symptoms of dysphagia, upper airway obstruction or even hemorrhage at any time from infancy to adulthood. Q.4: What are the investigations done in this case? Ans: • Thyroid function test • Scintigraphy • Ultrasound • CT scan • MRI • Biopsy—biopsy usually shows either normal or embryonal type thyroid tissue. Q.5: What are the treatment options for lingual thyroid? Ans: Treatment is either medical or surgical. Medical treatment usually involves thyroxine therapy or radioiodine (I-131) if symptomas are severe. Before surgical

186

Manual of Clinical Cases in ENT and Head and Neck Surgery

intervention is considered thyroid fuction tests and thyroid scan are recommended as the lingual thyroid may be the only functional thyroid tissue in the body. CASE 7: UNILATERAL TONSILLAR MASS

Presenting complaints: A 35-year-old man presenting with right side bulging of the tonsil medially since 2 months. Examination: The right side tonsillar bulging is pulsating on palpation. Q.1: What is the most possible diagnosis? Ans: Right side internal carotid artery aneurysm. As this right tonsillar mass presents with pulsation, it is aneurysm of internal carotid artery. Q.2: What are structures present in the parapharyngeal space? Ans: The styloid process and the muscles attached to it divide the parapharyngeal space into anterior and posterior compartments. Anterior compartment is related to tonsillar fossa medially and medial pterygoid muscle laterally. Posterior compartment is related to posterior part of lateral pharyngeal wall medially and parotid gland laterally. Through the posterior compartment pass the carotid artery, internal jugular vein, IXth, Xth, XIth, XIIth cranial nerves.

187

Neck Mass

27

Neck Mass CASE 1: THYROGLOSSAL CYST

Presenting complaints: A patient presents to the ENT OPD for a midline neck swelling which moves with deglutition and on protrusion of tongue. Neck examination: A small nodular swelling at the upper part of the neck, below hyoid bone in the midline. It has cystic to firm consistency and the swelling moves up with deglutition and with protrusion of tongue. Q.1: What is the most possible diagnosis? Ans: Thyroglossal cyst (Fig. 27.1). Thyroglossal cyst is a cyst arising from persistent thyroglossal duct.

Fig. 27.1: Thyroglossal cyst

Q.2: What are the sites of the thyroglossal cyst? Ans: Subhyoid (commonest), suprahyoid, over cricoid, over thyroid cartilage, floor of mouth and just below foramen cecum. 187

188

Manual of Clinical Cases in ENT and Head and Neck Surgery

Q.3: What are the preoperative investigations in the thyroglossal cyst? Ans: Ultrasonograpghy, FNAC, radionuclide isotope scanning with technetium99 m/radioiodine (I-131). Q.4: What is the treatment of thyroglossal cyst? Ans: Sistrunk’s operation. It is the excision of the tract from the skin to the base of the tongue including body of the hyoid bone. Q.5: What are the differential diagnoses of thyroglossal cyst? Ans: Dermoid cyst, infected lymph node, lipoma, minor salivary gland tumor, sebaceous cyst, cartilaginous tumor of the thyroid, hypertrophied pyramidal lobe. Q.6: What are the locations of thyroglossal cysts? Ans: This varies with persistent of tract and may found anywhere along the tract • Subhyoid (commonest) • Suprahyoid • Overcricoid • Over thyroid cartilage • Floor of the mouth • Just below the foramen cecum Q.7: Is thyroglossal cyst in the midline? Ans: It usually occupies midline except in the region of thyroid cartilage where the thyroglossal tract is pushed to one side, usually to left. Q.8: What are the clinical features of thyroglossal cyst? Ans: • Age: Commonly seen in between 15 and 30 years but can appear at any age group. • Midline neck swelling which is painless. • Oval swelling with long axis in the lonx axis of the neck. • Overlying skin is normal and free. • Smooth surface with well defined margin. • Fluctuation is positive. • Transillumination test is usually negative because of its thick content due to desquamated epithelial cells or debris of past infection. • It can move side by side but not up and down. • It moves with deglutition (attached to hyoid bone with fibrous tissue). • It moves on protrusion of the tongue (due to connection of base of tongue to the cyst by duct fibrous remnants). • Positive tugging sensation: Catch hold the cyst between the thumb and fore finger and ask the patient to deglutate, the cyst is tugged upwards. • Upward movement on protrusion may be absent if cyst lies below the level of thyroid cartilage.

Neck Mass

189

CASE 2: PLEOMORPHIC SALIVARY GLAND TUMOR

Presenting complaints: A 32-year-old lady is presenting with painless slow growing swelling in the right parotid region since one year. Neck examination: The mass is around 5 cm in diameter, firm, lobulated surface. The mass is not adherent to the masseter muscle. Overlying skin of the mass is free. Facial nerve examination: Normal. Ear examination: Right ear lobule is raised. Others are normal. Neck nodes: No regional lymph node is enlarged. Q.1: What is the provisional diagnosis? Ans: Right side mixed parotid tumor. Q.2: Why pleomorphic adenoma is called mixed tumor? Ans: There are different type of cells in this tumor. They derive from columnar or squamous epithelial cell by mucoid degeneration. The epithelial cells shows various forms, so it is called pleomorphic adenoma. Q.3: What are the common tumors of the parotid gland? Ans: Benign tumors are 80%. Malignant tumors are 20%. Pleomorphic adenoma is the most common benign tumor. Mucoepidermoid tumor is the most common malignant tumor. Q.4: What are the points in the favor of malignant conversion of pleomorphic adenoma? Ans: Sudden rapid growth in the long standing tumor, sudden pain, becoming fixed to skin or muscle or bone, facial nerve paralysis, neck node enlargement, restricted jaw movements, hard and irregular mass. Q.5: How the diagnosis of mixed or pleomorphic adenoma of the parotid is confirmed? Ans: Clinical features, FNAC, ultrasound, MRI and CT scan. Q.6: What is the treatment of mixed tumor? Ans: Superficial parotidectomy. Facial nerve is preserved. If deep lobe is involved, total parotidectomy is needed. Q.7: What are the causes for recurrence after operation of mixed parotid tumor? Ans: Incomplete pseudocapsule, inadequate and incomplete removal, spilage of the tumor mass during surgery and multicentric origin of the tumor. CASE 3: LINGUAL THYROID

Presenting complaints: A 15-year-old boy is examined by local physician, who notes a mass at the back of the young man’s tongue. The biopsy was taken a n d histopathology showed normal thyroid tissue. The diagnosis is: Lingual thyroid.

Manual of Clinical Cases in ENT and Head and Neck Surgery

190

Lingual thyroid is the presence of thyroid tissue at the junction of anterior 2/3 and posterior 1/3 of tongue. Failure of the thyroglossal tract to descend in neck results in lingual thyroid. It presents as a painless swelling at the base of tongue. CASE 4: CA TONGUE

Presenting complaints: A 60-year-old male presents with a growth over the lateral border of tongue (Fig. 27.2) and clinically negative neck nodes since 2 months. Setting: OPD. Biopsy: A biopsy from the growth was taken for histopathology. It was reported as squamous cell carcinoma.

Fig. 27.2: An irregular ulcerative growth in left lateral border of tongue

Q.1: The management of neck in this patient will be? Ans: Supraomohyoid neck dissection. This patient need selective neck dissection (supraomohyoid neck dissection) which removes levels I to IV neck nodes. Q.2: What are the divisions of neck nodes according to levels? Ans: Level I: Submental (IA), submental (IB); Level II: Upper jugular; Level III: Mid jugular; Level IV: Lower jugular; Level V: Posterior triangle group (spinal accessory and transverse cervical chains); Level VI: Prelaryngeal, pretracheal, paratracheal; Level VII: Nodes of the upper mediastinum. Neck node staging Nx: Regional lymph node cannot be assessed. N0: No regional lymph node metastasis. N1: Metastasis in single ipsilateral node, 3 cm in greatest dimension. N2a: Metastasis in single ipsilateral node, >3 cm but 6 cm. N2b: Metastasis in multiple ipsilateral nodes, all 6 cm. N2c: Metastasis in bilateral or contralateral nodes, all 6 cm. N3: Metastasis in lymph node >6 cm in greatest dimension.

Neck Mass

191

Q.3: What is selective neck dissection? Ans: It consists of preservation of one or more lymph node groups and all the three nonlymphatic structures, i.e. spinal accessory, sternocleidomastoid muscle and internal jugular vein. Supraomohyoid neck dissection includes removal of levels I to III, usually done in cancer of oral cavity with N0. Lateral: Removes levels II to IV. Posterolateral: Removes levels II to V with postauricular and occipital nodes. Anterior compartment: Removes nodes at level VI, i.e pretracheal, paratracheal and prelaryngeal. CASE 5: OCCULT PRIMARY WITH METASTATIC NECK NODE

Presenting complaints: A 67-year-old man presented with a large neck mass in the right side since 8 months. Examination of the neck: The mass was painless, progressively increase in size, hard and fixed to the deeper structures with size 7 × 6 cm. Indirect laryngoscopy: Larynx, hypopharynx and oropharynx were normal. Diagnostic nasal endoscopy: Bilateral nasal cavity and nasopharynx were within normal limits. FNAC: Metatstaic neck node of squamous cell carcinoma. Q.1: What is the provisional diagnosis? Ans: Occult primary with neck node metastasis. The incidence of occult primary with metastatic neck node is seen in about 5% cases (3–9%). Incidence of cervical metastasis: About 80% of the neck mass is cervical metastases, if thyroid is ecluded. After 5th decades of life; about 90% of the neck mass is malignant. Q.2: What are the different sites of primary in cases of occult primary? Ans: Nasopharynx, base of tongue, pyriform fossa between anterior pillar and lingual attachment of tonsil, posterior pharyngeal wall, postcricoid, subglottic and below the clavicle (esophagus, uterus, breast and testis). Q.3: What are the different investigations needed to find out occult primary? Ans: Flexible nasopharyngolaryngoscopy, flexible bronchoscopy, esophagoscopy, CT scan of the neck with contrast, MRI. Triple endoscopy: Direct laryngoscopy, bronchoscopy and esophagoscopy. Blind biopsy: It is successful in diagnosis only in 3–9% from the fossa of Rosenmuller, base of tongue and pyriform fossa. Q.4: What is the treatment of metastasis lymph node with occult primary? Ans: The treatment depends on level of lymph node (site), size of the lymph node and histopathology report. Neck dissection plus radiotherapy are useful for metastatic neck nodes. CASE 6: NECK NODE METASTASES

Presenting complaints: A 60-year-old man presents with a mass of lymph nodes in the upper cervical region since 2 months. He is a smoker and also drinks two to three times per week.

192

Manual of Clinical Cases in ENT and Head and Neck Surgery

Setting: Outpatient department Examination: Complete physical examination of the upper aerodigestive tract did not reveal any primary tumor. Q.1: What is the next diagnostic test? Ans: As this patient is in cancer age (old age) group with risk factors of smoking and drinking, metastases from an occult primary are highly suspected. FNAC is very sensitive and can detect squamous cell carcinoma. Q.2: If FNAC reveal squamous cell carcinoma, what is the next plan? Ans: Once metastases are proved, work up for occult primary should continue. CASE 7: GLOBUS HYSTERICUS

Presenting complaints: A 40-year-old female presents with complaints of lump in the throat not interfering with swallowing since one six months. Setting: OPD. Indirect laryngoscopy: Normal. Examination of neck: Normal. Q.1: Most probable diagnosis is? Ans: Globus hystericus. Globus hystericus is characterized by a foreign body sensation in throat usually in midline between thyroid cartilage and suprasternal notch. It is common in females. The diagnosis is established by exclusion. The diagnosis is established when radiological investigations and endoscopy are normal. Treatment is reassurance. Q.2: What is the treatment of globus hystericus/pharyngeus? Ans: Reassurance to the patient when no cause has been found. CASE 8: PAROTID TUMOR

Presenting complaints: A 32-year-old female presented with a progressively increasing lump in the left side parotid region (Fig. 27.3). Examination: On oral examination, the left side tonsil was pushed medially. Investigation: FNAC showed it to be pleomorphic adenoma. Q.1: What is the appropriate treatment? Ans: For parotid tumors which arise from the superficial lobe need superficial parotidectomy with preservation of CN VII. If the tumor extends into the deep lobe of the parotid, a total parotidectomy with CN VII preservation is performed.

Fig. 27.3: Left parotid tumor

193

Neck Mass

Q.2: What is superficial parotidectomy? Ans: The aim of the superficial parotidectomy (Patey’s operation) is to remove the tumor with a cuff of normal surrounding tissue. The term superficial parotidectomy is used as not all branches of the facial nerve are dissected particularly if a tumor lies in the lower pole of the parotid gland. Q.3: What is the name of incision used for superficial parotidectomy? Ans: Lazy ‘S’ preauricular mastoid cervical incision. Q.4: Why open surgical biopsy is contraindicated in salivary gland tumors? Ans: Because open surgical biopsy seeds the tumor to the surrounding tissue. Q.5: What are causes of bilateral parotid gland enlargement? Ans: • Sjögren’s disease • Sarcoidosis • Amyloidosis CASE 9: SIALOLITHIASIS

Presenting symtpoms: A 30-year-old lady came with complaint of pain and swelling in the right submandibular region (Fig. 27.4) for 3 months. The swelling is more after taking meals. Clinical examination: A swelling in the right side submandibular region along with swelling in the floor of the mouth in the right side felt by bimanual palpation. Q.1: What is the most possible diagnosis? Ans: Right side submandibular gland sialolithiasis. Q.2: What are the causes for the sialolithiasis? Ans: Exact etiology is uncertain. Salivary stasis and ductal inflammation and injury are important contributing factors for sialolithiasis. Approximately 80 to 90% of calculi develop in Wharton’s duct of submandibular gland. Stenson’s duct of parotid constitutes 10 to 20% and sublingual duct only 1%.

Fig. 27.4: Right submandibular gland sialolithiasis

Q.3: Why the calculi are more common in Wharton’s duct of submandibular gland? Ans: Wharton’s duct is longer and has larger caliber. It is angulated against gravity as it courses around the mylohyoid muscle. Submandibular secretions are more viscous and have higher calcium and phosphorous concentration. Q.4: What are the investigations done in sialolithiasis? Ans: • Plain radiographs (intraoral or occlusal view): It identifies radiopaque stones and in the submandibular gland 80% of stones are radiopaque.

194

Manual of Clinical Cases in ENT and Head and Neck Surgery

• Ultrasound: It detects 90% of the stones if they are more than 2 mm. • CT scan with fine cuts is very accurate at detecting salivary stones. • Digital subtraction sialography: It lessens the interference of surrounding bony structures. It can detect 95–100% of radiolucent stones. • MR sialography of the submandibular duct with evoked salivation is superior to USG and has accuracy similar to digital subtraction sialography. Q.5: What are the non-surgical treatments of the sialolithiasis? Ans: • Sialogogues • Local heat • Hydration • Massaging of the involved gland • Antibiotic coverage in case of infection • Manually milking out: Submandibular stones nearer the duct orifice may be manually milked out through the duct opening. Q.6: What are surgical management of the submandibular gland stone? Ans: • Incision of duct (open sialolithotomy) • Sialoadenectomy/excision of the submandibular gland. CASE 10: LIPOMA

Presenting complaints: A 32-year-old man presenting a soft, non-tender, l o b u l a t e d freely mobile swelling at the submental region of the neck since 3 months. Examination: Skin over the swelling can be lifted up. Edges of the swelling slip under the finger. Q.1: What is the possible diagnosis? Ans: Lipoma. It will be confirmed by cytological examination (FNAC/biopsy) Q.2: What are the midline swellings of the neck? Ans: From above downwards, the midline swellings are Ludwig’s angina, submental lymph node enlargement, sublingual dermoid, lipoma in submental area, thyroglossal cyst, subhyoid bursitis, goiter of thyroid isthmus, lipoma in the suprasternal space of burns and retrosternal goiter. Q.3: What is the characteristic of thyroglossal cyst during examination? Ans: It moves up on protrusion of the tongue. Q.4: What are the pulsatile neck swelling? Ans: Carotid body tumor, subclavian artery aneurysm, carotid artery aneurysm, lymph node overlying the carotid artery and rarely primarly toxic goiter.

Neck Mass

195

Q.5: What is cystic hygroma? Ans: It is seen in lower third of posterior triangle of the neck, which is round in shape with smooth or lobulated surface. Transillumination and cough impulse are positive. Overlying skin can be lifted up. CASE 11: LARYNGOCELE

Presenting compalaints: A 45-year-old man present horsensess of voice and with swelling at right upper part of the neck since 3 months. This swelling is increasing with cough or during performing Valsalva. X-ray neck: Shows an air filled sac at the upper part of the neck (Fig. 27.5).

Fig. 27.5: X-ray neck showing laryngocele

Q.1: What is the most possible diagnosis? Ans: Laryngocele Q.2: What is laryngocele? Ans: It is an air filled cystic swelling due to dilatation of the saccule of the larynx. Q.3: What are the types of the laryngocele? Ans: A Laryngocele may be: • Internal: It is confined within the larynx and presents as distension of false cord and aryepiglottic fold. • External: Here the distended saccule herniates through the thyrohyoid membrane and presents a swelling in the neck. • Combined or mixed: Here, both internal and external components are seen. Q.4: What are causes for development of the laryngocele? Ans: A laryngocele is supposed to arise from raised transglottic air pressure as in glass blowers, trumpet players or weight lifters. Q.5: What are the clinical presentations of the patients with laryngocele? Ans: A laryngocele presents with hoarseness of voice, cough and breathing difficulty (if large size laryngocele). An external laryngocele presents as a reducible swelling in the neck which increases in size on coughing or doing Valsalva.

196

Manual of Clinical Cases in ENT and Head and Neck Surgery

Q.6: What are the investigations performed in laryngocele patient? Ans: • Laryngoscopy • X-ray soft tissue of neck (AP and lateral view) with Valsalva • CT scan of the neck Q.7: What are the treatment options in laryngocele? Ans: Treatment is surgical exicison through an external neck incision. Marsupialisation of an internal laryngocele can be done by laryngoscopy but there are chance of recurrence. CASE 12: THYROID NODULE

Presenting complaints: A 22-year-old girl presenting with a swelling on the right side thyroid lobe since 5 months. The swelling moves up during swallowing. Examination of the neck: A palpable swelling in the right tyroid lobe with size approximately 2 × 2 cm. It moves up during swallowing. Q.1: What is the most possible diagnosis? Ans: Thyroid nodule. Q.2: What are the prevalence of the thyroid nodule? Ans: Clinically palpable nodules are seen in 4 to 7% of the population. Incidence increases as age advances; 5 to 10% of all the thyroid nodules are malignant. Q.3: What is Pemberton sign? Ans: This test is done in nodules extending retrosternally. Raising the arms above the head causes signs of thoracic inlet obstruction with engorgement of neck veins. Q.4: What are the risk factors for thyroid nodule being malignant? Ans: • Age less tha 20 years in males and more than 50 years • Male sex • History of radiation to the neck in the past • Family history of thyroid cancer • Size of the thyroid nodule more than 4 cm • Rapid growth • Pain in thyroid nodule • Fixed and hard lesion • Lymhadenopathy • Recurrent laryngeal nerve paralysis • History of stridor and dysphagia (compressive symptoms on trachea and esophagus) Q.5: What are the investigations required in thyroid nodule? Ans: • Thyroid function test • Ultrasound of the neck

Neck Mass

197

• CT/MRI neck and upper chest • FNAC • Thyroid scan—chances of malignancy in hot nodules is 4% while it is 10–15% in cold nodules. CASE 13: UNKNOWN PRIMARY

Presenting complaints: A 60-year-old man with a neck mass suspicious malignancy but with no clue from history and physical examination about the primary site. Q.1: What should be the next step in the management? Ans: The above case has presented with a cervical mass could be secondary with unknown primary. FNAC is a simple procedure and in most cases will establish the malignant nature or some other lesions. If FNAC is negative it should be repeated or a core needle biopsy is done. Open biopsy is done when FNAC is negative or FNAC shows probable lymphoma or an epithelial malignancy of uncertain type. CASE 14: HIV PATIENT WITH NECK MASS

Presenting complaints: A 30-year-old HIV infected patient presents with a nontender parotid gland swelling. Q.1: The most likely diagnosis is? Ans: Benign lymphoepithelial cysts may develop in the parotid glands on one or both sides. They arise from intraparotid lymph nodes. Diagnosis is confirmed on CT scan and fine needle aspiration cytology. Q.2: What is Kaposi’s sarcoma? Ans: Kaposi’s sarcoma is the most common malignancy seen in AIDS patients. It affecects skin or the mucous membrane. Association with herpes virus 8 has has been seen in the causation of Kaposi’s sarcoma in AIDS patients. Non-Hodgkin’s lymphoma is the second most common malignancy seen in HIV patients. Q.3: What is hairy leukoplakia in AIDS? Ans: Hairy leukoplakia in AIDS patients is caused by Epstein-Barr virus. It presents as white patches on the lateral border of tongue and may be an early manifestations of disease. CASE 15: THYROGLOSSAL FISTULA

Presenting complaints: A 20-year-old boy presents with a small persistent sinus below the hyoid level with glairy mucoid discharge. Examination: Fistulous opening has hood of skin above the fistula (semilunar fold of skin with concavity downward as superior border of the fistula). The opening moves up on deglutition and also with protrusion of the tongue. Q.1: What is the most possible diagnosis? Ans: Thyroglossal fistula

198

Manual of Clinical Cases in ENT and Head and Neck Surgery

Q.2: What is thyroglossal fistula? Ans: It is acquired fistula which results either secondary to burst opened of infected thyroglossal cyst following incomplete removal of thyroglossal cyst. Q.3: What is the lining of the thyroglossal fistula? Ans: It is lined by columnar epithelium. Q.4: What is hood sign? Ans: T h e s k i n a b o v e t h e f i s t u l o u s o p e n i n g i s p u l l e d u p w a r d s b y t h e thyroglossal tract and this gives rise to puckering of the skin resembling a hood of a snake. Q.5: What is the treatment? Ans: Excision by transverse elliptical incision with excision of centrlal part of hyoid bone with communicating tract. This is called Sistrunk operation. CASE 16: CYSTIC HYGROMA

Presenting complaints: A 5-year-old child presents with multiloculated cystic swelling in the right side posterior triangle of the neck. On examination: The swelling is lobulated, brilliantly transparent and partially compressible. Q.1: What is the most possible diagnosis? Ans: Cystic hygroma. Q.2: What is cystic hygroma? Ans: It is collection of lymphatic sacs which is lined by an endothelial layer. It contains clear colorless lymph. It is a cavernous lymphangioma or cycstic lymphangioma with wide ramifications. It is congenital malformation (hamartoma) affecting lymphatic channel. Q.3: What are the investigations required? Ans: X-ray chest: To exclude mediastinal extension. CT scan or MRI: To assess the exact extent of the lesion and its relation to vital structures. Q.4: What is the treatment of the cystic hygroma? Ans: • It can be observed for 1 to 2 years for spontaneous regression if not causing life threatening problem or not rapidly enlarging. • Preoperative sclerosing solution injection (hypertonic saline or boiling water) into the cysts. It reduce the size, fibrosing the cyst wall and easy to operate. • Excision of the cyst. CASE 17: LIPOMA

Presenting complaints: A 25-year-old lady presenting slow growing painless swelling in left submandibular area since 6 months.

Neck Mass

199

On examination: The swelling is smooth, lobulated, soft in consistency and mobile and overlying skin free (Fig. 27.6).

Fig. 27.6: Lipoma in the left submandibular region

Q.1: What is the provisional diagnosis? Ans: Lipoma. Q.2: What are the points in favor of lipoma? Ans: • Slow growing swelling • Painless swelling • Smooth and lobulated surface • Soft and mobile • Pseudofluctuation • Margin slips under examining fingers • Covering skin is free Q.3: What is Dercum’s disease? Ans: Diffuse or nodular painful deposits of fat in the neck, axilla, hips and thigh is called Dercum disease or adiposis dolorosa. It is associated with neurolipoma. Q.4: What is the treatment of lipoma? Ans: Excision, liposuction. Q.5: What are the peculiarities of the lipoma? • • • • •

These are usually not seen in children This is not a neoplasm but a hamartoma or excessive accumulation Universal tumor can occur anywhere Commonly solitary, may be multiple Spread is finger like process in subcutaneous spaces from lipoma.

200

Manual of Clinical Cases in ENT and Head and Neck Surgery

CASE 18: PAPILLARY CARCINOMA OF THYROID

Presenting complaints: A 30-year-old female presenting with soft to firm swelling of throid gland since one year. Ultrasound of the neck: Showed benign lesion with cystic component in the right lobe of the thyroid. Fine needle aspiration cytology: Reported follicular adenoma. She underwent right hemithyroidectomy and biopsy report was papillary carcinoma. Q.1: What is the next plan? Ans: Completion thyroidectomy. This is followed by radioactive iodine ablation for residual thyroid tissue and metastases. Q.2: Consistency of different types thyroid nodules? Ans: • Colloid nodules—doughy • Cysts—cystic filled with fluid • Papillary and medullary cancers—hard rubber • Anaplastic cancer—hard, fixed and craggy • Lymphomas—diffuse swelling Q.3: What are the risk factors for malignant tumors of thyroid? Ans: • Age of patients under 14 or over 65 years of age • Sex—male • Enlarging of the thyroid gland in spite of suppressive doses of thyroxine • Past history of isonizing radition or thyroid cancer • Family history of thyroid cancer Q.4: What is subtotal thyroidectomy? Ans: Partial resection of bulk of each thyroid lobe and leaving up to 8 gm of normal tissue in each lobe. Q.5: What is Kocher’s test? Ans: Slight pressure on thyroid lobes produce stridor in cases of scabbard trachea. Scabbard trachea—when the trachea is displaced and compressed from both sides by thyroid mass. CASE 19: HEMANGIOMA

Presenting complaints: A 15-day-old female infant presented with a unilateral parotid swelling with bluish overlying skin. Swelling increases when the child cries. No other abnormality was detected. Q.1: The most likely diagnosis is? Ans: Hemangioma of the parotid gland.

Neck Mass

201

Q.2: What is hemangioma of the parotid gland? Ans: Hemangioma is the most common benign neoplasm of the parotid gland seen at birth. The overlying skin shows bluish discolouration. It also increases in size on crying or straining. Hemangioma of parotid involutes spontaneously by 5 years of age and the treatment is only observation. Surgical excision may be required of they fail to involute. Q.3: What is the amount of saliva secreted in an adult in 24 hours? Ans: 1000–1500 ml, i.e. on an average 1 ml/min. Q.4: Major amount of saliva, when salivary glands are not stimulated, is contributed by? Ans: When salivary glands are not stimulated, major amount of saliva is contributed by submandibular glands (parotid 26%, submandibular 69% and sublingual 5%). However, when stimulated, major contribution is by parotid glands (about 66%) and rest by submandibular and sublingual glands. Minor salivary glands, independent of stimulation, contribute 7–8% of total saliva.

202

Manual of Clinical Cases in ENT and Head and Neck Surgery

28

Vocal Cord Paralysis CASE 1: LEFT RECURRENT LARYNGEAL NERVE PALSY

Presenting complaints: Doctor, I have hoarsed voice since 2 months. Setting: Out patient department of otorhinolaryngology History of presenting illness: A 38-year-old male presented with hoarseness of voice since two months. He has no dysphagia and breathing difficulty. General physical examination: The patient is well oriented with time, place and person. Pulse rate—90/minute, BP—110/80 mmHg. He has fever of 101°F. Examination of oral cavity and oropharynx: Right tonsil is congested and swollen. Uvula is pushed to the left side. Indirect laryngoscopy: Left vocal cord paralysis. Examination of the neck: Normal. No swelling in the neck Examiantion of the ear, nose, paranasal sinuses and nasopharynx: Within normal limit. Q.1: Why left recurrent laryngeal nerve commonly injured? Ans: Left recurrent laryngeal nerve has a longer course than right one. It passes inferior and posterior to the aortic arch and reverses its course to continue into the neck. Q.2: How do you classify nerve paralysis? Ans: Sunderland’s classification: • First degree—neuropraxia. • Second degree—Wallerian degeneration distal to injury axonotmesis and no synkinesis occurs as endoneural sheath is intact. • Third degree—endoneural scarring and misdirected regeneration. • Fourth degree—scarring can block regeneration. • Fifth degree—transection (neuromesis). Q.3: What are the risk factors (non-laryngeal) causing voice changes? Ans: • Hypothyroidism • GERD 202

Vocal Cord Paralysis

203

• Tobacco/smoking • Pulmonary pathology like asthma/emphysema • Deficit of laryngeal lubrications due to drugs/alcohol. Q. 4: What are the investigations done in vocal cord paralysis? Ans: • CT scan of the neck and chest • Upper GI endoscopy/bronchoscopy • Electromyography Q.5: What are the causes of the left recurrent nerve paralysis? Ans: Causes in the neck • Accidental trauma • Thyroid diseases (benign or malignant) • Thyroid surgery • Malignancy of cervical esophagus • Cervical lymphadenopathy Causes in mediastinum • Bronchogenic cancer • Carcinoma of thoracic esophagus • Aortic aneurysm • Mediastinal lymphadenopathy • Enlarged left auricle • Intrathoracic surgery • Idiopathic Q.6: What is Semon’s law? Ans: In all progressive organic lesions of motor laryngeal nerve, the abductors of the vocal cords are paralysed much earlier than adductors. The reason for abductors to be paralysed earlier are: • Adductors are first to develop so last to go. • Bulk of abductor muscle is more, so more toxin reaches to the abductors. • Chronaxy (response to the electric stimulation) is more in abductors. Exceptions of Semon’s law are: • TB of the larynx where cricothyroid (partial adductor) is first muscle to get involved causing bowing of vocal cords. • Malignancy. CASE 2: UNILATERAL VOCAL CORD PARALYSIS

Presenting complaints: Doctor, I have hoarsed voice since 3 months after an attack of diptheria.

204

Manual of Clinical Cases in ENT and Head and Neck Surgery

Setting: Out patient department of otorhinolaryngology History of presenting illness: An 11-year-old boy presented hoarseness of voice since 3 months. Hoarseness of voice developed after attack of diphtheria. General physical examination: The patient is well oriented with time, place and person. Pulse rate—90/minute, BP—110/80 mmHg. He has fever of 101°F. Examination of oral cavity and oropharynx: Right tonsil is congested and swollen. Uvula is pushed to the left side. Indirect laryngoscopy: Right cord paralysis. Examination of the neck: Normal. No swelling in the neck Examiantion of the ear, nose, paranasal sinuses and nasopharynx: Within normal limit. Q.1: What is the treatment? Ans: Unilateral vocal cord paralysis due to neuritis (as in diphtheria) does not need any treatment as it recovers spontaneously. The characteristic features of diphtheria neuropathy is that it recovers completely. Type I laryngoplasty can be done, if compensation does not take place. Type I laryngoplasty with arytenoids adduction is done, if posterior glottis is also incompetent. Q.2: What are the causes of the right side vocal cord paralysis? Ans: Neck trauma, benign or malignant thyroid disease, thyroid surgery, esophageal cancer, cervical lymphadenopathy, aneurysm of subclavian artery, carcinoma apex right lung, TB of cervical pleura and idiopathic. Q.3: What are the causes of left vocal cord paralysis? Ans: Neck causes: Accidental trauma, thyroid diseases, thyroid surgery, cervical esophagus carcinoma, cervical lymphadenopathy. Mediastinal causes: Bronchogenic carcinoma, malignancy of thoracic esophagus, mediastinal lymphadenopathy, enlarged left atrium, intrathoracic surgery and idipathic. Q.4: What are the differences between paralysed vocal cord and fixed vocal cord? Ans: Paralysed vocal cord

Fixed vocal cord

Slight bowing of the cord

No bowing of the cord

Cord appears toneless

Appears shortened

Flickering of cord during phonation

No flickering

Arytenoid cartilage deviated medially

No medial deviation

Probing of arytenoid shows movement of cord

No movement of cord

CASE 3: INJURY TO SUPERIOR LARYNGEAL NERVE

Presenting complaints: Doctor, I have weak voice since 1 month after a surgery in the neck. Setting: Out patient department of otorhinolaryngology

Vocal Cord Paralysis

205

History of presenting illness: A 41-year-old man presented with weakness of voice since 1 month. He has no hoarseness of voice. He had undergone right side thyroidectomy one month back for colloid goiter. General physical examination: The patient is well oriented with time, place and person. Pulse rate—70/minute, BP—110/80 mmHg. Examination of oral cavity and oropharynx: Within normal limit. Indirect laryngoscopy: Normal. Examination of the neck: Normal. No swelling in the neck Examiantion of the ear, nose, paranasal sinuses and nasopharynx: Within normal limit. Q.1: What is the cause of voice weakness? It may be due to injury of external laryngeal nerve (branch of superior laryngeal nerve) which is tensor of vocal cord. Q.2: What are the clinical manifestations of superior laryngeal nerve paralysis? Ans: • Voice is weak and pitch cannot be raised. • Anesthesia of larynx above the level of vocal cords causing occasional aspiration. Q.3: What is the sensory supply of the larynx? Ans: • The internal branch of superior laryngeal nerve supply the larynx above the vocal cords. • Recurrent laryngeal nerve supply the larynx below the level of vocal cords. CASE 4: ADDUCTOR PARALYSIS

Presenting complaints: A 16-year-old girl attended the clinic with sudden aphonia since 1 day. Indirect laryngoscopy: Bilateral adductor paralysis of vocal folds (both vocal folds are seen in abducted position and fail to adduct on phonation). However, adduction of vocal cords are seen on coughing. Q.1: What is the possible cause? Ans: Functional aphonia (hysterical aphonia). Q.2: What is functional aphonia? Ans: It is a functional disorder mostly seen in emotionally labile females in the age group of 15–30 years. Aphonia is often sudden and unaccompanied by other laryngeal symptoms. Q.3: What are the laryngeal findings in functional aphonia? Ans: Bilateral vocal folds are in abducted position and fail to adduct on phonation. However, the adduction of vocal folds can be seen on coughing, indicating normal adductor function. Even though patient is aphonic, sound of cough is good.

206

Manual of Clinical Cases in ENT and Head and Neck Surgery

Q.4: What is the treatment for functional aphonia? Ans: Treatment given into reassure the patient of normal laryngeal function and psychotherapy. CASE 5: INJURY TO RECURRENT LARYNGEAL NERVE

Presenting complaints: A 30-year-old lady underwent right side thyroidectomy for euthyroid goiter with cystic degeneration. After surgery she developed hoarseness of voice. Indirect laryngoscopy: Right vocal fold assumes paramedian position. Q.1: Which muscle is responsible for paramedian position of the vocal fold? Ans: Cricothyroid muscle. This is because injury of the recurrent laryngeal nerve injury. Q.2: What is the nerve supply of the muscles of the larynx? Ans: All the muscles of the larynx are supplied by recurrent laryngeal nerve except cricothyroid muscle. The cricothyroid muscle receives its innervation from the external laryngeal nerve, a branch of superior laryngeal nerve. Q.3: What are adductors of the vocal folds? Ans: Lateral cricoarytenoid, interarytenoid (transverse arytenoid) and thyroarytenoid (external part). Q.4: What are the tensors of the vocal folds? Ans: Cricothyroid, vocalis (internal part of thyroarytenoid). Q.5: Which is the abductor of the vocal fold (safety muscle of the larynx)? Ans: Posterior cricoarytenoid. CASE 6: BILATERAL ABDUCTOR PARALYSIS

Presenting complaints: A patient presented with stridor and dyspnea which he developed after an attack of upper respiratory tract infection. Fiberoptic nasopharyngolaryngoscopy: Around 3 mm glottic opening. Q.1: What is the possible diagnosis? Ans: Glottic diameter of 3 mm indicates that patient is having laryngeal paralysis and exacerbated by URTI. Q.2: What are the clinical features in bilateral abductor paralysis? Ans: Both vocal cords lie in median or paramedian position and the inadequate airway results in dyspnea and stridor but the voice is good. Dyspnea and stridor becomes worse on exertion or during an attack of acut laryngitis. Q.3: What are the treatment options in this case? Ans: • Tracheostomy • Arytenoidectomy

Vocal Cord Paralysis

207

• Cordotomy • Thyroplasty type II • Vocal cord lateralization through endoscope Q.4: For a quiet respiration the glottic diameter should be? Ans: Approximately 14 mm wide. Q.5: What is Kashima operation? Ans: It is transverse cordotomy. Soft tissue at the junction of membranous cord and vocal process of arytenoid is excised laterally with laser. This provides good airway.

208

Manual of Clinical Cases in ENT and Head and Neck Surgery

29

Nasopharynx CASE 1: NASOPHARYNGEAL ANGIOFIBROMA

Presenting complaints: Doctor, I have recurrent nasal bleeding since 8 months and nasal obstruction since 3 months. Setting: Out patient department of otorhinolaryngology History of presenting illness: A 14-year-old boy presented repeated epistaxis since 8 months. Hoarseness of voice developed after attack of diphtheria. General physical examination: The patient is well oriented with time, place and person. Pulse rate—90/minute, BP—110/80 mm Hg. He has fever of 101°F. Examination of oral cavity and oropharynx: Right tonsil is congested and swollen. Uvula is pushed to the left side. Posterior rhinoscopy: A pinkish mass in the nasopharynx. Examination of the neck: Normal. No swelling in the neck Examiantion of the ear: Within normal limit. Q.1: What is the most possible diagnosis? Ans: Juvenile nasopharyngeal angiofibroma (Fig. 29.1).

Fig. 29.1: Endoscopic view of JNA 208

Nasopharynx

209

Q.2: What is the role of biopsy in JNA? Ans: Biopsy is contraindicated due to the fear of torrential bleeding. Q.3: What is the investigation of choice in JNA? Ans: Contrast enhanced CT scan of the nose and sinus. Q.4: A. B. C. D. Ans:

True about JNA It is a malignant tumor Increased incidence in females Surgery is the treatment of choice Hormones are not used in treatment C

Explanations: Treatment of choice of JNA is surgery. It is a benign tumor though it is locally invasive. It is exclusive to adolescent males. Due to its hormonal predilection, estrogen (diethyl stilbesterol) is used in the treatment. Q.5: What are the radiological signs in JNA? Ans: Lateral radiograpgh skull shows anterior bowing of posterior wall of maxilla due to tumor enlarging in the pterygopalatine fossa—this is known as antral sign or Holman-Miller sign. MRI is complementary to CT and characteristic finding is a “Bag of worms appearance”. Angiography may reveal typical blush in the nasopharynx. Q.6: What are the surgical approaches in JNA? • • • • • •

Treatment of choice in JNA is surgery. Surgical approach of choice is mid-facial degloving approach to the nasopharynx. Transpalatal approach (Wilson’s) Lateral rhinotomy with medial maxillectomy Craniofacial resection for intracranial extension Endoscopic endonasal excision coupled with laser is gaining popularity nowadays.

Q.7: What are the non-surgical treatment options in JNA? Ans: • Flutamide is a non-steroidal androgen blocker has been found to be helpful in inducing tumor shrinkage. • Previously an estrogen was used but due to their adverse effect like feminization, is now undesirable in male. It may be indicated to reduce vascularity preoperatively. • Radiotherapy is reserved for unresectable, tumor with intracranial extension or recurrence cases. Q.8: What is the role of digital subtraction angiography and embolization in JNA? Ans: • It is done preoperatively especially in large tumors to locate the feeding vessels. • It reduces the vascularity and thereby less operative bleeding.

210

Manual of Clinical Cases in ENT and Head and Neck Surgery

• The main feeding vessel in most of the cases in internal maxillary artery. • Paradoxically recurrence rates seem to be increased by preoperative embolization possibly due to the difficulty to define the actual extent following the shrinkage. CASE 2: NASOPHARYNGEAL CARCINOMA

Presenting complaints: Doctor, I have blood stained nasal discharge since 2 months intermittently and also mild nasal obstruction. Setting: OPD History of present illness: A 55-year-old lady presented with history of intermittent blood stained left nasal discharge. She was treated with antibiotics and nasal drops by her primary care physician, but the symptoms continued. She denies any otalgia, otorrhea, facial nerve weakness, vertigo or tinnitus. She is house wife and exposure history to passive smoking since 20 years. She denies voice change, dysphagia, vision change, sensation change, aspiration or weight loss. Her medical history is insignificant other than newly diagnosed diabetes mellitus. General physical examination: The patient is well oriented with time, place and person. Pulse rate—76/minute, BP—122/80 mmHg. Examination of oral cavity and oropharynx: Within normal limit. Indirect laryngoscopy: Normal. Posterior rhinoscopy: A hyperemic lobulated mass with well defined borders is seen in the left fossa of Rosenmüller measuring 2.5 × 2.5 cm in diameter (Fig. 29.2). Posterior end of septum, choana, roof of nasopharynx and right fossa of Rosenmüller are normal.

Fig. 29.2: An ulcerative growth at nasopharynx

Examination of the neck: A swelling in the left side of the neck. It is 3 × 2 cm size, nontender, firm, mobile lymph node in the left upper jugular level. Examiantion of the ear, nose, paranasal sinuses: Within normal limit

Nasopharynx

211

Examinations of cranial nerves: Normal Examination of eye: Normal movement of extraocular muscles. Q.1: What is the most probable diagnosis? Ans: Nasopharyngeal carcinoma. The NPC has bi-modal age occurrence with two peaks, the first at 15–25 years and the second peak at 55–65 years of age. Nearly 75% of patients of NPC present with painless cervical lymphadenopathy of mainly the upper deep cervical lymph node. The nasopharynx drains into the retropharyngeal lymph node of Rouviere but this is inaccessible to inspection as well palpation. From here second echelon nodes are the jugulodigastric and posterior triangle lymph nodes which are the first palpable nodes. Other clinical features are nasal obstruction, denasal voice and unilateral serous otitis media. Q.2: What is Trotter’s triad? Ans: Triad of trigeminal neuralgia, unilateral palatal immobility (IXth nerve) and conductive hearing loss (due to secretory otitis media) are seen in nasopharyngeal carcinoma. It is also known as sinus of Morgagni syndrome. Q.3: What are the etiologies behind the nasopharyngeal carcinoma? Ans: Epstein-Barr virus infection. Others include genetic factors, ingestion of salted fish (due to presence of nitrosamines) and inhalants like household smoke, tobacco, incense stick fumes. Q.4: What is the cause behind for secretory otitis media in nasopharyngeal carcinoma? Ans: It is due to Eustachian tube dysfunction secondary to mechanical effect of the tumor or due to infiltration of the tubal musculature by the tumor. In unilateral secretory otitis media in adults it is mandatory to rule out nasopharyngeal carcinoma. Q.5: What is the epidemiological factors unique to nasopharyngeal carcinoma? Ans: Maximum incidence of NPC is found in Southern China, especially among the Cantonese population of Guangdong province, hence it is often called Guangdong tumor in this region. The incidence is also high in Hong Kong where majority of the population are Cantonese. Q.6: What are the environmental factors responsible for nasopharyngeal carcinoma? Ans: • Diet high in salted fish, low in vitamins from fruits and vegetables are more prone. • Volatile nitrosamines which are alkylating agents induce malignant changes. • Consumption of shrimp paste, salted vegetables also increase the risk. • Emigrant Chineses continue to consume salted fish. Q.7: Why we think genetic predisposition for NPL. Ans: • High incidence among southern Chinese. • High incidence among southern Chinese immigrants settled in low incidence areas.

212

Manual of Clinical Cases in ENT and Head and Neck Surgery

• Low incidence in population with low risk living in high incidence areas, e.g. Indians in Singapore. • Familial clustering among relatives. • Factors like salted fish affects some population like southern Chinese but Japanese are spared. Q.8: How imaging help in nasopharyngeal carcinoma? Ans: • CT scan is useful to look skull base erosion. • CT scan is better than MRI for lymph node enlargement. • MRI with gadolinium and fat suppression gives excellent delineation of soft tissue and tissue planes. Tissue specificity is also good. Coronal plane is taken to show skull base foramina. Axial plane shows retropharyngeal space, pterygomaxillary fossa and infratemporal fossa. In nasopharyngeal carcinoma, there is lateral displacement of parapharyngeal fat. • Ultrasound of the neck useful for neck node enlargement if CT is not there. • PET scan distinguishes between post-irradiation edema and recurrence. Q.9: What are the immunological/serology tests in nasopharyngeal carcinoma (NPC)? Ans: • IgA anti-VCA (viral capsid antigen) titer is high in NPC. It is not very specific for NPC where it is 95% sensitivity and 80–90% specificity. • IgA anti EA (early antigen) titer is more specific where 95% specificity and 80% sensitivity. • In endemic areas, EBVNA (Epstein-Barr virus nuclear antigen) test is done on FNAC/biopsy material using specific monoclonal antibodies. Presence indicates likelihood of NPC. Q.10: What are the confirmatory methods for diagnosis of NPC? Ans: The diagnosis of NPC is established by nasopharyngoscopy and biopsy for histopathological examination. Q.11: What is Horner’s syndrome? Is NPC a cause? Ans: Horner’s syndrome is characterized by ptosis, miosis, anhydrosis and loss of ciliospinal reflex. It is seen in cases with involvement of cervical sympathetic ganglion. Common causes include nasopharyngeal carcinoma (NPC), parapharyngeal space tumors, deep neck space infections and penetrating neck injury. Q.12: What is the treatment of NPC? Ans: Treatment of NPC involves radiotherapy in early stages while salvage surgery and neck dissection in late stages. Intensity modulated radiotherapy (IMRT) is preferred as it delivers high dose of radiation to the tumor, sparing essential tissues like eye, brainstem. Concurrent chemoradiation is given in patients from stage II onwards and has shown to improve survival in advanced stage.

Nasopharynx

213

Q.13: What is the WHO classification of NPC? Ans: The World Health Organization classifies nasopharyngeal carcinoma into three types: • Type 1: Differentiated keratinizing squamous cell carcinoma. • Type 2: Non-keratinizing carcinoma. • Type 3: Undifferentiated carcinoma. Usually well differentiated keratinizing squamous cell carcinoma is relatively radio resistant but the undifferentiated nasopharyngeal carcinoma which is the most common subtype in endemic areas, is very radiosensitive though it is very aggressive. Q.14: What are different methods for examination of nasopharynx? Ans: • Digital palpation. • Posterior rhinoscopy. • Diagnostic nasal endoscopy. • X-ray of nasopharynx (lateral view). • CT/MRI. • Fiberoptic nasopharyngoscopy. CASE 3: TROTTER’S TRIAD

Presenting complaints: A 50-year-old man presents with conductive hearing loss, facial pain in temperoparietal and lower jaw area and immobile soft palate on the right side. Q. 1: What is the probable diagnosis? Ans: Trotter’s syndrome or triad. This is seen in nasopharyngeal carcinoma. Q. 2: What are the explanations of the Trotter’s triad in nasopharyngeal carcinoma? Ans: Conductive hearing loss is due to eustachian tube obstruction. There is ipsilateral immobility of palate due to malignant cell infiltartions into palate. Neurologic pain in distribution of mandibular division (V3) due to spread of growth laterally through sinus of Morgagni to involve (V3).

214

Manual of Clinical Cases in ENT and Head and Neck Surgery

30

Tumors of Hypopharynx CASE 1: POSTCRICOID CARCINOMA

Presenting complaints: A 32-year-old lady complains progressive painless dysphagia to the solid foods since 3 months. Indirect laryngoscopy: Exophytic growth seen posterior the larynx. Pooling of saliva in both pyriform fossa. Neck examination: Absence of laryngeal crepitus. No neck node enlargement was present. Q.1: What is the most possible diagnosis? Ans: Postcricoid carcinoma. Q.2: What is the postcricoid region? Ans: It is a part of anterior wall of laryngopharynx between upper and lower borders of cricoid lamina. This region constitutes about 30% of hypopharyngeal malignancies. Q.3: What are the etiological factors responsible for postcricoid carcinoma? Ans: Anemia, Plummer-Vinson syndrome, alcohol, tobacco and previous irradiation in the neck. Q.4: What are the common clinical presentations in postcricoid carcinoma? Ans: Commonly found in female, common in the age group of 20–30 years, progressive dysphagia, weight loss, voice change (affecting the larynx or its nerve), pooling of saliva in pyriform fossa (Chevalier Jackson’s sign), absent laryngeal crepitus (Bocca’s sign). Q.5: What are the investigations needed in postcricoid carcinoma? Ans: Hemoglobin (anemia), X-ray soft tissue of neck, barium swallow and endoscopy for biopsy and confirmation, CT scan of the neck with contrast and MRI. Q.6: What are the treatment options for postcricoid carcinoma? Ans: Radiotherapy, laryngopharyngo-esophagectomy with stomach pull up or in cases with failure of radiotherapy for T3, T4 tumor. Pharyngectomy with free jejuna transfer for T1, T2. 214

Tumors of Hypopharynx

215

CASE 2: PYRIFORM FOSSA MALIGNANCY

Presenting complaints: A 62-year-old man presented with increasing pain on swallowing since 3 months with swelling in right side of the upper neck. Examination of the neck: Right neck lymph node enlargement in the right upper jugular group which is firm, nontender about 3 cm in diameter and mobile. Indirect laryngoscopy: It reveals an exophytic growth on the right pyriform fossa, involving the lateral surface of aryepiglottic fold. Q.1: What is the most possible diagnosis? Ans: Malignancy pyriform fossa. Carcinoma of pyriform fossa constitutes about 60% of all hypopharyngeal cancer. It is common in males above 40 years. Growth at pyriform fossa remains asymptomatic for a long time because of large size fossa. Growth may be exophytic or ulcerative or deeply infiltrative. Q.2: What are the characteristics of pyriform fossa? Ans: It lies on either side of the larynx and extends from pharyngoepiglottic fold to the upper end of the esophagus. It is bounded laterally by the thyrohyoid membrane and thyroid cartilage and medially by aryepiglottic fold, posterolateral surface of arytenoids and cricoid cartilages. Pyriform fossa is richly supplied by lymphatics. Lymphatics of the pyriform fossa drains to deep cervical chain in levels IV and VI after exit through thyrohyoid membrane. It is lined by squamous epithelium so almost all malignant lesions are squamous cell carcinoma. Q.3: What are the clinical profiles of carcinoma in pyriform fossa? Ans: Pricking sensation on swallowing (earliest symptom), referred otalgia, pain on swallowing, dysphagia, mass in neck, hoarseness in laryngeal involvement stridor. Q.4: What are the diagnostic investigations in carcinoma of pyriform fossa? Ans: Endoscopy, barium swallow, CT scan of the neck with contrast. Q.5: What are the treatment options in carcinoma of pyriform fossa? Ans: Early lesion—radiotherapy, growth limited to pyriform fossa and larynx—total laryngectomy and partial pharyngectomy plus neck dissection and postoperative radiotherapy. CASE 3: PYRIFORM FOSSA MALIGNANCY

Presenting complaints: A 65-year-old man presented with foreign body sensation in throat since 4 months. He had also swelling in the left side of the neck since 4 months. History of presenting illness: Patient was apparently alright 4 months back when he developed swelling in the upper part of neck (left side), insidious in onset, painless, rapidly increasing in size not associated with stridor/dyspnea/hoarseness of voice/ odynophagia/cough/hemoptysis. He had history of referred otalgia to left side. After 2 months, he developed similar swelling in the lower part of the left side of neck, insidious in onset, painless, rapidly progressive in size.

216

Manual of Clinical Cases in ENT and Head and Neck Surgery

Past histoty: No history of diabetes mellitus, hypertension, pulmonary tuberculosis and ischemic heart disease. Indirect laryngoscopy: Pooling of saliva in bilateral pyriform fossae. Left arytenoid edematous. There is a proliferative growth involving left aryepiglottic fold and medial wall of the left pyriform fossa. Bilateral vocal fold mobile and normal. Neck examination: Left jugulodigastric lymph node 4 × 4 cm, firm to hard, mobile, non-tender. Level 3 lymph nodes 3 × 4 cm, firm to hard, non-tender and mobile. Laryngeal crepitus present. Q.1: What is the possible diagnosis? Ans: Malignancy hypopharynx involving left aryepiglottic fold—T2N2 Bmx. Q.2: Which nerve is responsible for referred for ear pain in cancer of hypopharynx? Ans: Superior laryngeal nerve which is a branch of vagus. Q.3: What are the investigations required in cancer hypopharynx? Ans: • Endoscopic examination • Barium swallow • CT/MRI with contrast • Biopsy • PET scan Q.4: What are different consistencies of neck swelling? Ans: • Hard—most of the carcinomas • Stony hard—secondary swelling • Hard but yielding—chondroma • Boy hard—osteoma • Rubbery—lymphoma • Firm—fibroma • Soft—lipoma

217

Tracheostomy

31

Tracheostomy CASE 1: TRACHEOSTOMY

Presenting complaints: A 60-year-old man presented with inspiratory stridor since 2 days. He had foreign body sensation in throat. Setting: OPD History of presenting illness: He is presenting stridor since 2 days. He has no dysphagia. Occasinally he presents foreign body sensation in throat. History of past illness: He is hypertensive under treatment but non-diabetic. Personal history: He is chronic smoker since 20 years. He take 10 cigarette per day. Indirect laryngoscopy: An irregular growth at right aryepiglottic fold and obstructing glottis airway. Q.1: What is the provisional diagnosis? Ans: Carcinoma of larynx (supraglottic growth). Q.2: Which type of tracheostomy is done in carcinoma of larynx? Ans: High tracheostomy. High tracheostomy is done above the level of thyroid isthmus (at the level of first tracheal ring). Q.3: Low tracheostomy is indicated in? Ans: Juvenile laryngeal papillomatosis. Tracheostomy is not a decision by choice in juvenile laryngeal papillomatosis. If the stoma is involved, chances of recurrence are more. The best way to be away from disease site is a low tracheostomy. Q.4: What are the differences between endotracheal intubation and tracheostomy? Ans: Endotracheal intubation

Tracheostomy

1. It is quicker procedure and need anesthetic help 2. Sometimes preceeds tracheostomy, particularly in severe respiratory distress and children 3. Endotracheal tube should not be kept for more than 48–72 hours 4. Assisted ventilation for short period 5. Chance of infection is less 6. Patient cannot speak 7. Tracheobronchial toilet is difficult 8. Chance of complication is less

1. It needs trained personnel. Usually done under LA/GA in operation theater or bedside 2. In acute respiratory distress, it may be difficult and may damage apical plura 3. Can be kept for prolonged period till the crisis is over 4. Assisted ventilation for long period 5. More chance of infection 6. Patient can speak by closing stoma with finger or tube with speaking valve 7. Easy and effective tracheobronchial toilet 8. Chance of postoperative complication is more

217

218

Manual of Clinical Cases in ENT and Head and Neck Surgery

Q.5: What are the precautions during tracheostomy in infants and children? Ans: a. It should be performed aftern endotracheal intubation b. Low tracheostomy is avoided. c. Incise trachea not too deeply. d. Minimum excision of cartilage to avoid stenosis and tracheomalacia. e. Avoid postoperative aerophagia by nasogastric feeding. f. Prevent lower respiratory tract infection by aseptic care and antibiotic. g. Should prepare for decannulation problem. Q.6: What are the functions of tracheostomy? Ans: • Alternative pathway for breathing. • Improves alveolar ventilation. It decrease dead space by 30–50% (normal dead space is 150 ml). It reduces the resistance to airflow. • Protects the airway by using cuffed tube. • Permits removal of tracheobronchial secretions. • If intermittent positive pressure is needed beyond 72 hours, tracheostomy is superior to intubation. • To adminster anesthesia where endotracheal intubation is difficult. Q.7: What is Bjork flap? Ans: It is a flap of the tracheal wall (one ring) which is sutured to the inferior flap of the skin (in case of transverse incision). It makes a permanent stoma and makes it easier to change the tube if it gets displaced. It is usually not done in children. CASE 2: CRICOTHYROTOMY

Presenting complaints: A 5-year-old boy having dinner suddenly becomes aphonic and develops respiratory distress. What is the appropriate initial management? Setting: OPD Q.1: What is the diagnosis? Ans: This child is likely having an accidental inhalation of food. Q.2: What is the initial management? Ans: Heimlich maneuver where multiple thrusts are given with wrist in the upper epigastrium while standing behind the patient. This forcefully pushes air out of the lungs pushing the foreign body along. Q.3: What is cricothyrotomy? Ans: Cricothyrotomy (Fig. 31.1) is an emergency procedure done by making an opening in the cricothyroid membrane to restore airway where immediate tracheostomy is not possible. It is faster and easier than tracheostomy and carries high risk of tube obstruction and laryngeal injury and stenosis. Cricothyrotomy should be changed to a formal tracheostomy as early as possible.

Tracheostomy

219

Q.4: What are the major complications of cricothyrotomy? Ans: Subglottic stenosis if cricothyrotomy is kept for more than 3 to 5 days. It is an emergency procedure and should not be kept for more than 3 to 5 days. It should be converted to elective tracheostomy.

Fig. 31.1: Cricothyrotomy

CASE 3: TRACHEOSTOMY

Presenting complaints: A 25-year-old man diagnosed as laryngeal papillomatosis presented with stridor since two days. This patient attended the emergency department with stridor. Immediately planned for tracheostomy. Fiberoptic nasopharyngolaryngoscopy: Showed inadequate glottic airway. Q.1: Which type of trachesotmy is required as per the site of stoma in this case? Ans: Low tracheostomy Q.2: What are the types of tracheostomy as per site of the stoma? Ans: • High tracheostomy: Stoma is done at the level of 1st and 2nd rings. • Mid tracheostomy: Stoma is done at the level of 3–5th rings. • Low tracheostomy: Stoma is done at the level below the 5th ring. Q.3: What are the types of tracheostomy as per purpose/need of the procedure? Ans: • Temporary tracheostomy: Once the purpose is served, the tracheostomy is allowed to close. • Permanent tracheostomy: This is done after laryngectomy and occasionally in morbid sleep apnea patients. Q.4: What are the types of tracheostomy as per timing of the surgery? Ans: • Elective tracheostomy: It is a planned tracheostomy as in cases of procedures on the oropharynx and larynx. • Emeregncy tracheostomy: This is performed immediately as in acute complete respiratory tract obstruction.

220

Manual of Clinical Cases in ENT and Head and Neck Surgery

CASE 4: POSTTRACHEOSTOMY COMPLICATIONS

Presenting symptoms: A tracheostomized patient with portex tracheostomy tube, in the ward developed sudden and complete blockage of the tube. Q.1: Which is the best step to help this patient? Ans: Immediate removal of the tracheostomy tube. As portex tracheostomy tube has no inner tube, so removal of the tube is the only option in this situation. In case of tracheostomy tube with inner canula as in Jackson’s or Fuller’s metallic tracheostomy tube removal of the inner canula can be done and not the entire tube. Q.2: How sudden and complete blockage of the tracheostomy tube is important for patient? Ans: Sudden complete obstruction of the tracheostomy tube is an emergency situation. Immediate removal of the tube is a safe alternative. CASE 5: PEDIATRIC TRACHEOSTOMY

Presenting complaints: A 3-year-old male child comint to the emergency with stridor. He had fever 3 days back with cold. The child underwent some medical treatment but failed. Q.1: What is the next to manage to stridor? Ans: Intubation/tracheostomy. Tracheostomy is done if intubation is required beyond 72 hours. Q.2: What are the common indications for tracheostomy in pediatric age group? Ans: Most important indications for tracheostomy in infants is subglottis hemangioma and subglottic stenosis while in children it is acute laryngotracheobronchitis and acute epiglottitis. Q.3: What are things to be considered in case of pediatric tracheostomy? Ans: • Avoid tracheostomy, if endotracheal tube can serve the purpose. • Avoid low tracheostomy. • Avoid excision of the tracheal cartilage. • Avoid too deep incision in the tracheal lumen. • Remember, apical pleura, great vessels of the neck and innominate artery may be easily damaged. • Decannulation problems are more found in children due to psychological dependence.

221

Congenital Lesions of Larynx

32

Congenital Lesions of Larynx CASE 1: LARYNGOMALACIA

Presenting complaints: A 3-month-old child presents with inspiratory stridor and normal cry. Parents tell that stridor disappears when the child is lying prone. Nasopharyngolaryngoscopy: Laryngoscopy showed omega-shaped epiglottis with short floppy aryepiglottic folds (Fig. 32.1).

Fig. 32.1: Laryngomalacia

Q.1: Most probable diagnosis? Ans: Laryngomalacia. Q.2: What is laryngomalacia? Ans: It is characterized by flaccidity of the supraglottic. It is the commonest congenital anomaly causing stridor in infants. It is also known as congenital laryngeal stridor. Q.3: What are the symptoms presented by laryngomalacia child? Ans: Intermittent high pitched inspiratory stridor that develops a few week after birth, usually associated with an acute upper airway obstruction and persists for about 2 weeks. Making the child in prone position leads to relieve of stridor. Q.4: What are the laryngeal findings in laryngomalacia? Ans: Epiglottis—tall, tubular, rolled in omega-shaped. Aryepiglottic fold—short, flaccid and medially prolapsing. Arytenoids—these are prominent, elongated and 221

222

Manual of Clinical Cases in ENT and Head and Neck Surgery

covered by loose mucosa and also characterized by a deep interarytenoid cleft. Whole supraglottic larynx is deepened and narrow. Q.5: What are the treatment strategies in laryngomalacia? Ans: Most of the children recover by 2 years of age. Hence reassurance of the parents should suffice. Very rarely severe respiratory distress may warrant a tracheostomy, excision of redundant mucosa or splitting of aryepiglottic folds. CASE 2: CONGENITAL SUBGLOTTIC STENOSIS

Presenting symptoms: A 3-year-old male child presented with breathing difficulty after upper respiratory tract infection. After some medications, upper respiratory tract infections subsided but mild breathing difficulty persists. His voice is normal with normal cry. Airway distress is not relieved by any position. Past history: No history of prolonged ventilation in past. Nasopharyngolaryngoscopy: Epiglottis, aryepiglottic folds and arytenoids are normal. The subglottis appears narrowed. X-ray neck (lateral view): Narrowing at subglottis. Q.1: What is the diagnosis? Ans: Congenital subglottic stenosis. Q.2: What are the anatomical criteria for subglottis to diagnose congenital subglottic stenosis? Ans: Diagnosis is made when subglottic diameter is less than 4 mm in full-term neonate (normal 4.5–5.5 mm) or 3 mm in premature neonate (normal 3.5 mm). Q.3: What is the treatment of congenital subglottic stenosis? Ans: Many cases of congenital subglottic stenosis improve as the larynx grows but some may need surgery for widening the subglottic airway. Q.4: What are the congenital causes of stridor? Ans: Laryngomalacia, laryngeal web, subglottic stenosis, subglottic hemangioma, vocal cord paralysis, tongue and jaw abnormalities. Q.5: What are clinical conditions, where stridor disappears, when baby lies in prone position? Ans: Laryngomalacia, micrognathia, macroglossia and innominate artery compression. Q.6: What are the differences between infant largnx and adult larynx? Ans: Infant larynx

Adult larynx

Situated high up Size is equal in both sexes Laryngeal cartilages are more soft Narrowest part is the sublottic space Infant epiglottis is narrow and folded Submucous space is more, hence edema more common

Situated opposite to C4–C6 vertebra Male larynx larger than female More rigid Narrowest part is the glottis Leaf like Submucous space is less

Congenital Lesions of Larynx

223

CASE 3: INFANT WITH RESPIRATORY DISTRESS

Presenting complaints: In a full term newborn infant presenting with respiratory distress. Q.1: The diagnosis of congential subglottic stenosis is made when tip of the bronchoscope of which diameter cannot be passed? Ans: Normal diameter of subglottic region of a full term neonate is 4.5 to 5.5 mm. If the tip of 4 mm bronchoscope cannot be passed through subglottic region. The diameter if subglottic region in premature neonate is 3.5 mm and diagnosis of the stenosis is made, if bronchoscope of 3 mm cannot be passed. Q.2: How subglottic stenosis found congenitally? Ans: Congenital subglottic stensosis is due to thickening of subglottic soft-tissues or thickening of anterior part of cricoid cartilage.

224

Manual of Clinical Cases in ENT and Head and Neck Surgery

33

Infections of Larynx CASE 1: ACUTE EPIGLOTTITIS

Presenting complaints: A 4-year-old boy presents with fever, dyspnea and throat pain since 2 days. He has also odynophagia. Setting: OPD History of present illness: The child was apparently alright one week back. Following upper respiratory tract infection, the child developed fever, throat pain and dyspnea. He has no history of foreign body aspiration. No history of trauma to the neck area. General physical examination: The patient is well oriented with time, place and person. Pulse rate—86/minute, BP—122/80 mmHg. Fever—40°C. Examination of oral cavity and oropharynx: Mild congestion at oropharyngeal area. Depressing the tongue with a tongue depressor, showing red and swollen epiglottis (Fig. 33.1).

Fig. 33.1: Acute epiglottitis

Indirect laryngoscopy: Avoided. Examination of the neck: Normal Examiantion of the ear, nose, paranasal sinuses: Normal 224

Infections of Larynx

225

Q.1: What is the most probable diagnosis? Ans: Acute epiglottitis. Q.2: What is the routine and earliest investigation in acute epiglottitis? Ans: X-ray neck showing swollen epiglottis, called thumb sign (Fig. 33.2).

Fig. 33.2: Thumb sign in lateral view of X-ray neck

Q.3: What is the causative organism for acute epiglottitis? Ans: Haemophilus influenzae type B. But Parainfluenza virus types I and II are causative agent for acute laryngotracheobronchitis. Q.4: In which age group, acute epiglottitis is common? Ans: It is common in 2–7 years. But acute laryngotracheobronchitis is common in 3 months to 3 years. Q.5: What is the treatment of acute epiglottitis? Ans: Antibiotics: Ampicillin or third generation cephalosporin. Steroids: Hydrocortisone or dexamethasone for relieving edema, adequate hydration, humidification and oxygen. Intubation and tracheotomy may be needed if respiratory obstruction occurs. Q.6: A child is toxic and febrile sitting upright mouth open with neck extended to optimize the airway and using the arms to support the shoulder girdle called tripod position, seen in? A. Acute epiglottitis B. Acute laryngotracheobronchitis C. Laryngitis D. Laryngeal papilloma Ans. A: Acute epiglottitis.

Manual of Clinical Cases in ENT and Head and Neck Surgery

226

Q.7: X-ray of anterior posterior view of neck showing steeple sign or pencil tip sign (Fig. 33.3) is associated with? A. Acute laryngotracheobronchitis (ALTB) B. Supraglottic cancer C. Epiglottic edema D. Acute epiglottitis. Ans. A: Acute laryngotracheobronchitis (ALTB)

Fig. 33.3: AP view of neck in normal case and croup (ALTB)

Q.8: What are the differences between acute epiglottitis and acute laryngotracheobronchitis? Acute epiglottitis

Acute laryngotracheobronchitis

Causative agent

Haemophilus influenzae type B

Parainfluenza virus types I, II

Age

2–7 years

3 months to 3 yeras

Pathological site

Supraglottis

Subglottic area

Onset

Sudden

Slow

Prodromal symptoms

Absent

Present

Fever

High grade

Low grade/no fever

Patient’s appearance

Toxic

Non-toxic

Cough

Usually absent

Present (barking seal like)

Stridor

Present and marked

Present

Odynophagia

Present with drooling of saliva

Usually absent

Imaging

Thumb sign on lateral view

Steeple sign on AP view

Treatment

Humudified oxygen, broad spectrum antibiotics

Humudified oxygen, steroids

CASE 2: TB LARYNX

Presenting complaints: A 45-year-old man presented with hoarseness of voice since 2 months. Indirect laryngoscopy: Multiple ulcers on the vocal cords. Both arytenoids are swollen. Epiglottis is swollen.

Infections of Larynx

227

History of present illness: He was apparently alright 2 months back. To start with change in voice and mild odynophagia since 2 months. Past illness: He had history of pulmonary tuberculosis 3 years back. No history of DM and HTN. Q.1: What is the most possible diagnosis? Ans: Tuberculosis of larynx. It is usually secondary to pulmonary tuberculosis. Q.2: What are the earliest symptom and signs of laryngeal TB? Ans: Earliest symptom is weakness of voice followed by hoarsness of voice. The earliest sign is hyperemia of the vocal cord and posterior commissure area and loss of complete adduction. Q.3: What are the clinical characteristic features of TB larynx? Ans: TB of ear and nose are painless whereas TB of larynx is an extremely painful condition. When epiglottis involved, patient often complain painful deglutition. Q.4: What are the findings of TB larynx in indirect laryngoscopy? Ans: Mamillated arytenoids—the arytenoids appear very swollen, mouse nibbled appearance of vocal cords—multiple ulcers over vocal cords, turban epiglottis— epiglottis appear swollen due to cellular infiltration (pseudoedema). TB of larynx starts in the posterior part of larynx. Q.5: What is the treatment of TB larynx? Ans: Antitubercular therapy. CASE 3: ANGIONEUROTIC EDEMA

Presenting complaints: A 29-year-old man suddenly developed hoarseness, wheeze and stridor while eating peanuts in a bar. Setting: Emergency department. Examination of oral cavity and oropharynx: Swollen tongue. Indirect laryngoscopy: Not done due to fear of exacerbation of breathing difficulty. Examination of neck, ear and nose: Normal. Q.1: What is the most probable diagnosis? A. Foreign body B. Angioedema C. Hypothyroidism D. Acute epiglottitis Ans. B: Angioedema Q.2: What is the basic radiological tests in this case? Ans: X-ray of soft tissues of neck (anteroposterior and lateral view). It will reveal the patency of airway. Chest X-ray is also helpful.

228

Manual of Clinical Cases in ENT and Head and Neck Surgery

CASE 4: CROUP/ACUTE LARYNGOTRACHEOBRONCHITIS

Presenting complaints: A 3-year-old boy attended the emergency department with hoarseness of voice and croupy cough since 3 days. He has mild breathing difficult. X-ray of neck: Anterior-posterior (AP) of the neck shows narrowing of the subglottic area. Q.1: What is the most possible diagnosis? Ans: Acute laryngotracheobronchitis (Croup). Q.2: What is steeple sign? Ans: Narrowing of the sublottic space in X-ray of the neck in acute laryngotracheobronchitis is called steeple sign. Q.3: What are etiological agents causing acute laryngotracheobronchitis? Ans: Parainfluenza virus type I and II. It commonly affects the children between the ages of 3 months to 3 years. Q.4: What are the clinical presentations of the acute laryngotracheobronchitis Ans: • Slow onset • Low grade or no fever • Barking seal like cough • Stridor • Odynophagia absent • Nontoxic • Prodromal symptoms are present Q.5: What are the treatment done in acute laryngotracheobronchitis? Ans: Hospitalization, antibiotics (amoxycillin), humidification, parenteral fluids, steroids (e.g. hydrocortisone 100 mg IV may be useful), adrenaline (racemic adrenaline administered via respirator is a bronchodilator and may relieve dyspnea and avert tracheostomy and intubation/tracheostomy in case of patient with respiratory obstruction. CASE 5: LARYNGITIS

Presnting complaints: A 25-year-old man has a 3 day history of hoarseness of voice. He has throat pain, which is worse on talking and eating. Examination: Oral cavity and oropharynx—normal. Indirect laryngoscopy: Bilateral vocal folds are congested (Fig. 33.4). There is erythema over arytenoids, aryepiglottic folds and epiglottis.

Infections of Larynx

Fig. 33.4: Bilateral vocal folds are congested

Q.1: What is the most possible diagnosis? Ans: Acute laryngitis. Q.2: What are treatment options in acute laryngitis? Ans: • Voice rest. • Avoidance of smoking and alcohol. • Steam inhalation with oil of eucalyptus pine and Tr Benzoin Co. • Cough sedatives. • Antibiotics is associated with secondary infections. • Analgesics is associated with local pain and discomfort. • Steroids are useful in case of laryngitis with thermal or chemical burns.

229

230

Manual of Clinical Cases in ENT and Head and Neck Surgery

34

Foreign Body in Airway CASE 1: FOREIGN BODY BRONCHUS

Presenting complaints: A 4-year-old boy presented with breathing difficulty since one day. His parents gave history of aspiration of button battery one day back. X-ray chest: It showed a radio-opaque foreign body at the level of right main bronchus. Q.1: What is the most possible diagnosis? Ans: Foreign body (button battery) in right bronchus. Q.2: What are the differences between metallic and non-metallic foreign bodies in airway? Ans: Metallic foreign bodies as compared to vegetable foreign bodies are inert and minimally reacting. As a result, these do not produce any immediate signs and symptoms of foreign body unless obstructing the airway significantly. In contrast vegetable foreign bodies are fast reacting, after lodging in tracheobronchial tree, these swell up blocking the passage and producing chemical reaction leading to pneumonitis. Q3: What is Heimlich’s maneuver? Ans: Sudden thrust directed upwards and backwards, below the epigastrium, squeezes the air from lungs which is sufficient to dislodge a foreign body from the airway. Q.4: What are the methods for removal of tracheobronchial foreign bodies? Ans: • Rigid bronchoscopy • Rigid bronchoscopy with optical forcep. • Bronchoscopy with C-arm fluoroscopy. • Using Dormia basket or Fogarty’s balloon for rounded objects. • Tracheostomy followed by foreign body removal through tracheostome. • Thoracotomy and bronchotomy for peripheral foreign bodies. • Flexible fiberoptic bronchoscopy in selected adult patients. 230

Foreign Body in Airway

231

CASE 2: FOREIGN BODY BRONCHUS

Presenting complaints: A child taking his food, suddenly he started coughing and gets chicked and develops respiratory stridor. Q.1: What is the possible diagnosis? Ans: Foreign body bronchus. Q.2: How the foreign body in airway is diagnosed? Ans: • History • Clinical examination • Radiograph of the chest • Endoscopy-foreign body is confirmed by fiberoptic bronchoscopy Q.3: What are the clinical features of the foreign body in airway? Ans: Earliest symtpms are sudden onset of bouts of cough and dyspnea while eating. In children sudden onset of cough while eating or playing with small toys, should raise suspicion. In some cases the foreign body may be silent in the early period. Later on cough increases and there is expectoration. Wheeze is quite common. Q.4: What are the common sites of foreign body inhalation? Ans: • Most common site of inhalated foreign body in children is bronchus. Most commonly at the right bronchus. • Laryngeal or tracheal locations of the foreign body inhalation occurs in approximately in 10% cases. • Laryngeal or tracheal locations occurs in 10% cases. Q.5: What are the common sites of foreign body in different positions of the patients? Ans: • If the patient has aspirated in upright or sitting position, basilar segment of lower lobe is the most likely to be involved. • In supine position either the posterior segment of upper (apical) lobe or superior segment of the lower lobe is likely to be involved. • In both cases, right side is more likely to be involved due to straight and shorter course of the right bronchi. Q.6: How the case of airway foreign body is managed? Ans: • Bronchoscopy should be done as soon as possible • In case where dyspnea is marked—tracheostomy is advisable either before or after bronchoscopy • Heavy dose of antibiotics is required in long standing airway foreign body

232

Manual of Clinical Cases in ENT and Head and Neck Surgery

35

Hoarseness of Voice CASE 1: VOCAL NODULES

Presenting complaints: A 30-year-old opera singer has developed horseness 2 days before a world premiere for which she has been rehearsing extensively. She does not have a sore throat. Setting: Outpatient department. Indirect laryngoscopy: Bilateral vocal cord nodules at the junction of anterior onethird to posterior two-thirds (Fig. 35.1).

Fig. 35.1: Vocal nodules

Examination of neck: Normal. Examination of oral cavity and oropharynx: Normal. Examination of ear, nose and sinus: Normal. Q.1: What is the diagnosis? Ans: Vocal cord nodules. Q.2: What are the common symptoms of vocal nodules? Ans: Commonest symptom is hoarseness. Vocal fatigue and pain in neck on prolonged phonation are other common symptoms. Q.3: What is the treatment of vocal nodules? Ans: Early cases of vocal nodules can be treated conservatively by educating the patient in proper use of voice. By this treatment, many cases of vocal nodules in 232

Hoarseness of Voice

233

children disappear completely. Surgery is needed for large nodules or nodules of long standing in adults. They are excised precisely under operating microscope either with cold instruments or laser avoiding any trauma to underlying vocal ligament. CASE 2: HOARSENESS DUE TO HYPOTHYROIDISM

Presenting complaints: A 45-year-old lady develops a progressively hoarse voice since 6 months. She has also gained 5 kg of weight and complains of constipation. She had undergone hemithyroidectomy 2 years back. Setting: Outpatient department. Indirect laryngoscopy: Bilateral vocal cords are mobile and thick. Examination of oral cavity, oropharynx: Normal. Examination of neck: Normal. Examination of ear, nose and sinus: Normal. Q.1: What is the most probable diagnosis? Ans: Hypothyroidism. Q.2: What are the causes for hoarseness of voice? Ans: • Acute/chronic laryngitis. • Vocal nodule. • Vocal cord polyp. • Reinke’s edema. • Glottic cancer. • Vocal cord paralysis. • Vocal cord papilloma. • Hypothyroidism. • TB larynx. Q.3: What are the differences between adult solitary papilloma and juvenile multiple papilloma? Ans: Adult solitary papilloma

Juvenile multiple papilloma

Occurs in adults

Occurs in children

Arises from free edge of vocal cord

Can arise from any part of glottis

Looks solitary in appearance

Looks like grape

No viral etiology

Viral etiology

Does not recur after removal

Recurrence is common

They are premalignant and may undergo malignant degeneration so long term follow up is necessary

Not premalignant and does not undergo malignant degeneration

234

Manual of Clinical Cases in ENT and Head and Neck Surgery

Q.4: What is Reinke’s edema? Ans: Reinke’s edema (bilateral diffuse polyposis) is due to collection of fluid in the subepithelial space of Reinke. The common cause is vocal abuse and smoking. Both vocal cords appear diffuse symmetrical swellings. Treatment of Reinke’s edema is vocal cord stripping, preserving enough mucosa for epithelialization. Cessation of smoking and speech therapy are essential to prevent recurrence. Q.5: What is dysphonia plica ventricularis? Ans: In this case, voice is produced by false cords/ventricular folds. Voice is rough, low pitched and unpleasant. This may be secondary to impaired function of true cord such as paralysis, fixation, surgical excision or tumor. Here, the ventricular bands try to compensate function of true cord. Treatment is voice therapy and psychological counseling. CASE 3: ADDUCTOR DYSPHONIA

Presenting complaints: A 25-year-old female complaining scratchy croaky voice since 2 months. Setting: OPD Indirect laryngoscopy: Larynx appears morphologically normal. During adduction, vocal cords showing tremors. Q.1: What is diagnosis? Ans: Adductor dysphonia. Q.2: What is best treatment option for adductor dysphonia? Ans: The best treatment is percutaneous botulinum injection in the thyroarytenoid muscle. Percutaneous injection in the cricoarytenoid muscle (the only abductor of vocal cord) is done in abductor dysphonias. Q.3: What is the quality of voice in adductor dysphonia? Ans: The voice is scratchy croaky which is a high pitched sound arising due to rubbing of vocal cords in the adductor dysphonias. In abductor dysphonia the voice produced is low pitched breathy and whispery. Q.4: What is etiology of adductor dysphonia? Ans: The etiology of this condition is uncertain but one should exclude neurological conditions such as parkinsonism, myoclonus, pseudobulbar palsy, multiple sclerosis, cerebellar disorders, tardive dyskinesia and amyotrophic lateral sclerosis. Q.5: What is abductor dysphonia? Ans: It is due to spasm of posterior cricoarytenoid muscle. Cause is not known. This condition is gradually progressive and symptoms get aggravated during periods of stress or when patient uses telephone. Q.6: What is the treatment of adductor dysphonia? Ans: It consists of botulinum toxin injections in the thyroarytenoid muscle on one or both sides to relieve spasm. Percutaneous electromyography (EMG) guided route

Hoarseness of Voice

235

through cricothyroid space is preferred. Dose of botulinum toxin depends on severity of the condition. Botulinum toxin injection relieve voice breaks due to spasms and improve airflow but benefit lasts only up to 16 weeks or so when repeat injection may be needed. Voice therapy has also benefit. Q.7: What are the treatments of abductor dysphonia? Ans: Botulinum toxin injection in the posterior cricoarytenoid muscle. Speech therapy is also effective. CASE 4: VOCAL CORD POLYP

Presenting complaints: A 28-year-old lady teacher presents with hoarseness of voice since 3 months. Setting: OPD. Indirect laryngoscopy: A single smooth, pink and pedunculated mass arising from the right vocal cord. Q.1: What is the most possible diagnosis? Ans: Right vocal cord polyp (Fig. 35.2).

Fig. 35.2: Right vocal cord polyp

Q.2: What is the pathophysiology of vocal cord? Ans: Localised vascular engorgement and microhemorrhages followed by edema. Polyp is a fluid filled swelling lesion at the anterior third of the vocal cord. It occurs due to excessive voice abuse, smoking, allergy, trauma, hematoma and chronic infections. Q.3: What are the differences between vocal cords polyp and cysts? Ans: The cysts are intracordal whereas polyps are pedunculated and may hung down on their stalk to sit below the cords. Q.4: What is the treatment of vocal cord polyp? Ans: Microlaryngoscopic excision (MLS with CO2 laser).

236

Manual of Clinical Cases in ENT and Head and Neck Surgery

CASE 5: CONTACT ULCER OF LARYNX

Presenting complaints: A 23-year-old boy presents with hoarseness of voice and pain in throat since one year. Setting: OPD. Indirect laryngoscopy: It reveals heaped edge of the vocal process of one side which may appear fitting in a greater of the opposite cord giving an appearance of ulcer. Q.1: What is the possible diagnosis? Ans: Contact ulcer of larynx. Contact ulcer of larynx was first described by Chevalier Jackson. So it often called ulcer of Jackson. These are not ulcers but a contact pachydermia resulting from misuse of voice. This lesion appears on the vocal process. The localized thickened hyperplastic epithelium becomes headed up around a crator. It may lead to granuloma formation. Q.2: What is the pathogenesis of the contact ulcer of larynx? Ans: There is thickened hyperplastic epithelium occurring on the vocal process without affecting the membranous part of the vocal cords. Thickening occurs in a form of oval mound on the inner surface of vocal folds but extends into upper surface due to the hammering on one vocal process to opposite one. Q.3: Why voice changes occur in contact ulcer of larynx? Ans: By phonation, the localized heaped up area comes into contact and prevents complete adduction of the vocal cords. Due to repeated striking of one mound with other mound of the vocal process, a crator like depression is formed on one mound. On phonation characteristic cup and ball filling of the mounds is seen. This vocal fatigue and hoarseness of the voice are main symptoms in contact ulcer larynx. Q.4: What is the treatment of contact ulcer of larynx? Ans: Voice rest, speech therapy, treatment of acid reflux and microlaryngoscopic excision. CASE 6: REINKE’S EDEMA

Presenting complaints: A 23-year-old boy attended the ENT OPD for complaints of hoarseness of voice since 2 months. Voice is low pitch and rough. Indirect laryngoscopy: Bilateral vocal cords appear fusiform swelling (Fig. 35.3) with pale translucent look. Ventricular bands may appear hyperemic and hypertrophic.

Fig. 35.3: Reinke’s edema

Hoarseness of Voice

237

Q.1: What is the most possible diagnosis? Ans: Reinke’s edema. Q.2: What is Reinke’s edema? Ans: It is bilateral symmetrical swelling of the whole of membranous part of the vocal cords, often seen in middle aged men and women. This is due to edema of the subepithelial space (Reinke’s space) of the vocal cords. Q.3: What are the causes for Reinke’s edema? Ans: Chronic irritation of vocal cords due to misuse of voice, heavy smoking, chronic sinusitis and laryngopharyngeal reflux are the probable etiological factors. It can also occur in myxoedema. Q.4: What are the treatments of Reinke’s edema? Ans: Decortication of the vocal cords, i.e. removal of the strip of epithelium, is done on one of the vocal cord and 3-4 weeks after on the other vocal cord. Voice rest. Speech therapy for proper voice production. Laser surgery is a recent advancement in the treatment by excision of the edematous mucosa along the margin of vocal cord without affecting deeper tissue. CASE 7: VOCAL CORD PARALYSIS

Presenting complaints: A 12-year-old boy developed hoarseness of voice following an attack of diphtheria. On examination: Indirect laryngoscopy showed right vocal cord paralysis. Q.1: What is the treatment of choice in this case? Ans: Unilateral paralysis of the vocal cord due to neuritis (as in diphtheria) does not require any treatment as it recovers spontaneously. The characteristic features of diphtheria neuropathy is that it recovers completely. Q.2: What are the causes of the only right side recurrent laryngeal nerve paralysis? Ans: Aneurysm of the subclavian artery, cancer at the apex of the right lungs and tuberculosis of the cervical pleura. Q.3: What are the causes of the only left side recurrent laryngeal nerve paralysis? Ans: Mediastinal causes, aortic aneurysm, carcinoma thoracic esophagus and enlarged left auricle. Q.4: What are the common causes of both right and left recurrent laryngeal nerve paralysis? Ans: Thyroid surgery, thyroid cancer, cancer cervical esophagus, cervical lymphadenopathy, peripheral neuritis and idiopathic. Q.5: In case of injury to the superior laryngeal nerve, what is the characteristic of voice? Ans: There will be loss of the timber of voice. Voice will be weak and pitch cannot be raised. Anesthesia of the larynx above the level of vocal cords leads to increased chance of aspiration.

238

Manual of Clinical Cases in ENT and Head and Neck Surgery

CASE 8: PHONASTHENIA

Presenting complaints: A 32-year-old man presented with change in voice (weak voice) since one month. He had history common cold one month back and developed hoarseness of voice but not treated properly and not obeyed voice rest. Indirect laryngoscopy: Key hole appearance of glottis. Q.1: What is the provisional diagnosis? Ans: Phonasthenia. Key hole glottis is seen in phonasthenia when both thyroarytenoid and interarytenoids are involved. Q.2: What is phonasthenia? Ans: It is weakness of voice due to fatigue of phonatory muscles. Thyroarytenoid and interarytenoids or both may be affected. It is seen in abise or misuse of voice or following laryngitis. Q.3: What are the characteristic findings of larynx in phonasthenia? Ans: Elliptical space between the vocal cords is seen in weakness of the thyroarytenoid. Triangular gap near the posterior commissure is seen in weakness of the interarytenoid. Keyhole appearance of the glottis is seen when both thyroarytenoid and interarytenoids are affected. CASE 9: PACHYDERMIA LARYNGES

Presenting complaints: A 35-year-old man presented with hoarseness of voice since 2 months. Indirect laryngoscopy: It showed heaping up of thickened hyperplastic epithelium in the interarytenoid region and posterior third of vocal cords. Q.1: What is the provisional diagnosis? Ans: Pachydermia laryngis. Q.2: What is pachydermia larynges? Ans: It is a form of chronic hypertrophic laryngitis affecting the posterior part of the larynx in the region of interarytenoid and posterior part vocal cords. Q.3: What are etiologies for pachydermia laryngis? Ans: • Misuse of voice is the commonest cause for pachydermia laryngis. It results from faulty production of voice rather than from its excessive use. • Prolonged intubation • Esophageal dysfunction • Excessive alcohol consumption • Chronic cigarette smoking Q.4: What are clinical features of the pachydermial laryngis? Ans: • It mostly seen in men more than 30 years of age • Indirect laryngoscopy shows heaping up of thickened hyperplastic epithelium in the interarytenoid region and posterior third of vocal cords. The thickened

Hoarseness of Voice

239

hyperplastic epithlium is saucer-shaped giving rise to an appearance of an ulcer. • This condition is bilateral and symmetrical • It does not undergo malignant transformation, however biopsy of this lesion is essential to differentiate it from carcinoma and tuberculosis. Q.5: What is the treatment of pachydermial laryngis? Ans: Treatment of choice is microlaryngeal excision of the hyperplastic epithlium and control of acid reflux and speech therapy. CASE 10: LASER ASSISTED MICROLARYNGEAL SURGERY

Presenting complaints: During laser removal of a benign laryngeal tumor, ignition of the endotracheal tube is seen. Q.1: What is your immediate response should be? Ans: Endotracheal tube is removed to prevent further burns to the larynx and trachea. Bronchoscopy is performed to assess the damage to trachea and remove any foreign material. Airway is established by another endotracheal tube. In severe burns, tracheostomy is needed. CASE 11: SPEECH DISORDER

Presenting complaints: A 4-year-old boy attended the ENT OPD for stammering. On examination: Oral cavity and oropharynx—normal. Ear and nose examination— normal. He has defect in fluency of speech. Q.1: What is the possible speech disorder? Ans: Stuttering Q.2: What is stuttering? Ans: It is a disorder of fluency of speech and consists of hesitation to start, repetitions, prolongations or blocks in the flow of speech. When well established, a stutterer may develop secondary mannerisms such as facial grimacing, eye blink and abnormal head movements. If too much attention is given or child reprimanded by parents and peers, this behavior pattern may become fixed and child may develop into an adult stutterer. Q.3: How you manage the stuttering? Ans: Stuttering can be prevented by proper education of the parents, not to overreact to child’s dysfluency in early stages of speech development. Treatment of an established stutterer is speech therapy and psychotherapy to improve the image of a speaker and reduce his/her fear of dysfluency. Q.4: What is rhinolalia clausa? Ans: It is lack of nasal resonance for words which are resonated in the nasal cvity, e.g. m, n, ng. It is due to blockage of the nasal cavity or nasopharynx.

240

Manual of Clinical Cases in ENT and Head and Neck Surgery

Q.5: What is rhinolalia aperta? Ans: It is seen when certain words which have little nasal resonance are resonated through nose. The defect is in the failure of the nasopharynx to cut off from the oropharynx or abnormal communication between the oral and nasal cavities. CASE 12: DYSPHONIA PLICA VENTRICULARIS

Presenting complaints: A 20-year-old girl presenting with rough voice since 2 months. Indirect laryngoscopy: Bilateral false cords are touching each other when patient is producing sound ‘eee’. Q.1: What is most possible diagnosis? Ans: Dysphonia plica ventricularis. Q.2: What is the cause for dysphonial plica ventricularis? Ans: Voice in dysphonia plica ventricularis is rough and produced by false cords (ventricular bands suggested in the name of the condition itself) which take over the function of true vocal folds when they are diseased as in paralysis, fixation, surgical excision or tumors of the cords. Ventricular band dysphonia can also be psychogenic without any true cord abnormality.

241

Laryngeal Growth

36

Laryngeal Growth CASE 1: GLOTTIC CARCINOMA

Presenting complaints: A 62-year-old male presenting with hoarseness of voice since 2 months. Personal examination: He is a chronic smoker since 12 years. General examination: Pulse rate is 72/minute, regular, good volume and blood pressure is 128/80 mm of Hg. Respiratory rate is 18/minute. Neck examination: Trachea central, no thyroid enlargement, JVP not raised, no cervical lymphadenopathy. Laryngeal crepitus present. Indirect laryngoscopy: Structures like base of tongue, vallecula, epiglottis and pyriform sinuses are normal. Left true vocal cord shows an exophytic irregular growth limited to the surface only (Fig. 36.1). Mobility of right vocal cord is normal while of left vocal cord is restricted.

Fig. 36.1: Glottic growth

Q.1: What is the most possible diagnosis? Ans: Left vocal cord growth. The appearance of growth seems to be malignant. Taking biopsy and histopathological examination confirms the diagnosis. Q.2: What are the precancerous lesions of larynx? Ans: Leukoplakia, erythroplakia, solitary papilloma and chronic laryngitis. 241

242

Manual of Clinical Cases in ENT and Head and Neck Surgery

Q.3: What are the investigations in glottic carcinoma? Ans: Routine blood tests (DC, TLC, Hb%, BT, CT, platelets), microlaryngoscopy and biopsy with HPE, chest X-ray, CT scan of neck, complete urine examinations, LFT and renal function tests. Q.4: What will be the management of glottic carcinoma? Ans: Aim of the treatment is to give the patient a disease free period. Treatment is radiotherapy or surgery or combination of two. In stage 1, RT is equally effective with advantage of preserving the good quality of voice. Q.5: What is the prognosis of glottic cancer? Ans: As glottic cancer has early presentation and lack of lymphatic drainage, it is also called “Welcome cancer”. It has good prognosis than any other types of laryngeal carcinoma. Q.6: When we will suspect glottic cancer in patient? Ans: Any elderly male smoker, presenting with change in voice lasting for more than 15 days duration should be suspected to have glottic malignancy until proved otherwise. Q.7: What are the indications for total laryngectomy? Ans: • T3 lesions (e.g. vocal cord fixed). • All T4 lesions. • Invasion of malignancy into thyroid and cricoid cartilage. • Bilateral arytenoids cartilage involvement. • Lesions of posterior commissure. • Failure after radiotherapy or conservative surgery. • Transglottic cancer. Q.8: What are the factors must be considered in the treatment options for laryngeal cancer? Ans: • Age of the patient. • Mobility of the vocal cord. • Status of lung function. • Cartilage invasion. • Subsites of supraglottis. • Age of the patients. • Subsites of supraglottic involved. • Status of pre-glottic space involvement. CASE 2: GLOTTIC CARCINOMA

Presenting complaints: A 52-year-old male school teacher presented to his family doctor complaining of hoarseness of voice for 3 weeks. The patient was referred to the otolaryngologist.

Laryngeal Growth

243

On examination: Indirect laryngoscopic examination revealed an irregular growth of the right vocal fold. Remaining examinations of the head and neck region are within normal limit. Flexible nasopharyngolaryngoscopic examination: Irregular growth at the medial border of the right vocal fold, sparing the anterior commissure and ventricle. Q.1: What is the next plan in this case? Ans: Planned for direct laryngoscopy and biopsy under general anesthesia. Tissue will be sent for histopathological examination. Q.2: If the histopathological staging of the mass is squamous cell carcinoma, what is the staging of the tumor? Ans: T1N0M0. This is stage I squamous cell carcinoma of the larynx. Q.3: What is the treatment option in this case? Ans: Transoral microlaryngeal resection of the early stage of the glottic cancer or primary radiation. Q.4: What are the important risk factors for laryngeal cancer? Ans: Smoking and alcohol are recognized major risk factors for laryngeal cancer. Human papilloma virus (HPV) may play role in laryngopharyngeal cancer. Laryngopharyngeal reflux is also considered as a risk factor for laryngeal cancer. CASE 3: JUVENILE LARYNGEAL PAPILLOMATOSIS

Presenting complaints: A 4-year-old boy presented in emergency with mild respiratory distress since 3 days. Laryngoscopic examination: Multiple papilloma over the vocal folds. Q.1: What is possible diagnosis? Ans: Juvenile laryngeal papillomatosis. Q.2: What is juvenile laryngeal papillomatosis? Ans: • Most common laryngeal tumors of the childhood but are otherwise rare. • These are viral in origin and probably caused by human papilloma virus. • These are multiple. • Often affect infants and young children. • Common clinical presentations are hoarseness of voice and stridor. • They are known for recurrence after removal and, therefore, multiple laryngoscopies may be needed. Q.3: What are treatment options of juvenile laryngeal papillomatosis? Ans: • Excision by endoscopy or microlaryngeal surgery. • Serial CO2 laser ablation (being preferred these days). • Coblation.

244

Manual of Clinical Cases in ENT and Head and Neck Surgery

• Diathermic cauterization of each papilloma. • Cryosurgery. • Interferon—used to prevent recurrence. Patients should be followed regularly till it resolves completely. CASE 4: LARYNGEAL CANCER

Presenting complaints: A 65-year-old nam diagnosed with carcinoma of the larynx with involvement of the anterior commissure and right vocal fold. He developed perichondritis of the thyroid cartilage. Q.1: What is the treatment plan in this case? Ans: Perichondritis of thyroid cartilage in a patient of carcinoma of larynx suggests invasion of thyroid cartilage, i.e. stage T4. Stage T4 lesion irrespective of the site of carcinoma of larynx (i.e. supraglottic, glottic, subglottic) is managed by total laryngectomy with neck dissection for clinically positive neck nodes with postoperative radiotherapy. Q.2: What are the indications of total laryngectomy in carcinoma of larynx? Ans: • T3 lesions (i.e. vocal cord fixed) • All T4 lesions • Invasions into thyroid or cricoid cartilage • Bilateral arytenoid cartilage involvement • Lesions of posterior commissure • Failure after radiotherapy or conservative surgery • Transglottic cancers, i.e. tumors involving supraglottis and glottis across the ventricle causing fixation of the vocal cord. CASE 5: LARYNGEAL GROWTH

Presenting complaints: A middle aged male presents with change in voice since one month. Examination: Poor orodental hygiene and pigmented buccal mucosa. Indirect laryngoscopy: Exophytic pinkish growth on the surface of the left vocal cord with no mobility. Neck examination: No lymphadenopathy. Q.1: What is the most possible diagnosis? Ans: Growth of the left vocal cord Q.2: What are other differential diagnoses? Ans: • Benign lesion like vocal nodule • Solitary papilloma • Malignancy

Laryngeal Growth

245

Q.3: Is it a malignant growth? Ans: The appearance of the vocal cord mass look like a malignancy. Q.4: How we will confirm the diagnosis? Ans: Biopsy Q.5: What are other methods to confirm its malignant growth? Ans: Besides history and examination of the larynx, supravital stain and FNAC of the lymph nodes, if present can be helpful. Q.6: What are the precancerous lesions of the larynx? Ans: • Leukoplakia • Erythroplakia • Solitary papilloma • Chronic laryngitis

246

Manual of Clinical Cases in ENT and Head and Neck Surgery

37

Esophagus CASE 1: FOREIGN BODY ESOPHAGUS

Presenting complaints: A 65-year-old lady presents with a piece of mutton stuck in her throat since one day. Setting: Out patient department. Indirect laryngoscopy: Pooling of saliva at pyriform fossa. General examination: Normal findings. X-ray soft tissue of neck: A calcified bolus at the level of cricopharynx. Q.1: What is most possible diagnosis? Ans: Foreign body at cricopharynx. Q.2: What is the most common site of foreign body of upper digestive tract? Ans: Cricopharynx—cricopharynx is the narrowest part of GI tract except for vermiform appendix. Q.3: How foreign body from esophagus is removed? Ans: Most of the foreign bodies in esophagus can be removed by esophagoscopy under general anesthesia. Q.4: What are the complications of esophageal foreign body? Ans: • Respiratory obstruction • Periesophageal cellulitis and abscess in the neck • Perforation of esophageal wall • Tracheoesophageal fistula • Ulceration and stricture of esophagus CASE 2: CARDIAC ACHALASIA

Presenting symptoms: A 42-year-old lady presented with regurgitation of undigested food after 4 hours of ingestion with fetid flatulence. On examination: Patient has ill health, weakness and weight loss. 246

Esophagus

247

Radiological findings: Barium swallow shows smooth pencil-shaped narrowing lower segment (Fig. 37.1), dilatation and tortuosity of lower esophagus, lack of gas bubble in stomach and incordinated peristalsis.

Fig. 37.1: Barium swallow showing cardiac achalasia

Q.1: What is the most possible diagnosis? Ans: Cardiac achalasia. Q.2: What is the etiology of cardiac achalasia? Ans: The exact cause is not known but may be due to degeneration of Auerbach’s plexus, leading to incoordination of peristalsis causing failure of relaxation of spincture and thereby, retention of food takes place in lower esophagus. Q.3: What is the surgical treatment of cardiac achalasis? Ans: Heller’s operation (esophago-cardiomyotomy). CASE 3: DIFFUSE ESOPHAGEAL SPASM

Presenting complaints: A 32-year-old man presenting with nonprogressive dysphagia for solid and liquids. Barium swallow shows corkscrew esophagus. Q.1: What is the diagnosis of the above case? Ans: The presence of non-progressive dysphagia to both solids and liquids suggests the diagnosis of diffuse esophageal spasm. Q.2: What is characteristic findings of barium swallow in diffuse esophageal spasm? Ans: Corkscrew appearance. Q.3: Diffuse esophageal spasm is best diagnosed by? Ans: Manometry.

248

Manual of Clinical Cases in ENT and Head and Neck Surgery

CASE 3: FOREIGN BODY IN ESOPHAGUS

Presenting complaints: A child has swallowed a coin. X-ray neck: It shows a radio-opaque rounded foreign body (Fig. 37.2) at the level of cricopharyngeal sphincter.

Fig. 37.2: X-ray neck and chest (AP view) showing rounded radio-oipaque foreign body (coin) at cricopharynx

Q.1: How it will be removed? Ans: Esophagoscopy/hypopharyngoscopy under general anesthesia. Q.2: What is the commonest site of foreign body of upper digestive tract? Ans: Cricopharynx as it is the narrowest part of the alimentary tract except for the vermiform appendix. Q.3: What are the contraindications for esophagoscopy? Ans: • Trismus • Diseases of the cervical spine. • Receding mandible. • Aneurysm of aorta. • Advanced heart, liver or kidney diseases are relative contraindications. Q.4: What is the commonest cause of esophageal perforation? Ans: Rigid esophagoscopy. Q.5: What are the clinical manifestations in esophageal perforation? Ans: • Severe pain in the neck, chest or upper abdomen depending on the site. • Emphysema in the neck. • Tachycardia, dyspnea. • Escape of air result in mediastinal (crunch)—Hamman’s sign. • Fever.

249

Neck Abscess

38

Neck Abscess CASE 1: PARAPHARYNGEAL ABSCESS

Presenting complaints: A 35-year-old man presented with fever, trismus, enlarged the tonsil in left side. History of past illness: He had done removal of 3rd molar few days back for dental caries. Q.1: What is the provisional diagnosis? Ans: Parapharyngeal abscess. Fever, trismus (due to spasm of medial pterygoid muscle), medial displacement of tonsil with involvement of neck indicates the diagnosis of parapharyngeal abscess. The statement on 3rd molar extraction points to the possibility of dental cause of infection. Q.2: What are the complications of parapharyngeal abscess? Ans: Jugular vein thrombosis, carotid artery rupture, lower cranial palsy with Horner’s syndrome, rupture of abscess and aspiration, laryngeal edema. Q.3: What is anatomy of parapharyngeal space? Ans: Parapharyngeal space is also called lateral pharyngeal space, extends from skull base to the hyoid bone. It is divided by styloid process into prestyloid and retrostyloid compartments. It was named by Mosher as Lincoln Highway of the neck after the famous Lincoln Highway of USA. The prestyloid compartment contains internal maxillary artery, lymph nodes, lingual nerve, auriculotemporal nerve and loose areolar tissue. The retrostyloid compartment contains internal carotid artery, internal jugular vein, IX, X, XI, XII cranial nerves and upper deep cervical lymph nodes. Q.4: What is Lemierre’s syndrome? Ans: Suppurative thrombophlebitis of the internal jugular vein is called Lemierre’s syndrome. It occurs secondary to deep neck space infection involving the carotid space. Patient presents with swelling and tenderness at the angle of jaw and along the sternocleido mastoid muscle with fever and sometimes present pulmonary emboli. Q.5: What are the etiologies of parapharyngeal abscess? Ans: • Peritonsillar abscess • Dental infections 249

250

• • • • • • •

Manual of Clinical Cases in ENT and Head and Neck Surgery

Penetrating injury to neck Upper aerodigestive tract infection/trauma. Congenital cycts and infected fistula. Iatrogenic injuries. Sialadenitis/parotitis Petrositis Acute suppurative otitis media

Microbiology Common organisms are: • • • •

Staphylococcus Alpha and beta streptococci Escherichia coli Others: Bacteroides, Peptostreptococcus, Fusobacterium, Actinomyces and Spirochaetes, etc.

CASE 2: RETROPHARYNGEAL ABSCESS

Presenting complaints: A 9-year-old boy presented with painful swallowing since 5 days with fever. Past history: History of trauma to the posterior pharyngeal wall by accidental entry of stick through oral cavity. Examination of the oral cavity and oropharynx: Smooth bulging of posterior pharyngeal wall. Oral cavity is normal. Q.1: What is the most possible diagnosis? Ans: Acute retropharyngeal abscess. Q.2: What is acute retropharyngeal abscess? Ans: This is an abscess of true retropharyngeal space of Gillette due to suppuration of the medial group of retropharyngeal lymph nodes. It is common on children. As larynx is high up in infants and early childhood, posterior pharyngeal wall cause laryngeal obstruction. Q.3: What are the differences between acute and chronic retropharyngeal abscesses? Ans: Acute retropharyngeal abscess 1. Commonly seen among infant and small children 2. It involves true retropharyngeal space, i.e. space of Gillette 3. Caused by pyogenic bacteria 4. Suppuration of lymph nodes 5. Acute onset 6. Dysphagia is acute in nature

Chronic retropharyngeal abscess 1. Seen in elderly and young adults 2. It involves prevertebral retropharyngeal space, i.e. behind the prevertebral fascia 3. Tubercular origin 4. Caries of vertebral bodies 5. Insidious onset 6. Chronic dysphagia 7. Respiratory difficulty is uncommon (Contd...)

Neck Abscess Acute retropharyngeal abscess 7. Respiratory difficulty is commonly seen 8. Patient has high fever 9. Bulging is on one side of midline on the posterior pharyngeal wall 10. Signs of acute inflammation are present 11. Treatment includes transoral drainage urgently and antibiotics for pyogenic bacteria

251

Chronic retropharyngeal abscess 8. If fever is present, it is low grade 9. Midline bulging on the posterior pharyngeal wall 10. Not seen 11. Drainage, though rarely needed, through posterior triangle of neck. Anti-tubercular therapy is given

Q.4. What is radiograpghic features of retropharyngeal abscess? Ans: Radiograph of soft tissue lateral view neck shows widening of prevertebral space with gas formation (Fig. 38.1). To diagnose retropharyngeal abscess, the prevertebral soft tissue shadow should exceed width of body of C2 (or should be more than 7 mm) in both children and adults opposite C2. Opposite C6, the soft tissue shadow should exceed 14 mm in children less than 15 years and 22 mm in adults.

Fig. 38.1: X-ray neck lateral view showing retropharyngeal abscess

CASE 3: TB CERVICAL LYMPHADENITIS

Presenting complaints: A 42-year-old man presenting with progressive swelling in the upper part of the lateral side of the neck since 3 months. Setting: OPD. Neck examination: The neck swelling was matted and mildly painful. There is an opening in the center of a few mass over which the skin is thin and mild bluish coloration. General examination: The patient is well oriented with time, place and person. Pulse—84/minute, blood pressure—120/70 mmHg.

252

Manual of Clinical Cases in ENT and Head and Neck Surgery

Q.1: What is the provisional diagnosis? Ans: Tuberculous cervical lymphadenitis with cold abscess. Q.2: What are the characteristic clinical presentations in favor of tuberculosis? Ans: Swelling consists of matted neck nodes, softening at the center of few swelling (cold abscess), swelling has no signs of acute inflammation. Q.3: What are the stages of tubercular cervical lymphadenitis? Ans: Stage of discrete lymph node enlargement, stage of matting (due to periadenitis), stage of cold abscess and stage of ulcer formation or sinus development. Q.4: What is cold abscess? Ans: An infected lymph node casemates and convert into pus. This procedure is very slow and does not stimulate excessive hyperemia. So the tempretaure of the skin over the mass is normal, so called cold abscess. As the waxy sheath around the bacilli is present, necrosis to the tissue is less than the surface of rapidly multiplying bacteria, so the type of cellular response is less acute. Q.5: What is collar-stud abscess? Ans: A tubercular abscess penetrates deep fascia and comes superficially into subcutaneous tissue. It has two compartment, one on either side of the deep fascia connected by small central tract. The shape resembles a collar-stud, hence called collarstud abscess. Q.6: What are the investigations in tubercular cervical lymphadenopathy? Ans: Blood for TLC, DC and ESR. X-ray chest PA view for lung lesion, Mantoux test, pus culture and smear for AFB, FNAC, sputum examination for AFB, excisional biopsy. Q.7: What is the treatment of tubercular cervical lymphadenitis? Ans: Antitubercular drugs for 9–12 months, repeated aspiration from cold abscess. Residula lymph node excision. CASE 4: LUDWIG’S ANGINA

Presenting complaints: A 23-year-old boy presented with swelling in the upper part of the neck at right side since five days. Patient has mild fever and drooling of saliva and dysphagia. Setting: ENT OPD. ENT examination: Right submandibular and submental regions are swollen and tender. Floor of the mouth is also swollen. There is caries tooth of right lower second molar. General physical examination: Within normal limit. Q.1: What is the possible diagnosis? Ans: Ludwig’s angina (Fig. 38.2).

Neck Abscess

253

Fig. 38.2: Ludwig’s angina

Q.2: What are the criteria for diagnosing Ludwig’s angina? Ans: The criteria are: • Rapidly progressive cellulitis; not an abscess. • Develops along fascial planes with direct extension, does not involve lymphatic spread. • Does not involve submandibular gland or lymph nodes. • Involves both sublingual and submaxillary spaces and is usually bilateral. Q.3: What are etiologies for Ludwig’s angina? Ans: • Dental infections: It accounts for 80% of the cases. • Submandibular sialadenitis. • Injury to oral mucosa. • Fractures of the mandible. Q.4: What are bacteriology in Ludwig’s angina? Ans: Mixed infections including both aerobes and anaerobes are common. Alpha hemolytic streptococci, staphylococci and bacteroids groups are common. Q.5: What are the classical clinical presentations in Ludwig’s angina? Ans: The clinical features in Ludwig’s angina are: • Initially unilateral and progresses to bilateral. • Pain, odynophagia and voice change. • Fever, malaise, neck rigidity and trismus. • Swollen floor of mouth which compromising the airway. • Patient posture is more erect and tachypneic. • Indirect laryngoscopy is not possible. • An attempt for doing intubation, precipitate the airway obstruction. • Submandibular and submental regions are tense, swollen and tender.

254

Manual of Clinical Cases in ENT and Head and Neck Surgery

• Floor of the mouth is tense, indurated, massive mucosal swelling which protrudes over the edges of the lower teeth. • No fluctuation is seen. • Tongue is pushed superiorly. Q.6: What is the treatment of Ludwig’s angina? Ans: The treatments of Ludwig’s angina are: • Early stage: Hospitalization, IV antibiotics, supportive measures. Extraction of infected tooth if identifiable. • Late stage: If airway compromised—tracheostomy in sitting position. Incision runs along the mandibular arch involving both the submandibular regions. IV antibiotics like pencillins, cephalosporins and metronidazole. CASE 5: PERITONSILLAR ABSCESS (QUINSY)

Presenting complaints: A 7-year-old male child presenting with right side peritonsillar abscee and trismus. Q.1: What is the best treatment in the above case? Ans: Immediate transoral abscess drainage. Quinsy is a surgical emergency and need drainage as early as possible. No other treatment except drainage of the abscess can relieve the symptoms. Antibiotics, analgesics and antipyretics serve as supportive treatment. Q.2: What is the cause of trismus in quinsy? Ans: Trismus is due to spasm of pterygoid muscles. Q.3: It is better to aspirate a quinsy before incision and drainage, Why? Ans: Aspiration confirms the presence of an abscess and so prevents an unnecessary I/D. It is less painful and hence patient compliance will be more especially in children. Above all in the event of a rare internal carotid artery aneurysm or vascular tumor of the parapharyngeal space masquerading as quinsy, so I/D may be catastrophic and this may be averted by a prior aspiration. Q.4: When will you advise tonsillectomy in peritonsillar abscess? Ans: Tonsillectomy is done after 6 to 8 weeks after treatment of quinsy; this is called interval tonsillectomy. Interval tonsillectomy is now advised only after the 2nd attack of quinsy or after 1st attack of quinsy in a patient with history of recurrent tonsillitis. CASE 6: PARAPHARYNGEAL ABSCESS

Presenting complaints: A patient who had undergone tooth extraction for dental caries, presents with swelling in posterior part of the middle 1/3rd of the sternocleidomastoid muscle and tonsil is pushed medially. Q.1: What is the most possible diagnosis? Ans: Parapharyngeal abscess. History of tooth extraction for dental caries, tonsil being pushed medially and swelling in the posterior part of middle 1/3rd of sternocleidomastoid indicated parapharyngeal abscess.

Neck Abscess

255

Q.2: What are the synonyms of the parapharyngeal abscess? Ans: Pharyngomaxillary abscess or lateral pharyngeal abscess. Q.3: What is Vincent’s angina? Ans: It is also called Trench mouth or ulcerative gingivitis. The organisms causing Vincent’s angina are Borellia vincentii (spirochete) and Bacillus fusiformis (anaerobe). Patients present with necrotizing gingivitis, i.e. gums are covered with necrotic membrane. They also present with bleeding gums and cervical lymphadenopathy. CASE 7: TUBERCULAR LYMPHADENITIS

Presenting complaints: A 35-year-old man presents with swelling at the right upper neck since one month. On examination: He had mild tenderness. There was no fluctuation on the swelling. FNAC: Presence of caseation with periadenitis favors the diagnosis of TB lymphadenitis. Q.1: What is tubercular lymphadenitis? Ans: Tubercular lymphadenitis commonly affects upper deep jugular nodes in the neck. Initial stage is enlargement of multiple nodes followed by matting due to periadenitis. This may further progress to caseation followed by rupture and cold abscess formation. Q.2: What is cold abscess? Ans: Cold abscess is termed cold because the swelling is without heat, redness, pain or fever. Q.3: How to manage the case of tubercular lymphadenitis? Ans: All patients with tubercular lymphadenitis should be screened for pulmonary tuberculosis. WHO multidrug regimen for TB is followed (2 months treatment with isoniazid, rifampicin, pyrazinamide, ethambutol +4 months continuation therapy with isoniazid and rifampicin). Nonresponders or MDRI cases are referred to dedicated TB centers for treatment. Q.4: What is the histopathological picture of tubercular lymphadenitis? Ans: Microscopically, tuberculous lymph node is characterized by granulomas with caseation consisting of lymphocytes, epithelioid cells and Langerhans, giant cells.

Index Acoustic neuroma 34, 63, 64 Acoustic trauma 77 Acute epiglottitis 225 laryngitis 229 laryngotracheobronchitis 226 sinusitis 116 suppurative otitis media 27 tonsillitis 162 Adductor dysphonia 234 Adenoid facies 168 hypertrophy 168 Adenoidectomy 168 Alderman’s nerve 3 Allergic fungal sinusitis 113 rhinitis 108 salute 108 Angioneurotic edema 227 Ankyloglossia 158 Anterior ethmoidal nerve syndrome 95 Antrochoanal polyp 119 Antrum threshold angle 45 Aphthous stomatitis 153 Arnold’s nerve 3 Atrophic rhinitis 130 Atticoantral disease 33 BAHA 66 Balloon sinuplasty 123 Bat ear 8, 20 Battle’s sign 49 Behçet’s syndrome 154, 159 Bell’s palsy 47 phenomenon 47 sign 47 Benign paroxysmal positional vertigo (BPPV) 80 Bjork flap 218 Blue mantles of Manasse 51

Bocca’s sign 214 Botulinum toxin 235 BPPV 84 Brainstem evoked response audiometry (BERA) 73 Bridge 45 Browne’s area 104

Canal of Huguier 11 Carcinoma of larynx 217 the tongue 181 Cardiac achalasia 247 Carhart’s notch 51 Cartilaginous pointer 49 Cavernous sinus thrombosis 115 Cerumen 21 Chevalier Jackson’s sign 214 Choanal atresia 90 Cholesteatoma 11 Chorda tympani nerve 11 Chronic suppurative otitis media 30 tonsillitis 163 Cluster headache 138 Cochlear implant 67 COG 46 Cogan’s syndrome 61 Cold abscess 252, 255 Cold spatula test 95 Collar-stud abscess 252 Cone of light 9 Congenital subglottic stenosis 222, 223 Cortical mastoidectomy 43 Cottle’s line 96 test 95 Cotton wool test 95 Crescent sign 120 Cricopharynx 246, 248 Cricothyrotomy 218 Crouzon syndrome 69 257

Crowe-Beck test 38 CSF rhinorrhea 143 Cystic hygroma 195, 198

Darrier’s line 108 Dennie-Morgan line 108 Dentigerous cyst 137 Dercum’s disease 199 Dermoid cyst 92 Deviated nasal septum 94 Diagnostic nasal endoscopy 112 Diffuse esophageal spasm 247 Diplacusis 62 Dix-Hallpike test 84 Donaldson’s line 43 Double target sign 144 Dysphonia 234 Dysphonia plica ventricularis 240 Eagle’s syndrome 141 Earmould 67 Empty nose syndrome 94 Encephalocele 87 Endaural incision 6 Endoscopic sinus surgery 121 Epistaxis 103 Epstein-Barr virus 167, 211 Epulis 156 Epworth sleepiness scale 175 Erythroleukoplakia 184 Esthesioneuroblastoma 135 Eustachian tube dysfunction 25 Exostosis 17 External auditory canal 3 Facial bridge 44 palsy 12 recess 10, 45 ridge 44 Fallopian canal 12 Fissure of Santorini 4

258

Manual of Clinical Cases in ENT and Head and Neck Surgery

Foramen of Huschke 4 Fordyce’s spot 154 Foreign body bronchus 231 esophagus 246 nose 146 Fossa incudes 10 Frenzel maneuver 12 Functional aphonia 205 endoscopic sinus surgery 121, 122 Fungal ball 113 Furuncle 14

Gamma knife surgery

65

Gerlach tonsil 12 Glaserian fissure 11 Glioma 122 Glomus tumor 57, 58, 64 Glossopharyngeal neuralgia 158 Glottic carcinoma 242 Gradenigo syndrome 37 Griesinger’s sign 38

Hand, foot and mouth disease 160 Hearing aid 66 loss 70 Hebra nose 126 Heimlich maneuver 218 Heimlich’s maneuver 230 Heller’s operation 247 Hemangioma 201 Hitzelberger’s sign 3 Hoarseness 232 Holman-Miller sign 209 Horner’s syndrome 212 Hot tonsillectomy 165 Hyposmia 101 Hypothyroidism 233

Ignition 239 Incisura terminalis present 6 Infant larynx 222 Infant with respiratory distress 223 Infectious mononucleosis 167 Intensity modulated radiotherapy 212 Interval tonsillectomy 165

Inverted papilloma 134 Irwin Moore sign 178 Isthmus 4

Juvenile laryngeal papillomatosis 243 nasopharyngeal angiofibroma 105

Kashima operation 207 Kiesselbach’s plexus 103 Killian’s polyp 120 Kocher’s septum 9, 44 test 200

Mondini’s dysplasia 66 Mucormycosis 106, 127 Myiasis of nose 148 Myringoplasty 40 Myringotomy 28, 169

Nasal furunculosis 89 septal perforation 99 septum 94 vestibulitis 89 Nasopharyngeal carcinoma 211 Noise induced hearing loss 77

Laryngeal web 222 Laryngocele 195 Laryngomalacia 221 Larynx 236 Lateral sinus thrombophlebitis 38 Lautenslager’s operation 131 Lederman’s classification 136 Lemierre’s syndrome 249 Lermoyez syndrome 59 Leukoplakia 184 Lingual thyroid 185 Lipoma 194 Little’s area 103 Ludwig’s angina 252

Obersteiner-Redlich zone 63 Obstructive sleep apnea 175 Ohngren’s classification 136 Oral submucosal fibrosis 155 Oropharyngeal malignancy 183 Osler-Rendu-Weber’s syndrome 106 Osteoma 17 Osteomeatal complex 118 Ostmann’s pad of fat 12 Otalgia 5 Otitis externa 13 media with effusion 168 Otomycosis 15 Otosclerosis 34 Ototoxicity 71, 78

MacEwen’s triangle 6, 9 Malignancy pyriform fossa 215 Malignant otitis externa 20 Malignant rhinosporidiosis 125 Mastoiditis 14 Maxillary carcinoma 136 McGovern’s technique 91 Meniere’s disease 34, 59, 62 Meniett apparatus 60 Meningoencephalocele 87, 121 Merciful anosmia 130 Microlaryngeal surgery 243 Microtia 19, 24 Migraine 139 Migraine related vertigo (MRV) 81, 82 Mikulicz cell 126 Mini FESS 123 Modified radical mastoidectomy 44

Pachydermia laryngis 238 Paracusis Willisii 50 Parapharyngeal abscess 249, 255 space 249 Patulous eustachian tube 12 Pediatric tracheostomy 220 Pemberton sign 196 Perforation of esophageal wall 246 Peritonsillar abscess 165 Petrositis 37 Phlep’s sign 58 Phonasthenia 238 Pinna 8 Plummer-Vinson syndrome 159 Plunging ranula 181 Ponticulus 45 Postcricoid carcinoma 214 Pott’s puffy tumor 114

259

Index Preauricular sinus 8 Presbyacusis 34, 71 Presbyosmia 101 Prussak’s space 11, 46 Pyriform fossa 215

Quinsy

166

Radicular cyst

137 Ramsay Hunt syndrome 16 Ranula 180 Recurrent laryngeal nerve paralysis 237 Referred otalgia 4 Reinke’s edema 233, 234, 237 space 237 Retropharyngeal abscess 250 Rhinitis caseosa 127 medicamentosa 111 sicca 127 Rhinolalia aperta 240 clausa 239 Rhinolith 149 Rhinophyma 88 Rhinoscleroma 126 Rhinosporidiosis 125 Rigid esophagoscopy 248 Rinne’s test 76 Romberg’s test 82 Rose position 171 Rosen’s incision 6 Russell bodies 126

Samter’s triad

110 Secretory otitis media 29 Semon’s law 203

Sensorineural hearing loss 32 Septal hematoma 98 Septoplasty 96 Sinodural angle 44 Sinus of Morgagni 213 tympani 45 Sinusitis 115 Sistrunk’s operation 188 Sluder’s neuralgia 95, 100 Stapedial reflex 10 Stapedotomy 52 Steeple sign 228 Stridor 231 Stuttering 239 Stylagia 141 Subglottic hemangioma 222 stenosis 222 Subiculum 45 Superficial parotidectomy 193

Tapir nose 126 TB larynx 233 Temporal arteritis 140 Tension headache 139 Thumb sign 226 Thyroglossal cyst 187 fistula 198 Thyroid nodule 196 Tinnitus 52 Tobey-Ayer test 38 Tongue tie 158 Tonsillar carcinoma 174 Tonsillectomy 170, 254 Total laryngectomy 242 Tracheoesophageal fistula 246 Tracheostomy 217

Trautmann’s triangle 9, 44 Treacher-Collins syndrome 66 Trigeminal neuralgia 141 Trismus 254 Trotter’s method 104 syndrome 213 triad 211, 213 Tubercular lymphadenitis 255 otitis media 35 Tuberculosis of larynx 227 Tullio’s phenomenon 62 Tuning fork tests 51 Tympanic membrane 8 Tympanogram 77

Usher syndrome

68

Valsalva

82 Van der Hoeve syndrome 53 Vasomotor rhinitis 110 Verrucous carcinoma 156, 184 Vertigo 52 Vidian neurectomy 111 Vincent’s angina 160, 255 Vocal cord paralysis 203 nodules 232

Waardenburg syndrome 66, 68 Wax 3 Webner’s test 76 Wegener’s granulomatosis 99, 128 What 216 Wilde’s incision 7 Winkler’s nodule 4 Wittmack’s operation 131 Woodruff’s area 104